Nbme 24 Blank

You might also like

Download as pdf or txt
Download as pdf or txt
You are on page 1of 201

hi Comprehensive Basic Science Self-Assessment - Google Chrome x

i starttest.com/ITDVersions/19.3.0.1/ITDStart.aspx?SVC=3f50c3c0-1874-400e-af3a-5036e79dec9f

Exam Section 1: Item 1 of 50 National Board of Medical Examiners Time Remaining:


• Mark Comprehensive Basic Science Self-Assessment 4 hr 57 min 11 sec

1. A 40-year-old woman with a 25-year history of type 1 diabetes mellitus has a serum creatinine concentration of 2.5 mg/dl. A creatinine clearance test is ordered to determine the
glomerular filtration rate . This test is limited, compared to an inulin clearance test, because of which of the following properties of creatinine?

CJ A) Actively pumped into the distal convoluted tubule


B) Crosses freely into the loop of Henle
C) Reabsorbed into the collecting duct
U D) Secreted by the proximal tubule

~ ~ p , ,.,,.
Next Lab Values Calculator Review Help Pause
hi Comprehensive Basic Science Self-Assessment - Google Chrome x
i starttest.com/ITDVersions/19.3.0.1/ITDStart.aspx?SVC=3f50c3c0-1874-400e-af3a-5036e79dec9f

Exam Section 1: Item 2 of 50 National Board of Medical Examiners Time Remaining:


• Mark Comprehensive Basic Science Self-Assessment 4 hr 57 min 19 sec

2. A 10-year-old boy who was adopted from the Democratic Republic of Congo 2 weeks ago is brought to the physician for an initial examination. He appears slim, has thin extremities,
and is in no distress. His temperature is 37 .3°C (99.1 °F), pulse is 100/min, respirations are 20/min, and blood pressure is 118/68 mm Hg. The lungs are clear, and heart sounds are
normal. The abdomen is soft and nontender, and there is no hepatosplenomegaly. There are four 0.5-mm, firm , nontender nodules: two over the right iliac crest, one on the left thigh,
and one on the left knee. Examination of a skin snip from one of the nodules shows microfilariae. Which of the following is the most likely vector of this patient's infection?

A) Black fly
B) Bodylouse
u C) Cu/ex species mosquito
U D) lxodes species tick
E) Reduviid bug

, ~ ~ p , ,.,,.
Previous Next Lab Values Calculator Review Help Pause
hi Comprehensive Basic Science Self-Assessment - Google Chrome x
i starttest.com/ITDVersions/19.3.0.1/ITDStart.aspx?SVC=3f50c3c0-1874-400e-af3a-5036e79dec9f

Exam Section 1: Item 3 of 50 National Board of Medical Examiners Time Remaining:


• Mark Comprehensive Basic Science Self-Assessment 4 hr 57 min 28 sec

3. A 3-year-old girl is brought to the physician for a well-child examination. She has had a flat purplish lesion that measures 2.1 cm in diameter on the right side of her face since birth.
The lesion shows large cavernous vascular channels. Which of the following is the most likely outcome of the lesion?

CJ A) Concurrent congenital heart disease


B) Local invasion
C) Malignant degeneration
U D) Necrosis and scarring
E) Spontaneous regression

, ~ ~ p , ,.,,.
Previous Next Lab Values Calculator Review Help Pause
hi Comprehensive Basic Science Self-Assessment - Google Chrome x
i starttest.com/ITDVersions/19.3.0.1/ITDStart.aspx?SVC=3f50c3c0-1874-400e-af3a-5036e79dec9f

Exam Section 1: Item 4 of 50 National Board of Medical Examiners Time Remaining:


• Mark Comprehensive Basic Science Self-Assessment 4 hr 57 min 49 sec

4. At 2:00 AM , after sleeping for 5 hours, a 32-year-old man with type 2 diabetes mellitus has a decrease in serum glucose concentration to 37 mg/dl. In response to the hypoglycemia,
cortisol secreted by the adrenal cortex induces the synthesis of which of the following enzymes in the adrenal medulla?

CJ A) Acetyl-CoA carboxylase
B) Homocysteine methyltransferase
C) Methionine adenosyltransferase
U D) Methylmalonyl-CoA racemase
E) Phenylethanolamine N-methyltransferase

, ~ ~ p , ,.,,.
Previous Next Lab Values Calculator Review Help Pause
hi Comprehensive Basic Science Self-Assessment - Google Chrome x
i starttest.com/ITDVersions/19.3.0.1/ITDStart.aspx?SVC=3f50c3c0-1874-400e-af3a-5036e79dec9f

Exam Section 1: Item 5 of 50 National Board of Medical Examiners Time Remaining:


• Mark Comprehensive Basic Science Self-Assessment 4 hr 57 min 59 sec

5. A 25-year-old woman takes an overdose of barbiturates and is found unresponsive several hours later by her husband. She is comatose and undergoes endotracheal intubation and
placement on a mechanical ventilator. Three weeks later, she opens her eyes spontaneously and has roving conjugate eye movements. She has no purposeful responses to stimuli.
Brain stem reflexes remain intact. An EEG is diffusely slow without evidence of cortical reactivity. The patient has signed a living will and, consistent with its directives, her husband
requests that mechanical ventilation be discontinued. Which of the following is the most appropriate next step in patient care?

A) Extubate the patient and discontinue mechanical ventilation; do cardiopulmonary resuscitation if her heart stops beating
B) Extubate the patient and discontinue mechanical ventilation; make no attempt to do cardiopulmonary resuscitation in case of cardiac or respiratory failure
u C) Extubate the patient and discontinue mechanical ventilation; reintubate the patient if spontaneous respirations cease
U D) Obtain a court order to maintain mechanical ventilation
E) Refer the case to the hospital ethics committee

, ~ ~ p , ,.,,.
Previous Next Lab Values Calculator Review Help Pause
hi Comprehensive Basic Science Self-Assessment - Google Chrome x
i starttest.com/ITDVersions/19.3.0.1/ITDStart.aspx?SVC=3f50c3c0-1874-400e-af3a-5036e79dec9f

Exam Section 1: Item 6 of 50 National Board of Medical Examiners Time Remaining:


• Mark Comprehensive Basic Science Self-Assessment 4 hr 58 min 9 sec

6. A 63-year-old woman undergoes operative repair of a leaking berry aneurysm in the circle of Willis. Two days later, a CT scan of the head shows a widening of the subarachnoid
space. Which of the following changes in cerebrospinal fluid is the most likely cause of this finding?

CJ A) Decreased absorption by the choroid plexus


B) Decreased movement through the arachnoid villi
C) Decreased movement through the cerebral aqueduct
U D) Increased absorption by the choroid plexus
E) Increased movement through the arachnoid villi
F) Increased production by the choroid plexus

, ~ ~ p , ,.,,.
Previous Next Lab Values Calculator Review Help Pause
hi Comprehensive Basic Science Self-Assessment - Google Chrome x
i starttest.com/ITDVersions/19.3.0.1/ITDStart.aspx?SVC=3f50c3c0-1874-400e-af3a-5036e79dec9f

Exam Section 1: Item 7 of 50 National Board of Medical Examiners Time Remaining:


• Mark Comprehensive Basic Science Self-Assessment 4 hr 58 min 34 sec

7. A 23-year-old woman is brought to the emergency department because of shortness of breath for 2 weeks. Her respirations are 28/min. Physical examination shows no other
abnormalities. Laboratory studies show:
Serum
Na+ 135 mEq/L
K+ 4.0 mEq/L
c1- 110 mEq/L
Hco3- 15 mEq/L
Arterial pH on room air 7.25
Urine
Sodium 20 mEq/L
Chlorine 30 mEq/L
Potassium 15 mEq/L

Which of the following is the most likely diagnosis?

A) Alcoholic ketoacidosis
B) Crohn disease
CJ C) Diabetic ketoacidosis
D) Lactic acidosis
E) Renal tubular acidosis
F) Salicylate poisoning

, ~ ~ p , ,.,,.
Previous Next Lab Values Calculator Review Help Pause
hi Comprehensive Basic Science Self-Assessment - Google Chrome x
i starttest.com/ITDVersions/19.3.0.1/ITDStart.aspx?SVC=3f50c3c0-1874-400e-af3a-5036e79dec9f

Exam Section 1: Item 8 of 50 National Board of Medical Examiners Time Remaining:


• Mark Comprehensive Basic Science Self-Assessment 4 hr 56 min 59 sec

8. A 30-year-old woman comes to the physician because of a 2-week history of right flank pain. Her blood pressure is 168/98 mm Hg. Physical examination shows a 7-cm mass that is
palpated in the right side of the abdomen. A CT scan shows a mass in the retroperitoneal space on the right side that is compressing the renal artery. If the perfusion pressure to the
affected kidney is decreased, but glomerular filtration rate and renal plasma flow remain unchanged, which of the following mechanisms mediates the autoregulation in the renal
arterioles in this patient?

A) Decreased afferent and efferent arteriolar resistance


B) Decreased afferent arteriolar resistance
u C) Decreased efferent arteriolar resistance
U D) Increased afferent and efferent arteriolar resistance
E) Increased afferent arteriolar resistance

, ~ ~ p , ,.,,.
Previous Next Lab Values Calculator Review Help Pause
hi Comprehensive Basic Science Self-Assessment - Google Chrome x
i starttest.com/ITDVersions/19.3.0.1/ITDStart.aspx?SVC=3f50c3c0-1874-400e-af3a-5036e79dec9f

Exam Section 1: Item 9 of 50 National Board of Medical Examiners Time Remaining:


• Mark Comprehensive Basic Science Self-Assessment 4 hr 56 min 52 sec

9. A male newborn is delivered in the hospital at 40 weeks' gestation to a 31-year-old primigravid woman following an uncomplicated spontaneous vaginal delivery. The mother
immigrated to the USA from Brazil at 20 weeks' gestation. At 8 weeks' gestation, she was evaluated because of a 1-month history of low-grade fever, marked fatigue, and diffuse
lymphadenopathy. Her symptoms resolved spontaneously, and she received no treatment. At that time, test results for Epstein-Barr virus and cytomegalovirus were negative. The
mother's only medication was a prenatal vitamin. The newborn is 33 cm (13 in; 25th percentile) long and weighs 2438 g (5 lb 6 oz; 35th percentile); head circumference is 31 cm (12
in; 3rd percentile). Temperature is 37.0°C (98.6°F), pulse is 120/min, respirations are 18/min, and blood pressure is 80/50 mm Hg. Eye examination shows chorioretinitis. CT scan of
the head shows hydrocephalus and cranial calcifications. This newborn's congenital infection was most likely acquired by the mother via which of the following modes of
transmission?

U A) Contact with fomites


B) Ingestion of undercooked meat
C) Inhalation of air droplets
u D) Kissing
E) Tick bite

, ~ ~ p , ,.,,.
Previous Next Lab Values Calculator Review Help Pause
hi Comprehensive Basic Science Self-Assessment - Google Chrome x
i starttest.com/ITDVersions/19.3.0.1/ITDStart.aspx?SVC=3f50c3c0-1874-400e-af3a-5036e79dec9f

Exam Section 1: Item 10 of 50 National Board of Medical Examiners Time Remaining:


• Mark Comprehensive Basic Science Self-Assessment 4 hr 56 min 44 sec

10. A 53-year-old woman comes to the physician because of a 2-week history of fainting precipitated by exercise and intermittent substernal chest pain that is relieved by rest. Her blood
pressure is 120/80 mm Hg . Cardiac examination shows a prominent left ventricular impulse. A late-peaking, harsh midsystolic murmur is heard best at the second right intercostal
space with radiation to the carotid arteries bilaterally. An ECG shows evidence of left ventricular hypertrophy. Angiography shows no coronary artery lesions. Which of the following is
the best explanation for the syncope?

A) Coronary artery compression by hypertrophic left ventricular myocardium


B) Coronary artery spasm
u C) Fixed cardiac output in spite of increased demand
U D) Mitral valve prolapse
E) Vasopressor-induced hypotension

, ~ ~ p , ,.,,.
Previous Next Lab Values Calculator Review Help Pause
hi Comprehensive Basic Science Self-Assessment - Google Chrome x
i starttest.com/ITDVersions/19.3.0.1/ITDStart.aspx?SVC=3f50c3c0-1874-400e-af3a-5036e79dec9f

Exam Section 1: Item 11 of 50 National Board of Medical Examiners Time Remaining:


• Mark Comprehensive Basic Science Self-Assessment 4 hr 56 min 36 sec

11. A 4 7-year-old man comes to the physician because of abdominal enlargement and rectal bleeding for 3 days. He drinks approximately twelve 12-ounce cans of beer daily. Physical
examination shows scleral icterus and bleeding internal hemorrhoids. Increased pressure in which of the following veins is the most likely cause of the hemorrhoids in this patient?

CJ A) Inferior rectal vein


B) Inferior vena cava
C) Internal pudenda! vein
U D) Middle rectal vein
E) Superior rectal vein

, ~ ~ p , ,.,,.
Previous Next Lab Values Calculator Review Help Pause
hi Comprehensive Basic Science Self-Assessment - Google Chrome x
i starttest.com/ITDVersions/19.3.0.1/ITDStart.aspx?SVC=3f50c3c0-1874-400e-af3a-5036e79dec9f

Exam Section 1: Item 12 of 50 National Board of Medical Examiners Time Remaining:


• Mark Comprehensive Basic Science Self-Assessment 4 hr 56 min 27 sec

12. A new screening test for colon cancer is done in 86 patients with colon cancer and 124 persons who do not have colon cancer. The results are shown:
Colon Cancer
Present Absent
Positive 67 6 73
Test Results
Negative 19 118 137
86 124 210

The specificity of the test is which of the following?

A) 67/73 = 92%
U B) 67/86 = 78%
C) 118/124 = 95%
D) 118/137 = 86%
E) 137/210 = 65%

, ~ ~ p , ,.,,.
Previous Next Lab Values Calculator Review Help Pause
hi Comprehensive Basic Science Self-Assessment - Google Chrome x
i starttest.com/ITDVersions/19.3.0.1/ITDStart.aspx?SVC=3f50c3c0-1874-400e-af3a-5036e79dec9f

Exam Section 1: Item 13 of 50 National Board of Medical Examiners Time Remaining:


• Mark Comprehensive Basic Science Self-Assessment 4 hr 56 min 13 sec

13. A clinical study is designed to evaluate the association of caffeine consumption and pancreatic cancer. In this study, alpha is set at 0.05 and beta at 0.10. Wh ich of the following best
describes the likelihood of missing an association between caffeine consumption and pancreatic cancer in this study?

CJ A) 5%
B) 10%
C) 85%
U D) 90%
E) 95%

, ~ ~ p , ,.,,.
Previous Next Lab Values Calculator Review Help Pause
hi Comprehensive Basic Science Self-Assessment - Google Chrome x
i starttest.com/ITDVersions/19.3.0.1/ITDStart.aspx?SVC=3f50c3c0-1874-400e-af3a-5036e79dec9f

Exam Section 1: Item 14 of 50 National Board of Medical Examiners Time Remaining:


• Mark Comprehensive Basic Science Self-Assessment 4 hr 56 min 5 sec

14. A 68-year-old woman comes to the emergency department 2 hours after she vomited blood. Three weeks ago, she began pharmacotherapy after experiencing a vertebral
compression fracture secondary to osteoporosis. Her temperature is 36.8°C (98.2°F), pulse is 104/min, respirations are 16/min, and blood pressure is 110/60 mm Hg. Physical
examination shows mild epigastric tenderness. Endoscopy shows multiple lower esophageal erosions. Which of the following medications is the most likely cause of the
hematemesis?

A) Alendronate
B) Calcitonin
u C) Calcium carbonate
U D) Conjugated estrogens
E) Raloxifene
0 F) Sodium fluoride

, ~ ~ p , ,.,,.
Previous Next Lab Values Calculator Review Help Pause
hi Comprehensive Basic Science Self-Assessment - Google Chrome x
i starttest.com/ITDVersions/19.3.0.1/ITDStart.aspx?SVC=3f50c3c0-1874-400e-af3a-5036e79dec9f

Exam Section 1: Item 15 of 50 National Board of Medical Examiners Time Remaining:


• Mark Comprehensive Basic Science Self-Assessment 4 hr 55 min 58 sec

15. A 33-year-old woman at 34 weeks' gestation has a grade 2/6 systolic ejection murmur heard at the second intercostal space. The remainder of the physical examination is
unremarkable. The best explanation for this finding is an increase in which of the following?

CJ A) Blood pressure
B) Peripheral vascular resistance
C) Pulmonary vascular resistance
U D) Pulse
E) Stroke volume

, ~ ~ p , ,.,,.
Previous Next Lab Values Calculator Review Help Pause
hi Comprehensive Basic Science Self-Assessment - Google Chrome x
i starttest.com/ITDVersions/19.3.0.1/ITDStart.aspx?SVC=3f50c3c0-1874-400e-af3a-5036e79dec9f

Exam Section 1: Item 16 of 50 National Board of Medical Examiners Time Remaining:


• Mark Comprehensive Basic Science Self-Assessment 4 hr 55 min 46 sec

16. A 27-year-old man sustains a spinal cord transection at C-8 in a skiing accident. In which of the following situations is he most likely to be able to achieve and sustain an erection?

A) Erotic dreams
U B) Penile stimulation
U C) Reading erotic literature
D) Testosterone injections
E) Viewing erotic material

, ~ ~ p , ,.,,.
Previous Next Lab Values Calculator Review Help Pause
hi Comprehensive Basic Science Self-Assessment - Google Chrome x
i starttest.com/ITDVersions/19.3.0.1/ITDStart.aspx?SVC=3f50c3c0-1874-400e-af3a-5036e79dec9f

Exam Section 1: Item 17 of 50 National Board of Medical Examiners Time Remaining:


• Mark Comprehensive Basic Science Self-Assessment 4 hr 55 min 37 sec

17. A 40-year-old man comes to the physician because of a 6-month history of constipation and weakness. He has a family history of
colorectal carcinoma in multiple members of both sexes. Physical examination shows no abnormalities. Laboratory studies show a
hemoglobin concentration of 10 g/dl, hematocrit of 32%, and mean corpuscular volume of 73 µm 3. A colonoscopy is done, and a lesion
is found. A photograph of a section of the colon removed during subsequent colectomy is shown. Analysis shows mutations in the
MHS2 gene. Which of the following is the most likely diagnosis?

U A) Familial adenomatous polyposis


B) Fanconi anemia
U C) HNPCC syndrome
D) Li-Fraumeni syndrome
E) Neurofibromatosis, type 1
F) Xeroderma pigmentosum

, ~ ~ p , ,.,,.
Previous Next Lab Values Calculator Review Help Pause
hi Comprehensive Basic Science Self-Assessment - Google Chrome x
i starttest.com/ITDVersions/19.3.0.1/ITDStart.aspx?SVC=3f50c3c0-1874-400e-af3a-5036e79dec9f

Exam Section 1: Item 18 of 50 National Board of Medical Examiners Time Remaining:


• Mark Comprehensive Basic Science Self-Assessment 4 hr 55 min 26 sec

18. A 22-year-old woman comes to the physician because of nausea and vomiting for 1 week. She has a 6-month history of headaches. She works as a data entry technician and has
recently found it difficult to focus on the text. She also has had trouble looking up from her desk at the clock on the wall without moving her head. Neurologic examination shows
marked vertical gaze palsy and impaired accommodation. Horizontal gaze is normal. An MRI of the brain is most likely to show a tumor at which of the following locations?

A) Arcuate nucleus
B) Frontal cortex
C) Pineal gland
D) Pituitary gland
E) Pontine paramedian reticular formation

, ~ ~ p , ,.,,.
Previous Next Lab Values Calculator Review Help Pause
hi Comprehensive Basic Science Self-Assessment - Google Chrome x
i starttest.com/ITDVersions/19.3.0.1/ITDStart.aspx?SVC=3f50c3c0-1874-400e-af3a-5036e79dec9f

Exam Section 1: Item 19 of 50 National Board of Medical Examiners Time Remaining:


• Mark Comprehensive Basic Science Self-Assessment 4 hr 55 min 7 sec

19. In a study of antibiotic resistance, a strain of Escherichia coli resistant to ampicillin but sensitive to streptomycin is cultured with a strain of Salmonella enteritidis sensitive to
ampicillin but resistant to streptomycin. After 4 hours of cocultivation, the broth is plated onto solid medium containing both ampicillin and streptomycin. S. enteritidis bacteria that are
resistant to both antibiotics grow at a frequency of one per 100 cells. Which of the following is the most likely mechanism of the acquisition of ampicillin resistance by the
S. enteritidis?

A) Conjugation
B) Gene duplication
U C) Point mutation
U D) Transduction
E) Transposition

, ~ ~ p , ,.,,.
Previous Next Lab Values Calculator Review Help Pause
hi Comprehensive Basic Science Self-Assessment - Google Chrome x
i starttest.com/ITDVersions/19.3.0.1/ITDStart.aspx?SVC=3f50c3c0-1874-400e-af3a-5036e79dec9f

Exam Section 1: Item 20 of 50 National Board of Medical Examiners Time Remaining:


• Mark Comprehensive Basic Science Self-Assessment 4 hr 54 min 59 sec

20. A 15-month-old boy is brought to the physician by his mother because of a 9-month history of recurrent bacterial infections. The patient has not had recurrent viral infections. He is at
the 10th percentile for length and weight. Physical examination shows multiple areas of honey yellow, crusted lesions over the lower extremities. A Gram stain of one of the lesions
shows many gram-positive cocci in clusters but no leukocytes. Laboratory studies show no abnormalities except for a leukocyte count of 30,000/mm3. This patient most likely has a
rare autosomal recessive disease leading to a lack of CD18 expression. The leukocytes would be deficient in which of the following characteristics?

A) Cytokine production
B) Helper T-cell function
u C) lmmunoglobulin gene rearrangement
U D) Killing of intracellular bacteria
E) Migration

, ~ ~ p , ,.,,.
Previous Next Lab Values Calculator Review Help Pause
hi Comprehensive Basic Science Self-Assessment - Google Chrome x
i starttest.com/ITDVersions/19.3.0.1/ITDStart.aspx?SVC=3f50c3c0-1874-400e-af3a-5036e79dec9f

Exam Section 1: Item 21 of 50 National Board of Medical Examiners Time Remaining:


• Mark Comprehensive Basic Science Self-Assessment 4 hr 54 min 52 sec

21. A 10-year-old girl is brought to the physician by her mother because of increased hair growth on her face and increased muscle mass during the past 3 months. Breast development
is Tanner stage 1, and pubic hair development is Tanner stage 5. Physical examination shows clitorimegaly. Pelvic examination shows a normal-appearing vagina. Serum studies
show:
Luteinizing hormone 0.8 mlU/mL (N=1.0-5.9)
Dehydroepiandrosterone 6.2 nmol/L (N=5.2-19.8)
Dehydroepiandrosterone sulfate 0.9 µmol/L (N=0.9-3.4)
Testosterone 14 nmol/L (N=0.2-0.7)

An unregulated increase in hormone production by which of the following types of cells is the most likely cause of this patient's hirsutism?

A) Adrenal zona fasciculata cells


B) Adrenal zona glomerulosa cells
C) Ovarian follicle cells
D) Ovarian Sertoli-Leydig cells
E) Pituitary acidophils
F) Pituitary basophils

, ~ ~ p , ,.,,.
Previous Next Lab Values Calculator Review Help Pause
hi Comprehensive Basic Science Self-Assessment - Google Chrome x
i starttest.com/ITDVersions/19.3.0.1/ITDStart.aspx?SVC=3f50c3c0-1874-400e-af3a-5036e79dec9f

Exam Section 1: Item 22 of 50 National Board of Medical Examiners Time Remaining:


• Mark Comprehensive Basic Science Self-Assessment 4 hr 54 min 45 sec

22. A 45-year-old man is brought to the emergency department 2 hours after the sudden onset of severe pain in the left flank that awakened him from sleep. He reports that the pain
moved over the next 30 minutes to the left lower abdomen and now is principally localized in the scrotum. Physical examination shows no palpable masses in the abdomen or the
scrotum. There is no rebound tenderness of the abdominal wall. Which of the following is the most likely cause of the pain?

A) Appendicitis
B) Carcinoma of the testis
C) Diverticulitis of the descending colon
D) Renal calculi in the left ureter
E) Testicular torsion

, ~ ~ p , ,.,,.
Previous Next Lab Values Calculator Review Help Pause
hi Comprehensive Basic Science Self-Assessment - Google Chrome x
i starttest.com/ITDVersions/19.3.0.1/ITDStart.aspx?SVC=3f50c3c0-1874-400e-af3a-5036e79dec9f

Exam Section 1: Item 23 of 50 National Board of Medical Examiners Time Remaining:


• Mark Comprehensive Basic Science Self-Assessment 4 hr 54 min 38 sec

23. A 38-year-old woman , gravida 4, para 4, comes to the physician because of a 6-month history of urinary incontinence when sneezing and coughing. Physical examination shows
normal-appearing female external genitalia. When the patient is asked to perform a Valsalva maneuver, leakage of urine from the urethra is detected. She is given instructions to
perform Kegel exercises to strengthen the muscles of the perineum. Which of the following muscles in this patient is most likely to remain unaffected by the exercises?

A) Bulbospongiosus
B) Deep transverse perinea! muscle
C) External urethral sphincter
D) Internal anal sphincter
E) lschiocavernosus

, ~ ~ p , ,.,,.
Previous Next Lab Values Calculator Review Help Pause
hi Comprehensive Basic Science Self-Assessment - Google Chrome x
i starttest.com/ITDVersions/19.3.0.1/ITDStart.aspx?SVC=3f50c3c0-1874-400e-af3a-5036e79dec9f

Exam Section 1: Item 24 of 50 National Board of Medical Examiners Time Remaining:


• Mark Comprehensive Basic Science Self-Assessment 4 hr 54 min 29 sec

24. A 1-week-old newborn is brought to the physician because of poor feeding , vomiting, and progressive lethargy during the past 4 days. She was born at term ; pregnancy, labor, and
delivery were uncomplicated, and she had no congenital anomalies. She is being breast-fed. She has a healthy 2-year-old brother; a sister died at the age of 10 days after a full-term
birth. Physical examination shows decreased muscle tone and poor responsiveness; reflexes are normal. Her serum bicarbonate concentration is 8 mEq/L, pH is 7.15, and plasma
ammonia concentration is 10 times the upper limit of normal. Which of the following is the most likely cause?

A) Mitochondrial disorder
B) Mucopolysaccharidosis disorder
u C) Organic acid metabolism disorder
U D) Renal tubular acidosis
E) X-linked leukodystrophy

, ~ ~ p , ,.,,.
Previous Next Lab Values Calculator Review Help Pause
hi Comprehensive Basic Science Self-Assessment - Google Chrome x
i starttest.com/ITDVersions/19.3.0.1/ITDStart.aspx?SVC=3f50c3c0-1874-400e-af3a-5036e79dec9f

Exam Section 1: Item 25 of 50 National Board of Medical Examiners Time Remaining:


• Mark Comprehensive Basic Science Self-Assessment 4 hr 54 min 23 sec

25. A 45-year-old man comes to the physician because of a 3-day history of fever and muscle pain. He recently went on a hunting expedition to northern Canada, where he consumed
polar bear meat. His temperature is 39.5°C (103.1 °F). Physical examination shows periorbital edema and muscle tenderness. Which of the following tests is most likely to establish
the diagnosis?

A) Eosinophil count
B) Examination of the stool for ova
C) Gram stain of the stool
D) X-ray survey of large muscular masses
E) Muscle biopsy

, ~ ~ p , ,.,,.
Previous Next Lab Values Calculator Review Help Pause
hi Comprehensive Basic Science Self-Assessment - Google Chrome x
i starttest.com/ITDVersions/19.3.0.1/ITDStart.aspx?SVC=3f50c3c0-1874-400e-af3a-5036e79dec9f

Exam Section 1: Item 26 of 50 National Board of Medical Examiners Time Remaining:


• Mark Comprehensive Basic Science Self-Assessment 4 hr 54 min 16 sec

26. A 71-year-old woman with non-Hodgkin lymphoma is diagnosed with lymphomatous meningitis. Her serum creatinine concentration is 2.3 mg/dl. A course of intrathecal
methotrexate is planned. Intravenous administration of which of the following adjuvant agents is most appropriate for this patient?

CJ A) Folic acid
B) Leucovorin
C) Vitamin B 1 (thiamine)
U D) Vitamin 8 6 (pyridoxine)
E) Vitamin C

, ~ ~ p , ,.,,.
Previous Next Lab Values Calculator Review Help Pause
hi Comprehensive Basic Science Self-Assessment - Google Chrome x
i starttest.com/ITDVersions/19.3.0.1/ITDStart.aspx?SVC=3f50c3c0-1874-400e-af3a-5036e79dec9f

Exam Section 1: Item 27 of 50 National Board of Medical Examiners Time Remaining:


• Mark Comprehensive Basic Science Self-Assessment 4 hr 54 min 10 sec

27. A 55-year-old woman is brought to the physician by her husband because of a change in behavior during the past 2 days. Her husband says, "Yesterday, she didn't recognize a
picture of her own mother." Neurologic examination shows that she has an inability to recognize objects unless she touches them or hears the sound that they make. These findings
most likely indicate a lesion involving the area supplied by which of the following arteries?

A) Anterior cerebral
B) Anterior choroidal
C) Lenticulostriate
D) Posterior cerebral
E) Thalamoperforating

, ~ ~ p , ,.,,.
Previous Next Lab Values Calculator Review Help Pause
hi Comprehensive Basic Science Self-Assessment - Google Chrome x
i starttest.com/ITDVersions/19.3.0.1/ITDStart.aspx?SVC=3f50c3c0-1874-400e-af3a-5036e79dec9f

Exam Section 1: Item 28 of 50 National Board of Medical Examiners Time Remaining:


• Mark Comprehensive Basic Science Self-Assessment 4 hr 54 min 5 sec

28. A 24-year-old woman who has diffuse toxic goiter (Graves disease) undergoes a partial thyroidectomy. Several months later, she develops muscle cramps, tetany, and
hypocalcemia. Which of the following sets of laboratory findings in serum is most consistent with these findings?

Parathyroid
Phosphate Hormone 25-Hydroxyvitamin D
A) i i l
B) i l normal
C) i l l
D) l i normal
E) l l l

, ~ ~ p , ,.,,.
Previous Next Lab Values Calculator Review Help Pause
hi Comprehensive Basic Science Self-Assessment - Google Chrome x
i starttest.com/ITDVersions/19.3.0.1/ITDStart.aspx?SVC=3f50c3c0-1874-400e-af3a-5036e79dec9f

Exam Section 1: Item 29 of 50 National Board of Medical Examiners Time Remaining:


• Mark Comprehensive Basic Science Self-Assessment 4 hr 53 min 56 sec

29. A 56-year-old man has a 3-year history of progressive memory loss followed by aphasia, visuospatial disorientation, and inappropriate behavior. His father died in a nursing home at
age 65, 6 years after the onset of progressive dementia. His 55-year-old brother has had some difficulty remembering recent events and recalling familiar names. The patient dies 5
years after the onset of these symptoms. At autopsy, examination of the brain shows cerebral atrophy, predominantly in the mesial temporal and parietal cortices; in these areas
there are neurofibrillary tangles , neuritic plaques, and marked neuronal loss. This disorder most likely indicates a mutation in a gene encoding for which of the following molecules?

A) Amyloid A protein
B) ~ 2-Microglobulin
u C) Neurofilament protein
U D) Presenilin

, ~ ~ p , ,.,,.
Previous Next Lab Values Calculator Review Help Pause
hi Comprehensive Basic Science Self-Assessment - Google Chrome x
i starttest.com/ITDVersions/19.3.0.1/ITDStart.aspx?SVC=3f50c3c0-1874-400e-af3a-5036e79dec9f

Exam Section 1: Item 30 of 50 National Board of Medical Examiners Time Remaining:


• Mark Comprehensive Basic Science Self-Assessment 4 hr 53 min 48 sec

30. A 60-year-old woman is receiving cisplatin therapy for advanced transitional cell bladder cancer. She develops paresthesias, and the medication
is stopped. The medication likely damaged the largest cells in the region labeled "B" in the drawing of the spinal cord shown. The damage to
these cells would most likely lead to which of the following motor signs?

A) Babinski sign
B) Clonus
U C) Fibrillations
0 D) Hyporeflexia
U E) Muscle atrophy

, ~ ~ p , ,.,,.
Previous Next Lab Values Calculator Review Help Pause
hi Comprehensive Basic Science Self-Assessment - Google Chrome x
i starttest.com/ITDVersions/19.3.0.1/ITDStart.aspx?SVC=3f50c3c0-1874-400e-af3a-5036e79dec9f

Exam Section 1: Item 31 of 50 National Board of Medical Examiners Time Remaining:


• Mark Comprehensive Basic Science Self-Assessment 4 hr 53 min 41 sec

31. A 30-year-old woman comes to the physician for follow-up treatment of chronic renal failure and normocytic, normochromic anemia. Following the administration of recombinant
human erythropoietin, her hemoglobin concentration increases in part due to increased activity of 5-aminolevulinic acid (o-ALA) synthase in erythrocytes. This enzyme regulates the
reaction involving the condensation of which of the following compounds?

A) Acetate and hexacarboxylic porphyrinogen


B) Ferrous ion and protoporphyrin
C) Glycine and succinyl CoA
D) Lysine and o-ALA
E) Zinc and porphobilinogen

, ~ ~ p , ,.,,.
Previous Next Lab Values Calculator Review Help Pause
hi Comprehensive Basic Science Self-Assessment - Google Chrome x
i starttest.com/ITDVersions/19.3.0.1/ITDStart.aspx?SVC=3f50c3c0-1874-400e-af3a-5036e79dec9f

Exam Section 1: Item 32 of 50 National Board of Medical Examiners Time Remaining:


• Mark Comprehensive Basic Science Self-Assessment 4 hr 53 min 35 sec

32. An investigator is studying an outbreak of Escherichia coli 0157:H7 among university students who had eaten at the on-site cafeteria. The investigators administer a food
questionnaire to 100 students with E. coli 0157:H7 and 100 healthy control subjects. Two exposures emerge as possible concerns: cookies made from frozen commercial dough
(odds ratio=6.0; p<0.001) and milk from a local farm vendor (odds ratio=3.9; p<0.001 ). The exposure data is stratified. Results show:
Drank Milk Did Not Drink Milk
Patients Controls Patients Controls
72 36 8 4
Ate Cookies Yes
No 2 6 18 54
Odds ratio=1.0 Odds ratio=6.0

Ate Cookies Did Not Eat Cookies


Patients Controls Patients Controls
Yes 72 36 2 6
Drank Milk
No 8 4 18 54
Odds ratio=1 .0 Odds ratio=1.0

Which of the following conclusions can be drawn from these data?

A) Both milk and cookies are independently associated with E. coli cases
B) No confounding exists between milk and cookies
C) Only cookies are independently associated with E. coli cases
D) Only milk is independently associated with E. coli cases

, ~ ~ p , ,.,,.
Previous Next Lab Values Calculator Review Help Pause
hi Comprehensive Basic Science Self-Assessment - Google Chrome x
i starttest.com/ITDVersions/19.3.0.1/ITDStart.aspx?SVC=3f50c3c0-1874-400e-af3a-5036e79dec9f

Exam Section 1: Item 33 of 50 National Board of Medical Examiners Time Remaining:


• Mark Comprehensive Basic Science Self-Assessment 4 hr 53 min 28 sec

33. A 68-year-old woman comes to the physician because of a 9-month history of abdominal pain after meals; she also has had an 11-kg (24-lb) weight loss during this period. She says
she avoids eating because of cramping pain that lasts for approximately 1 hour after meals. She underwent coronary artery bypass grafting for angina 6 years ago. Current
medications include aspirin, atenolol, and simvastatin. Her pulse is 62/min, respirations are 16/min, and blood pressure is 160/90 mm Hg. Physical examination shows a thin body
frame and a soft, nontender abdomen; there are no masses and no organomegaly. A CT scan of the abdomen shows a 3-cm ectatic aorta. Which of the following pairs of arteries is
most likely involved in this patient's condition?

A) Celiac and inferior mesenteric


B) Inferior mesenteric and hepatic
C) Inferior mesenteric and superior mesenteric
lJ D) Inferior phrenic and celiac
U E) Superior mesenteric and hepatic

, ~ ~ p , ,.,,.
Previous Next Lab Values Calculator Review Help Pause
hi Comprehensive Basic Science Self-Assessment - Google Chrome x
i starttest.com/ITDVersions/19.3.0.1/ITDStart.aspx?SVC=3f50c3c0-1874-400e-af3a-5036e79dec9f

Exam Section 1: Item 34 of 50 National Board of Medical Examiners Time Remaining:


• Mark Comprehensive Basic Science Self-Assessment 4 hr 53 min 17 sec

34. A 12-year-old boy is brought to the physician by his mother because of a persistent rash around his nose and mouth for 4 months. He also has had a decreased appetite and an
unintentional 4.5-kg (10-lb) weight loss during the past 3 months. His mother states, "The rash developed shortly after he started at his new school. I wonder if he keeps getting into
something. I've tried hydrocortisone cream, but it didn't work." She adds, "And now his grades are falling, and he's not eating as much as he used to. I hope he's not depressed
because of the rash." The rash began as mild redness and then became dry and started to flake. The patient says there is no pain or discomfort with the rash. Physical examination
shows injection of both conjunctivae, an eczematous eruption extending inferiorly from the nasolabial fold to the upper lip, and an erythematous nasal mucosa that is friable with gold
stippling. When asked, he states that his mood is fine. Which of the following is the most likely cause of this patient's condition?

A) Atopic dermatitis
B) Impetigo
C) Inhalant abuse
() D) Mercury poisoning
u E) Wegener granulomatosis

, ~ ~ p , ,.,,.
Previous Next Lab Values Calculator Review Help Pause
hi Comprehensive Basic Science Self-Assessment - Google Chrome x
i starttest.com/ITDVersions/19.3.0.1/ITDStart.aspx?SVC=3f50c3c0-1874-400e-af3a-5036e79dec9f

Exam Section 1: Item 35 of 50 National Board of Medical Examiners Time Remaining:


• Mark Comprehensive Basic Science Self-Assessment 4 hr 53 min 10 sec

35. A 26-year-old man is brought to the emergency department 40 minutes after being involved in a motor vehicle collision. His blood pressure is 90/60 mm Hg. Systolic blood pressure
decreases 20 mm Hg with inspiration. Physical examination shows jugular venous distention. There are distant heart sounds. Which of the following is the most likely diagnosis?

CJ A) Hemothorax
B) Myocardial infarction
C) Pericardia! tamponade
U D) Pneumothorax
E) Pulmonary edema

, ~ ~ p , ,.,,.
Previous Next Lab Values Calculator Review Help Pause
hi Comprehensive Basic Science Self-Assessment - Google Chrome x
i starttest.com/ITDVersions/19.3.0.1/ITDStart.aspx?SVC=3f50c3c0-1874-400e-af3a-5036e79dec9f

Exam Section 1: Item 36 of 50 National Board of Medical Examiners Time Remaining:


• Mark Comprehensive Basic Science Self-Assessment 4 hr 53 min 3 sec

36. Forty of 100 attendees at a company Christmas party develop explosive, watery diarrhea followed by abdominal cramps and vomiting 12 to 48 hours after the party. About half of the
affected individuals also had headache. All of the affected individuals reported feeling better within several days. All of the affected individuals, and 6 of the 60 unaffected individuals,
consumed raw oysters. There was no other significant association between the consumption of specific foods and symptoms. Which of the following is the most likely causal
organism?

A) Bacillus cereus
B) Campy/obacter jejuni
u C) C/ostridium pertringens
U D) Staphylococcus aureus
E) Vibrio parahaemolyticus

, ~ ~ p , ,.,,.
Previous Next Lab Values Calculator Review Help Pause
hi Comprehensive Basic Science Self-Assessment - Google Chrome x
i starttest.com/ITDVersions/19.3.0.1/ITDStart.aspx?SVC=54cd3e5f-e4bf-49d0-8371-ba261251f2a1

Exam Section 2: Item 37 of 50 National Board of Medical Examiners Time Remaining:


• Mark Comprehensive Basic Science Self-Assessment 4 hr 55 min 34 sec

37. A 57-year-old woman requires intubation in the intensive care unit after developing progressive dyspnea and hypoxemia 45 minutes after the initiation of a transfusion of packed red
blood cells. The patient has cirrhosis and was admitted to the hospital because of melena; the transfusion was initiated after she was found to have a hemoglobin concentration of
6.4 g/dl. Medical history includes hepatitis C diagnosed at the age of 41 years. She has a 25-year history of illicit intravenous drug use. The patient is intubated and sedated. Her
temperature is 36.9°C (98.4 °F), pulse is 92/min , respirations are 16/min, and blood pressure is 94/58 mm Hg. There is no jugular venous distention. Bilateral basilar crackles are
heard on inspiration; there are no wheezes. Physical examination shows spider angiomata on the chest and abdomen, minimal ascites, and trace bilateral pedal edema. Chest x-ray
shows bilateral diffuse airspace disease without pleural effusions or cardiomegaly. This patient most likely developed which of the following types of transfusion reactions?

A) Acute hemolytic reaction


B) Acute lung injury
C) Anaphylactic reaction
() D) Rh incompatibility
u E) Serum sickness

, ~ ~ p , ,.,,.
Previous Next Lab Values Calculator Review Help Pause
hi Comprehensive Basic Science Self-Assessment - Google Chrome x
i starttest.com/ITDVersions/19.3.0.1/ITDStart.aspx?SVC=3f50c3c0-1874-400e-af3a-5036e79dec9f

Exam Section 1: Item 38 of 50 National Board of Medical Examiners Time Remaining:


• Mark Comprehensive Basic Science Self-Assessment 4 hr 52 min 26 sec

38. An investigator is conducting a study of a novel protein in an experimental animal model. It is found that this protein interferes with the normal structure of desmosomes (maculae
adherentes). Which of the following cell junctions in skin are most likely to be affected by the presence of this protein?

CJ A) Basal keratinocyte:lamina lucida


B) Basal keratinocyte:suprabasal keratinocyte
C) Granular keratinocyte:stratum corneum
U D) Lamina lucida:lamina densa
E) Melanocyte:basal keratinocyte

, ~ ~ p , ,.,,.
Previous Next Lab Values Calculator Review Help Pause
hi Comprehensive Basic Science Self-Assessment - Google Chrome x
i starttest.com/ITDVersions/19.3.0.1/ITDStart.aspx?SVC=3f50c3c0-1874-400e-af3a-5036e79dec9f

Exam Section 1: Item 39 of 50 National Board of Medical Examiners Time Remaining:


• Mark Comprehensive Basic Science Self-Assessment 4 hr 52 min 11 sec

39. An otherwise healthy 45-year-old man comes to the office because of a 10-year history of episodes of painful sores in his mouth that recur every few months. He has had no skin
lesions, genital or anal ulcers, gastrointestinal or joint problems, or fever. Physical examination shows the findings in the photograph. Which of the following is the most likely cause of
this patient's condition?

A) Aphthous ulcers
B) Herpangina
C) Herpes zoster
D) Secondary syphilis

, ~ ~ p , ,.,,.
Previous Next Lab Values Calculator Review Help Pause
hi Comprehensive Basic Science Self-Assessment - Google Chrome x
i starttest.com/ITDVersions/19.3.0.1/ITDStart.aspx?SVC=3f50c3c0-1874-400e-af3a-5036e79dec9f

Exam Section 1: Item 40 of 50 National Board of Medical Examiners Time Remaining:


• Mark Comprehensive Basic Science Self-Assessment 4 hr 52 min 3 sec

40. A 25-year-old woman is brought to the emergency department by her husband 30 minutes after losing consciousness at home. She had been home from work for the past 5 days
because of fever. She has a history of recurrent upper urinary tract infections. Her temperature is 39.4 °C (103°F), pulse is 135/min, and blood pressure is 82/48 mm Hg. Diffuse
crackles are heard over both lung fields. Arterial blood gas analysis on room air shows:
pH 7.18
Pco 2 32 mm Hg
Po 2 64 mm Hg

A blood culture grows gram-negative rods. A chest x-ray shows diffuse bilateral infiltrates. Which of the following is the most likely cause of this patient's pulmonary symptoms?

u A) Hypoventilation
B) Hypoxic vasoconstriction
C) Left ventricular failure
lJ D) Paradoxical thromboembolism
E) Pulmonary capillary leakage

, ~ ~ p , ,.,,.
Previous Next Lab Values Calculator Review Help Pause
hi Comprehensive Basic Science Self-Assessment - Google Chrome x
i starttest.com/ITDVersions/19.3.0.1/ITDStart.aspx?SVC=3f50c3c0-1874-400e-af3a-5036e79dec9f

Exam Section 1: Item 41 of 50 National Board of Medical Examiners Time Remaining:


• Mark Comprehensive Basic Science Self-Assessment 4 hr 51 min 56 sec

41. A 13-year-old girl who has a 6-year history of type 1 diabetes mellitus has had poor glucose control for the past 2 months. Previously, the diabetes was well controlled. Her parents
say that the stress of her illness is causing family problems and that they have been fighting with each other and their daughter, especially at meals. The parents argue with each
other while in the physician's office. The patient takes her insulin as prescribed but is having difficulty following the prescribed diet. Which of the following is the most appropriate
next step?

A) Advise the parents to stop bickering


B) Discuss further the impact of the patient's illness on the family
u C) Readjust the patient's sliding-scale insulin coverage
U D) Recommend individual psychotherapy for the patient
E) Refer both the patient and her parents to a dietician
0 F) Refer the patient to a dietician

, ~ ~ p , ,.,,.
Previous Next Lab Values Calculator Review Help Pause
hi Comprehensive Basic Science Self-Assessment - Google Chrome x
i starttest.com/ITDVersions/19.3.0.1/ITDStart.aspx?SVC=3f50c3c0-1874-400e-af3a-5036e79dec9f

Exam Section 1: Item 42 of 50 National Board of Medical Examiners Time Remaining:


• Mark Comprehensive Basic Science Self-Assessment 4 hr 51 min 51 sec

42. A 90-year-old man has a 1-week history of constipation and abdominal swelling. He is afebrile. A 4-cm mass is palpated in the right groin. The mass is not reducible, but moderately
increased tympanitic bowel sounds are audible over it. Which of the following is the most likely diagnosis?

CJ A) Appendiceal abscess
B) Carcinoma of the colon
C) Diverticulitis
U D) Fecal impaction
E) Incarcerated inguinal hernia

, ~ ~ p , ,.,,.
Previous Next Lab Values Calculator Review Help Pause
hi Comprehensive Basic Science Self-Assessment - Google Chrome x
i starttest.com/ITDVersions/19.3.0.1/ITDStart.aspx?SVC=3f50c3c0-1874-400e-af3a-5036e79dec9f

Exam Section 1: Item 43 of 50 National Board of Medical Examiners Time Remaining:


• Mark Comprehensive Basic Science Self-Assessment 4 hr 51 min 44 sec

43. A 45-year-old man with hypercholesterolemia (LDL 260 mg/dl) requires pharmacotherapy. A 3-hydroxy-3-methylglutaryl (HMG) CoA reductase inhibitor will be prescribed. Which of
the following is the mechanism of action of this treatment?

CJ A) Decreased synthesis of HDL cholesterol


B) Enhanced activity of lipoprotein lipase
C) Increased synthesis of bile acids
U D) Increased synthesis of LDL receptors
E) Sequestration of bile acids in the gastrointestinal tract

, ~ ~ p , ,.,,.
Previous Next Lab Values Calculator Review Help Pause
hi Comprehensive Basic Science Self-Assessment - Google Chrome x
i starttest.com/ITDVersions/19.3.0.1/ITDStart.aspx?SVC=3f50c3c0-1874-400e-af3a-5036e79dec9f

Exam Section 1: Item 44 of 50 National Board of Medical Examiners Time Remaining:


• Mark Comprehensive Basic Science Self-Assessment 4 hr 51 min 38 sec

44. A 19-year-old man has had weakness of the muscles of his neck and extremities for the past week. Bladder and bowel function is normal. Position sense is impaired. Which of the
following is the most likely diagnosis?

CJ A) Amyotrophic lateral sclerosis


B) Parkinson disease
C) Poliomyelitis
U D) Polyneuropathy
E) Syringomyelia

, ~ ~ p , ,.,,.
Previous Next Lab Values Calculator Review Help Pause
hi Comprehensive Basic Science Self-Assessment - Google Chrome x
i starttest.com/ITDVersions/19.3.0.1/ITDStart.aspx?SVC=3f50c3c0-1874-400e-af3a-5036e79dec9f

Exam Section 1: Item 45 of 50 National Board of Medical Examiners Time Remaining:


• Mark Comprehensive Basic Science Self-Assessment 4 hr 51 min 30 sec

45. A 72-year-old woman with coronary artery disease comes to the physician because of a 2-month history of progressive angina symptoms with exertion. Five months ago, the patient
underwent stent placement for significant stenoses of the proximal anterior interventricular (left anterior descending) and right coronary arteries. She remained symptom-free for
3 months. Her pulse is 76/min and regular, and blood pressure is 135/85 mm Hg. An ECG at rest shows no abnormalities; an exercise stress test shows ST-segment changes in
leads II, Ill , and aVF. Which of the following is the most likely cause of this patient's recurrent angina symptoms?

A) Aneurysm of the right coronary artery


B) Dissection of the right coronary artery
u C) Neointima formation in the right coronary stent
U D) Thrombosis of the right coronary stent
E) Vasospasm in the stented right coronary artery

, ~ ~ p , ,.,,.
Previous Next Lab Values Calculator Review Help Pause
hi Comprehensive Basic Science Self-Assessment - Google Chrome x
i starttest.com/ITDVersions/19.3.0.1/ITDStart.aspx?SVC=3f50c3c0-1874-400e-af3a-5036e79dec9f

Exam Section 1: Item 46 of 50 National Board of Medical Examiners Time Remaining:


• Mark Comprehensive Basic Science Self-Assessment 4 hr 51 min 24 sec

46. A 63-year-old woman develops flank pain, tachycardia , and hypotension while receiving a blood transfusion in the intensive care unit. Her urine is wine-colored. Which of the
following is the most likely cause of these findings?

CJ A) ABO incompatibility
B) Bacterial contamination of the transfused blood
C) Extravascular hemolysis
U D) Immediate (type I) hypersensitivity reaction
E) Rh incompatibility

, ~ ~ p , ,.,,.
Previous Next Lab Values Calculator Review Help Pause
hi Comprehensive Basic Science Self-Assessment - Google Chrome x
i starttest.com/ITDVersions/19.3.0.1/ITDStart.aspx?SVC=3f50c3c0-1874-400e-af3a-5036e79dec9f

Exam Section 1: Item 47 of 50 National Board of Medical Examiners Time Remaining:


• Mark Comprehensive Basic Science Self-Assessment 4 hr 51 min 17 sec

47. A 55-year-old woman comes to the physician for a routine health maintenance examination. She has never smoked cigarettes. Physical examination shows no abnormalities. A
chest x-ray shows a 2-cm left pulmonary nodule (coin lesion). Microscopic examination of the excised lesion shows nests and cords of regular cells with uniform round nuclei and
rare mitoses. Focally, the cells are arranged in trabeculae, and scattered rosettes are seen. Electron microscopy shows dense-core, membrane-bound granules in the cytoplasms of
these cells. These cells are most likely to closely resemble which of the following types of normal respiratory tract cells?

A) Alveolar capillary endothelial cell


B) Alveolar macrophage
u C) Chondrocyte
U D) Ciliated columnar epithelial cell
E) Goblet cell
0 F) Neuroendocrine cell
u G) Squamous epithelial cell
H) Type I pneumocyte

, ~ ~ p , ,.,,.
Previous Next Lab Values Calculator Review Help Pause
hi Comprehensive Basic Science Self-Assessment - Google Chrome x
i starttest.com/ITDVersions/19.3.0.1/ITDStart.aspx?SVC=3f50c3c0-1874-400e-af3a-5036e79dec9f

Exam Section 1: Item 48 of 50 National Board of Medical Examiners Time Remaining:


• Mark Comprehensive Basic Science Self-Assessment 4 hr 51 min 11 sec

48. A 40-year-old man who recently immigrated to the USA from Kenya is brought to the emergency department because of a 3-week
history of diarrhea. He appears disoriented and cachectic. He recalls zero of three objects after 5 minutes. A photograph of the head,
neck, and upper chest is shown. Similar findings are seen over the dorsal aspects of the hands. The diagnosis of a vitamin deficiency
is made. The deficient vitamin in this patient is part of a cofactor that contains which of the following components?

A) Adenine
U B) Alanine
C) Apolipoprotein
U D) Arachidonic acid
E) Aspartate

, ~ ~ p , ,.,,.
Previous Next Lab Values Calculator Review Help Pause
hi Comprehensive Basic Science Self-Assessment - Google Chrome x
i starttest.com/ITDVersions/19.3.0.1/ITDStart.aspx?SVC=3f50c3c0-1874-400e-af3a-5036e79dec9f

Exam Section 1: Item 49 of 50 National Board of Medical Examiners Time Remaining:


• Mark Comprehensive Basic Science Self-Assessment 4 hr 51 min 4 sec

49. A 64-year-old man undergoes surgical repair of an abdominal aortic aneurysm. During the repair, the left testicular artery is ligated. Anastomotic supply from which of the following
arteries will maintain adequate arterial supply to the left testis in this patient?

CJ A) Artery of the ductus deferens


B) Inferior vesical artery
C) Obturator artery
U D) Posterior scrotal artery
E) Superficial circumflex iliac artery

, ~ ~ p , ,.,,.
Previous Next Lab Values Calculator Review Help Pause
hi Comprehensive Basic Science Self-Assessment - Google Chrome x
i starttest.com/ITDVersions/19.3.0.1/ITDStart.aspx?SVC=3f50c3c0-1874-400e-af3a-5036e79dec9f

Exam Section 1: Item 50 of 50 National Board of Medical Examiners Time Remaining:


• Mark Comprehensive Basic Science Self-Assessment 4 hr 50 min 57 sec

50. A full-term male newborn has lethargy, poor feeding , and vomiting 48 hours after delivery. He appears flaccid. Serum studies show increased concentrations of ammonia and orotic
acid. Which of the following labeled steps in the metabolic pathway is the most likely cause of these findings?

~
H 4 + 2ATP 2ADP+ P i
N~
_ \ ./ , Carbamoyl- - -
_,l phosphate ~----..._,

C02 \:;:>
N-Acetylglutamate
Ornithine Citrulline
CoA-SHl(!l
Acetyl CoA + glutamate

Mitochondrial inner membrane

Mitochondrial outer membrane

Ornithine Citrulline

AMP + PPj

Arginine Argininosucc in ate

Fumarate

0 A) B) u C) D) E)

, ~ ~ p , ,.,,.
Previous Next Lab Values Calculator Review Help Pause
hf Comprehensive Basic Science Self-Assessment - Google Chrome x
i starttest.com/ITDVersions/19.3.0.1/ITDStart.aspx?SVC=54cd3e5f-e4bf-49d0-8371-ba261251f2a1

Exam Section 2: Item 1 of 50 National Board of Medical Examiners Time Remaining:


• Mark Comprehensive Basic Science Self-Assessment 4 hr 59 min 50 sec

1. An 11-year-old boy is brought to the emergency department by ambulance with his mother 30 minutes after she was unable to awaken him from a nap. The mother states that he was
recently diagnosed with type 1 diabetes mellitus and she has been having difficulty adjusting his insulin regimen. He is unconscious, and his breath smells of acetone. He is
responsive only to sharp pain. A rapid test shows a serum glucose concentration greater than 500 mg/dl. Which of the following best explains this patient's altered state of
consciousness?

A) Altered affinity of oxygen from hemoglobin A 1c


B) Diminished transport of glucose across the blood-brain barrier
u C) Excess free fatty acids in the cerebrospinal fluid
U D) Inability of neurons to perform glycolysis
E) Intracellular and extracellular dehydration

~ ~ p, ,.,,.
Next Lab Values Calculator Review Help Pause
hi Comprehensive Basic Science Self-Assessment - Google Chrome x
i starttest.com/ITDVersions/19.3.0.1/ITDStart.aspx?SVC=54cd3e5f-e4bf-49d0-8371-ba261251f2a1

Exam Section 2: Item 2 of 50 National Board of Medical Examiners Time Remaining:


• Mark Comprehensive Basic Science Self-Assessment 4 hr 59 min 35 sec

2. A researcher hypothesizes that exposure to more than 50 µg/L of arsenic in drinking water is associated with an increased risk for development of a common cancer compared with
persons whose drinking water contains less than 5 µg/L of arsenic. Which of the following is the most time-efficient experimental design to investigate this hypothesis?

CJ A) Case-control study
B) Case series study
C) Cohort study
U D) Open-label, dose-ranging study
E) Randomized, double-blind, controlled trial

, ~ ~ p , ,.,,.
Previous Next Lab Values Calculator Review Help Pause
hi Comprehensive Basic Science Self-Assessment - Google Chrome x
i starttest.com/ITDVersions/19.3.0.1/ITDStart.aspx?SVC=54cd3e5f-e4bf-49d0-8371-ba261251f2a1

Exam Section 2: Item 3 of 50 National Board of Medical Examiners Time Remaining:


• Mark Comprehensive Basic Science Self-Assessment 4 hr 59 min 26 sec

3. A 43-year-old woman with a 2-year history of labile hypertension comes to the physician for a follow-up examination. Her blood pressure has ranged from 123/84 mm Hg to 165/ 112
mm Hg since being diagnosed. One day after propranolol therapy is started, she develops a severe headache and her blood pressure is 214/132 mm Hg. Serum and urine
catecholamine concentrations are four times the upper limit of the reference ranges. A CT scan shows a 2-cm nodule in the right adrenal gland. Which of the following is most likely
responsible for the increased blood pressure in this patient after propranolol therapy was started?

A) Anxiety reaction
B) Impaired degradation of catecholamines
u C) Increased adrenocorticotropic hormone release
U D) Infarction of the adrenal tumor
E) Stimulation of aldosterone production
0 F) Unopposed a-adrenergic tone

, ~ ~ p , ,.,,.
Previous Next Lab Values Calculator Review Help Pause
hi Comprehensive Basic Science Self-Assessment - Google Chrome x
i starttest.com/ITDVersions/19.3.0.1/ITDStart.aspx?SVC=54cd3e5f-e4bf-49d0-8371-ba261251f2a1

Exam Section 2: Item 4 of 50 National Board of Medical Examiners Time Remaining:


• Mark Comprehensive Basic Science Self-Assessment 4 hr 59 min 17 sec

4. A 4-year-old girl is brought to the physician because of a 6-month history of foul-smelling stools and failure to gain weight appropriately. She is at the 3rd percentile for height and
weight. Abdominal examination shows distention. Stool analysis shows an increased fat concentration. Serum concentrations of anti-endomysial and anti-tissue transglutaminase
antibodies are increased. Which of the following findings in the gastrointestinal tract is the most likely cause of the disorder in this patient?

A) Eosinophilic infiltration of the mucosa of the small intestine


B) Erosions of epithelial cells in the small intestine
C) Lactase deficiency
D) Periodic acid-Schiff (PAS)-positive granules in macrophages in the lamina propria of the small intestine
E) Villous atrophy

, ~ ~ p , ,.,,.
Previous Next Lab Values Calculator Review Help Pause
hi Comprehensive Basic Science Self-Assessment - Google Chrome x
i starttest.com/ITDVersions/19.3.0.1/ITDStart.aspx?SVC=54cd3e5f-e4bf-49d0-8371-ba261251f2a1

Exam Section 2: Item 5 of 50 National Board of Medical Examiners Time Remaining:


• Mark Comprehensive Basic Science Self-Assessment 4 hr 59 min 10 sec

5. A 26-year-old man comes to the physician for a follow-up examination. One month ago, his total serum cholesterol concentration was 325 mg/dl, and the physician prescribed a 3-
hydroxy-3-methylglutaryl (HMG) CoA reductase inhibitor. At the time of the present examination, his total serum cholesterol concentration is 235 mg/dl. Which of the following is the
most likely cause of the decrease?

A) Decreased absorption of dietary cholesterol


B) Decreased conversion of chylomicrons to VLDL
C) Increased concentration of HDL in plasma
D) Increased intracellular esterification of cholesterol
E) Increased number of LDL receptors on hepatocytes
F) Increased synthesis of apolipoprotein B-100

, ~ ~ p , ,.,,.
Previous Next Lab Values Calculator Review Help Pause
hi Comprehensive Basic Science Self-Assessment - Google Chrome x
i starttest.com/ITDVersions/19.3.0.1/ITDStart.aspx?SVC=54cd3e5f-e4bf-49d0-8371-ba261251f2a1

Exam Section 2: Item 6 of 50 National Board of Medical Examiners Time Remaining:


• Mark Comprehensive Basic Science Self-Assessment 4 hr 59 min 1 sec

6. A 28-year-old man with AIDS has fever, headache, and lethargy. Examination shows papilledema and nuchal rigidity. A mucicarmine-
stained smear of his cerebrospinal fluid is shown. Which of the following is the most likely primary site of infection with this organism?

A) Eyes
B) Gastrointestinal tract
C) Lungs
CJ D) Middle ear
E) Skin

, ~ ~ p , ,.,,.
Previous Next Lab Values Calculator Review Help Pause
hi Comprehensive Basic Science Self-Assessment - Google Chrome x
i starttest.com/ITDVersions/19.3.0.1/ITDStart.aspx?SVC=54cd3e5f-e4bf-49d0-8371-ba261251f2a1

Exam Section 2: Item 7 of 50 National Board of Medical Examiners Time Remaining:


• Mark Comprehensive Basic Science Self-Assessment 4 hr 58 min 51 sec

7. A 68-year-old woman with acute myelogenous leukemia begins treatment with standard induction chemotherapy. After completion of the regimen , her leukocyte count is 1200/mm 3
with an absolute neutrophil count of 475/mm 3. Which of the following is the most appropriate pharmacotherapy?

CJ A) Bcr-Abl tyrosine kinase inhibitor


B) Erythropoietin
C) Granulocyte-macrophage colony-stimulating factor
U D) lnterleukin-2 (IL-2)
E) Thrombopoietin
F) Transforming growth factor-13

, ~ ~ p , ,.,,.
Previous Next Lab Values Calculator Review Help Pause
hi Comprehensive Basic Science Self-Assessment - Google Chrome x
i starttest.com/ITDVersions/19.3.0.1/ITDStart.aspx?SVC=54cd3e5f-e4bf-49d0-8371-ba261251f2a1

Exam Section 2: Item 8 of 50 National Board of Medical Examiners Time Remaining:


• Mark Comprehensive Basic Science Self-Assessment 4 hr 58 min 43 sec

8. A 70-year-old man is brought to the emergency department because of a 1-week history of progressive shortness of breath with mild exertion, cough, fatigue, and weakness. The
shortness of breath frequently awakens him from sleep and is partly relieved by sitting or standing. He has a 20-year history of hypertension treated with diuretics and ACE inhibitors;
however, he does not always take his medications. His pulse is 85/min, respirations are 20/min, and blood pressure is 180/110 mm Hg. Crackles are heard on auscultation. A chest x-
ray shows cardiomegaly. Echocardiography shows decreased left ventricular systolic function. Which of the following patterns best characterizes this patient's cardiovascular
variables at this time?

Left Ventricular
Stroke End-Diastolic Cardiac
Volume Volume Output
0 A) i i i
B) i i l
C) i l i
U D) i l l
E) l i i
U F) l i l
G) l l i
H) l l l

, ~ ~ p , ,.,,.
Previous Next Lab Values Calculator Review Help Pause
hi Comprehensive Basic Science Self-Assessment - Google Chrome x
i starttest.com/ITDVersions/19.3.0.1/ITDStart.aspx?SVC=54cd3e5f-e4bf-49d0-8371-ba261251f2a1

Exam Section 2: Item 9 of 50 National Board of Medical Examiners Time Remaining:


• Mark Comprehensive Basic Science Self-Assessment 4 hr 58 min 35 sec

9. A 52-year-old woman comes to the physician because of severe pain in her midback for 2 weeks. She has a history of left breast cancer treated with mastectomy and chemotherapy.
Physical examination shows tenderness to palpation over the thoracic spine. An MRI of the back shows metastases in thoracic vertebral bodies. Which of the following veins is the
most likely path for tumor cells in the breast to obtain access to the vertebral bodies in this patient?

A) Axillary
B) lntercostal
C) Internal thoracic
D) Lateral thoracic
E) Subclavian

, ~ ~ p , ,.,,.
Previous Next Lab Values Calculator Review Help Pause
hi Comprehensive Basic Science Self-Assessment - Google Chrome x
i starttest.com/ITDVersions/19.3.0.1/ITDStart.aspx?SVC=54cd3e5f-e4bf-49d0-8371-ba261251f2a1

Exam Section 2: Item 10 of 50 National Board of Medical Examiners Time Remaining:


• Mark Comprehensive Basic Science Self-Assessment 4 hr 58 min 28 sec

10. An 80-year-old woman with poorly controlled type 2 diabetes mellitus and hypertension comes to the physician for a follow-up examination. She also has a 5-year history of chronic
kidney disease with increasing proteinuria and a recent creatinine clearance of 30 ml/min. Her blood pressure is 165/95 mm Hg. Physical examination shows edema over the lower
extremities. Pulmonary and cardiac examinations show no abnormalities. Compared with a healthy individual, which of the following sets of laboratory findings is most likely in this
patient?

Serum
Hematocrit Parathyroid Hormone Serum Calcitriol
A) Increased normal increased
B) Increased normal decreased
C) Normal increased decreased
D) Normal decreased normal
E) Decreased increased decreased
U F) Decreased decreased decreased

, ~ ~ p , ,.,,.
Previous Next Lab Values Calculator Review Help Pause
hi Comprehensive Basic Science Self-Assessment - Google Chrome x
i starttest.com/ITDVersions/19.3.0.1/ITDStart.aspx?SVC=54cd3e5f-e4bf-49d0-8371-ba261251f2a1

Exam Section 2: Item 11 of 50 National Board of Medical Examiners Time Remaining:


• Mark Comprehensive Basic Science Self-Assessment 4 hr 58 min 20 sec

11. An experimental study is conducted to examine the function of the pro-opiomelanocortin gene. The gene is found to encode an mRNA from which protein products are generated.
This protein serves as the precursor of adrenocorticotropic hormone and f3-lipotropin. Which of the following processes is necessary in order to generate these hormones?

CJ A) Alternative splicing
B) Gene rearrangement
C) Methylation of the gene
U D) Post-transcriptional modification
E) Post-translational modification

, ~ ~ p , ,.,,.
Previous Next Lab Values Calculator Review Help Pause
hi Comprehensive Basic Science Self-Assessment - Google Chrome x
i starttest.com/ITDVersions/19.3.0.1/ITDStart.aspx?SVC=54cd3e5f-e4bf-49d0-8371-ba261251f2a1

Exam Section 2: Item 12 of 50 National Board of Medical Examiners Time Remaining:


• Mark Comprehensive Basic Science Self-Assessment 4 hr 58 min 12 sec

12. A 24-year-old man receives an injection of histamine in the brachia! artery. Which of the following changes is most likely to occur in his forearm in response to the histamine?

A) Decreased capillary filtration rate


U B) Decreased interstitial hydrostatic pressure
U C) Decreased interstitial oncotic pressure
D) Increased arteriole resistance
E) Increased lymph flow

, ~ ~ p , ,.,,.
Previous Next Lab Values Calculator Review Help Pause
hi Comprehensive Basic Science Self-Assessment - Google Chrome x
i starttest.com/ITDVersions/19.3.0.1/ITDStart.aspx?SVC=54cd3e5f-e4bf-49d0-8371-ba261251f2a1

Exam Section 2: Item 13 of 50 National Board of Medical Examiners Time Remaining:


• Mark Comprehensive Basic Science Self-Assessment 4 hr 58 min 5 sec

13. A 24-year-old woman at 28 weeks' gestation is brought to the emergency department because of a 3-hour history of shortness of breath. She has asthma, but she currently takes no
medications. Her pulse is 100/min, respirations are 32/min, and blood pressure is 120/83 mm Hg. Physical examination shows the use of accessory muscles of respiration. Diffuse
inspiratory and expiratory wheezes are heard. An inhaled ~ 2-adrenergic agonist is administered. Which of the following findings is most likely in this patient after this therapy?

A) Bradycardia
B) Diaphoresis
C) Dry mouth
D) Pallor
E) Tremor

, ~ ~ p , ,.,,.
Previous Next Lab Values Calculator Review Help Pause
hi Comprehensive Basic Science Self-Assessment - Google Chrome x
i starttest.com/ITDVersions/19.3.0.1/ITDStart.aspx?SVC=54cd3e5f-e4bf-49d0-8371-ba261251f2a1

Exam Section 2: Item 14 of 50 National Board of Medical Examiners Time Remaining:


• Mark Comprehensive Basic Science Self-Assessment 4 hr 57 min 58 sec

14. A 4-year-old boy is brought to the emergency department by his mother 6 hours after she noticed that his urine was red. He is otherwise feeling well. Fifteen days ago, the patient
had a sore throat, fever, and cough. His mother thought he had the flu and treated him symptomatically with rest and analgesics, and his status improved until now. His temperature
is 38.5°C (101.3°F), pulse is 110/ min, respirations are 22/min, and blood pressure is 100/50 mm Hg. Physical examination shows normal breath and cardiac sounds, no
organomegaly, and 1 + lower extremity edema, bilaterally. Laboratory studies show:
Serum
Urea nitrogen 40 mg/dl
Creatinine 2 mg/dl
Albumin 3.6 g/dl
Urine
Color red-brown
Blood 3+
Protein 2+
Ketones negative
RBC 30-50/hpf
RBC casts few

Which of the following is the most likely diagnosis?

A) Membranous nephropathy
B) Minimal change disease
C) Papillary necrosis
D) Proliferative glomerulonephritis
E) Tubulointerstitial nephritis

, ~ ~ p , ,.,,.
Previous Next Lab Values Calculator Review Help Pause
hi Comprehensive Basic Science Self-Assessment - Google Chrome x
i starttest.com/ITDVersions/19.3.0.1/ITDStart.aspx?SVC=54cd3e5f-e4bf-49d0-8371-ba261251f2a1

Exam Section 2: Item 15 of 50 National Board of Medical Examiners Time Remaining:


• Mark Comprehensive Basic Science Self-Assessment 4 hr 57 min 52 sec

15. A 1616-g (3-lb 9-oz) male newborn is delivered to a 16-year-old primigravid patient at 28 weeks' gestation. He develops severe respiratory distress within 1 hour and is treated with
intubation and 100% inspired oxygen with continuous positive end-expiratory pressure. One month later, a chest x-ray shows persistently increased densities in all lung fields. He
develops pneumonia and dies. At autopsy, the lungs are firm and poorly aerated. Microscopic examination of lung tissue shows prominent interstitial fibrosis surrounding irregular,
dilated airspaces with bronchiolar metaplasia. Which of the following is the most likely diagnosis?

A) Bronchial asthma
B) Bron ch iectasis
u C) Bronchiolitis obliterans
U D) Bronchopulmonary dysplasia
E) Chronic bronchitis
0 F) Cystic adenomatoid malformation

, ~ ~ p , ,.,,.
Previous Next Lab Values Calculator Review Help Pause
hi Comprehensive Basic Science Self-Assessment - Google Chrome x
i starttest.com/ITDVersions/19.3.0.1/ITDStart.aspx?SVC=54cd3e5f-e4bf-49d0-8371-ba261251f2a1

Exam Section 2: Item 16 of 50 National Board of Medical Examiners Time Remaining:


• Mark Comprehensive Basic Science Self-Assessment 4 hr 57 min 45 sec

16. The 35-year-old woman indicated by the arrow has a family history of retinitis pigmentosa. Which of the following modes of inheritance can be
eliminated based on this pedigree?

A) Autosomal dominant
B) Autosomal recessive
C) Multifactorial
CJ D) X-linked recessive

, ~ ~ p , ,.,,.
Previous Next Lab Values Calculator Review Help Pause
hi Comprehensive Basic Science Self-Assessment - Google Chrome x
i starttest.com/ITDVersions/19.3.0.1/ITDStart.aspx?SVC=54cd3e5f-e4bf-49d0-8371-ba261251f2a1

Exam Section 2: Item 17 of 50 National Board of Medical Examiners Time Remaining:


• Mark Comprehensive Basic Science Self-Assessment 4 hr 57 min 37 sec

17. A 35-year-old man comes to the physician because of a 1-week history of severe daily headaches of sudden onset. The pain is often located around his left eye and is associated
with excessive tearing and redness of the eye. The headaches last from 30 minutes to 2 hours. He has had similar episodes during the past 7 years, often with periods of up to 1
year during which no headaches occur. Vital signs are normal. Funduscopic examination shows no abnormalities. Neurologic examination shows no focal findings. Which of the
following is the most likely diagnosis?

A) Acute meningitis
B) Cluster headache
u C) Idiopathic intracranial hypertension
U D) Migraine
E) Subarachnoid hemorrhage
0 F) Temporal arteritis
u G) Tension-type headache

, ~ ~ p , ,.,,.
Previous Next Lab Values Calculator Review Help Pause
hi Comprehensive Basic Science Self-Assessment - Google Chrome x
i starttest.com/ITDVersions/19.3.0.1/ITDStart.aspx?SVC=54cd3e5f-e4bf-49d0-8371-ba261251f2a1

Exam Section 2: Item 18 of 50 National Board of Medical Examiners Time Remaining:


• Mark Comprehensive Basic Science Self-Assessment 4 hr 57 min 31 sec

18. A 3-year-old boy is brought to the emergency department because of epigastric pain and bile-stained vomitus. The child had been well prior to this episode and has reached all
milestones normally. Which of the following developmental abnormalities is most likely to be the cause of this patient's problem?

CJ A) Annular pancreas
B) Esophageal atresia
C) Extrahepatic biliary atresia
U D) Hypertrophic pyloric stenosis

, ~ ~ p , ,.,,.
Previous Next Lab Values Calculator Review Help Pause
hi Comprehensive Basic Science Self-Assessment - Google Chrome x
i starttest.com/ITDVersions/19.3.0.1/ITDStart.aspx?SVC=54cd3e5f-e4bf-49d0-8371-ba261251f2a1

Exam Section 2: Item 19 of 50 National Board of Medical Examiners Time Remaining:


• Mark Comprehensive Basic Science Self-Assessment 4 hr 57 min 24 sec

19. A 7-year-old boy has metabolic acidosis and persistent phosphaturia. This patient most likely has a primary defect of which of the following labeled sites in the drawing of the
nephron shown?

U A) B) U C) D) E)

, ~ ~ p , ,.,,.
Previous Next Lab Values Calculator Review Help Pause
hi Comprehensive Basic Science Self-Assessment - Google Chrome x
i starttest.com/ITDVersions/19.3.0.1/ITDStart.aspx?SVC=54cd3e5f-e4bf-49d0-8371-ba261251f2a1

Exam Section 2: Item 20 of 50 National Board of Medical Examiners Time Remaining:


• Mark Comprehensive Basic Science Self-Assessment 4 hr 57 min 16 sec

20. A 65-year-old woman comes to the physician because of blood in her urine with no pain for 5 days. Physical examination
shows no abnormalities. Cystoscopy depicts multiple lesions as shown in the photomicrograph. Which of the following is
the strongest predisposing risk factor for the development of this lesion?

A) Alcoholism
B) Cigarette smoking
U C) Human papillomavirus infection
0 D) Schistosomiasis
U E) Vinyl chloride exposure

, ~ ~ p , ,.,,.
Previous Next Lab Values Calculator Review Help Pause
hi Comprehensive Basic Science Self-Assessment - Google Chrome x
i starttest.com/ITDVersions/19.3.0.1/ITDStart.aspx?SVC=54cd3e5f-e4bf-49d0-8371-ba261251f2a1

Exam Section 2: Item 21 of 50 National Board of Medical Examiners Time Remaining:


• Mark Comprehensive Basic Science Self-Assessment 4 hr 57 min 11 sec

21. After completing 6 courses of chemotherapy for cancer of the breast, a 45-year-old woman has shortness of breath, dyspnea on exertion, and orthopnea. Basilar crackles are heard
bilaterally, and there is an S 3 gallop. X-rays of the chest show an enlarged heart, bilateral interstitial and alveolar edema, and bilateral pleural effusions. Which of the following is the
chemotherapeutic drug most likely to have caused these findings?

A) Bleomycin
B) Cytosine arabinoside
C) Doxorubicin
D) Methotrexate
E) Vincristine

, ~ ~ p , ,.,,.
Previous Next Lab Values Calculator Review Help Pause
hi Comprehensive Basic Science Self-Assessment - Google Chrome x
i starttest.com/ITDVersions/19.3.0.1/ITDStart.aspx?SVC=54cd3e5f-e4bf-49d0-8371-ba261251f2a1

Exam Section 2: Item 22 of 50 National Board of Medical Examiners Time Remaining:


• Mark Comprehensive Basic Science Self-Assessment 4 hr 57 min 5 sec

22. A 10-year-old girl with a lifelong history of intermittent dyspnea is brought to the physician for a follow-up examination. During an episode of dyspnea , she turns blue. The episode
resolves after she squats. A systolic murmur was heard at birth. Today, a grade 4/6 systolic murmur is heard at the pulmonic area, radiating widely to the back and left side. Cardiac
catheterization shows:
Location Pressure (mm Hg) 0 2 Saturation
Aorta 120/80 82%
Vena cava 610 70%
Pulmonary artery 20/8 70%
Right atrium 4 70%
Left atrium 6 95%
Right ventricle 120/6 70%
Left ventricle 120/6 82%

Which of the following is most likely in this patient?

U A) Diastolic murmur
B) Enlarged left ventricle on chest x-ray
0 C) Hypervascular lung fields on chest x-ray
D) Palpable right ventricular lift
E) Pulsating aortic mass on CT scan of the chest

, ~ ~ p , ,.,,.
Previous Next Lab Values Calculator Review Help Pause
hi Comprehensive Basic Science Self-Assessment - Google Chrome x
i starttest.com/ITDVersions/19.3.0.1/ITDStart.aspx?SVC=54cd3e5f-e4bf-49d0-8371-ba261251f2a1

Exam Section 2: Item 23 of 50 National Board of Medical Examiners Time Remaining:


• Mark Comprehensive Basic Science Self-Assessment 4 hr 56 min 59 sec

23. A 10-year-old girl is scheduled to undergo fixation of a right femoral fracture. A neuromuscular blocking agent is administered prior to insertion of an endotracheal tube.
Subsequently, she develops severe muscle rigidity. Her temperature is 41 .1°C (106°F), pulse is 100/min, and blood pressure is unstable. Which of the following drugs most likely
caused this reaction?

A) Etomidate
B) Midazolam
C) Nitric oxide
D) Nitrous oxide
E) Rocuronium
F) Succinylcholine

, ~ ~ p , ,.,,.
Previous Next Lab Values Calculator Review Help Pause
hi Comprehensive Basic Science Self-Assessment - Google Chrome x
i starttest.com/ITDVersions/19.3.0.1/ITDStart.aspx?SVC=54cd3e5f-e4bf-49d0-8371-ba261251f2a1

Exam Section 2: Item 24 of 50 National Board of Medical Examiners Time Remaining:


• Mark Comprehensive Basic Science Self-Assessment 4 hr 56 min 53 sec

*'-
24. The graph shows expiratory pressure-volume curves that compare a healthy adult with a patient who has a 9-month Normal
history of progressive shortness of breath. These findings are most consistent with which of the following?

A) Asthma
-
-
>-
100

TS 80
B) Bronchopneumonia co
a.
C) Chronic bronchitis co 60
(.)
CJ D) Diffuse pulmonary fibrosis O>
Patient
c: 40
E) Emphysema ~

-
co
._
0
20

10 20 30 40
Transpu lmonary p ressure
(cm H20)

, ~ ~ p , ,.,,.
Previous Next Lab Values Calculator Review Help Pause
hi Comprehensive Basic Science Self-Assessment - Google Chrome x
i starttest.com/ITDVersions/19.3.0.1/ITDStart.aspx?SVC=54cd3e5f-e4bf-49d0-8371-ba261251f2a1

Exam Section 2: Item 25 of 50 National Board of Medical Examiners Time Remaining:


• Mark Comprehensive Basic Science Self-Assessment 4 hr 56 min 47 sec

25. A thin 66-year-old man is brought to the emergency department because of confusion. Initial laboratory tests show severe hyponatremia. The syndrome of inappropriate ADH
(vasopressin) secretion is suspected. Which of the following findings best supports this diagnosis?

CJ A) Increased serum urea nitrogen and creatinine concentrations


B) Serum potassium concentration < 2.5 mEq/L
C) Urine osmolality > plasma osmolality
U D) Urine sodium concentration < 10 mEq/L
E) Urine specific gravity of 1.002

, ~ ~ p , ,.,,.
Previous Next Lab Values Calculator Review Help Pause
hi Comprehensive Basic Science Self-Assessment - Google Chrome x
i starttest.com/ITDVersions/19.3.0.1/ITDStart.aspx?SVC=54cd3e5f-e4bf-49d0-8371-ba261251f2a1

Exam Section 2: Item 26 of 50 National Board of Medical Examiners Time Remaining:


• Mark Comprehensive Basic Science Self-Assessment 4 hr 56 min 41 sec

26. A 71-year-old man is brought to the emergency department because of the sudden onset of dysarthria, dysphagia, and dizziness. Examination shows markedly decreased pain
sensation on the right side of the face and complete loss of pain sensation on the left side of the body. The most likely cause is occlusion of which of the following labeled blood
vessels in the normal brain stem shown?

0 A) U B) .J C) D) U E)

, ~ ~ p , ,.,,.
Previous Next Lab Values Calculator Review Help Pause
hi Comprehensive Basic Science Self-Assessment - Google Chrome x
i starttest.com/ITDVersions/19.3.0.1/ITDStart.aspx?SVC=54cd3e5f-e4bf-49d0-8371-ba261251f2a1

Exam Section 2: Item 27 of 50 National Board of Medical Examiners Time Remaining:


• Mark Comprehensive Basic Science Self-Assessment 4 hr 56 min 35 sec

27. A 40-year-old man with AIDS develops severe diarrhea that is refractory to all standard oral therapies. His diarrhea resolves after intravenous administration of a stable analogue of
a naturally occurring hormone. The analogue administered most likely mimics which of the following hormones?

CJ A) Cortisol
B) Glucagon
C) Somatostatin
U D) Testosterone
E) Vitamin D

, ~ ~ p , ,.,,.
Previous Next Lab Values Calculator Review Help Pause
hi Comprehensive Basic Science Self-Assessment - Google Chrome x
i starttest.com/ITDVersions/19.3.0.1/ITDStart.aspx?SVC=54cd3e5f-e4bf-49d0-8371-ba261251f2a1

Exam Section 2: Item 28 of 50 National Board of Medical Examiners Time Remaining:


• Mark Comprehensive Basic Science Self-Assessment 4 hr 56 min 28 sec

28. A 25-year-old woman at 16 weeks' gestation comes to the physician for a routine examination. One year ago, she had a serum thyroxine (T 4 ) concentration of 8.2 µg/dl and a serum
thyroid-stimulating hormone (TSH) concentration of 2.1 µU/ml. At that time, she was not taking any medications and she was not pregnant. At this visit, serum T 4 concentration is
13.2 µg/dl and serum TSH concentration is 2.2 µU/ml. Which of the following is most likely responsible for the increased serum T 4 concentration in this patient?

A) Diffuse toxic goiter (Graves disease)


B) High T 4 content in prenatal vitamins
C) Increased fetal production of T 4
D) Increased hepatic production of T 4-binding globulin
E) Struma ovarii
F) TSH-producing pituitary tumor

, ~ ~ p , ,.,,.
Previous Next Lab Values Calculator Review Help Pause
hi Comprehensive Basic Science Self-Assessment - Google Chrome x
i starttest.com/ITDVersions/19.3.0.1/ITDStart.aspx?SVC=54cd3e5f-e4bf-49d0-8371-ba261251f2a1

Exam Section 2: Item 29 of 50 National Board of Medical Examiners Time Remaining:


• Mark Comprehensive Basic Science Self-Assessment 4 hr 56 min 21 sec

29. A 3-month-old girl is brought to the emergency department by her parents because of a 2-day history of a deep cough "that sounds like a barking seal," a runny nose, and decreased
appetite. Her temperature is 37.8°C (100°F). Physical examination shows inspiratory strider. A diagnosis of croup is made, but the appropriate treatment does not result in
improvement. Examination of the airway after anesthesia is administered shows two squamous, warty lesions on the vocal cords. Which of the following viruses is the most likely
cause of these lesions?

A) Epstein-Barr virus
B) Herpes simplex virus
U C) HIV
U D) Human papillomavirus
E) Poxvirus

, ~ ~ p , ,.,,.
Previous Next Lab Values Calculator Review Help Pause
hi Comprehensive Basic Science Self-Assessment - Google Chrome x
i starttest.com/ITDVersions/19.3.0.1/ITDStart.aspx?SVC=54cd3e5f-e4bf-49d0-8371-ba261251f2a1

Exam Section 2: Item 30 of 50 National Board of Medical Examiners Time Remaining:


• Mark Comprehensive Basic Science Self-Assessment 4 hr 56 min 15 sec

30. An 80-year-old woman comes to the physician because of a 1-month history of intermittent headaches and aching pain in her jaw while chew ing. She has no history of similar
headaches or pain. Neurologic examination shows no focal findings. Which of the following is the most appropriate next step in establishing the diagnosis?

CJ A) Determination of erythrocyte sedimentation rate


B) Leukocyte count
C) Chest x-ray
U D) MRI of the brain
E) Cerebrospinal fluid analysis

, ~ ~ p , ,.,,.
Previous Next Lab Values Calculator Review Help Pause
hi Comprehensive Basic Science Self-Assessment - Google Chrome x
i starttest.com/ITDVersions/19.3.0.1/ITDStart.aspx?SVC=54cd3e5f-e4bf-49d0-8371-ba261251f2a1

Exam Section 2: Item 31 of 50 National Board of Medical Examiners Time Remaining:


• Mark Comprehensive Basic Science Self-Assessment 4 hr 56 min 9 sec

31. A 50-year-old man comes to the physician because of diarrhea and a 13.6-kg (30-lb) weight loss over the past 2 months. He has three to four large, pale, foul-smelling stools with oil
droplets daily. He has a history of alcohol dependence and chronic abdominal pain. X-ray of the abdomen shows calcifications in the mid-upper abdomen. Which of the following
pathophysiologic mechanisms is the most likely cause of this patient's condition?

A) Generalized malabsorption
B) Motility disorder
C) Osmotic diuresis
D) Portal hypertension
E) Vitamin 8 12 (cobalamin) deficiency

, ~ ~ p , ,.,,.
Previous Next Lab Values Calculator Review Help Pause
hi Comprehensive Basic Science Self-Assessment - Google Chrome x
i starttest.com/ITDVersions/19.3.0.1/ITDStart.aspx?SVC=54cd3e5f-e4bf-49d0-8371-ba261251f2a1

Exam Section 2: Item 32 of 50 National Board of Medical Examiners Time Remaining:


• Mark Comprehensive Basic Science Self-Assessment 4 hr 56 min 3 sec

32. A 58-year-old man is brought to the emergency department 1 hour after being kicked in the side by a horse. Two years ago, he was diagnosed with chronic idiopathic myelofibrosis.
He drinks two 12-ounce beers daily. His temperature is 37°C (98.6°F), pulse is 120/min, respirations are 18/min, and blood pressure is 90/50 mm Hg. Abdominal examination shows
guarding and tenderness over the left upper quadrant. Laboratory studies show:
Hemoglobin 9 g/dL
Hematocrit 27%
Leukocyte count 11 ,000/mm 3
Platelet count 280,000/mm3

Ultrasonography of the abdomen shows intraperitoneal fluid. Four hours later, his hematocrit is 24%. A laparotomy is most likely to show which of the following in this patient?

A) Mesenteric artery thrombosis


B) Pancreatic pseudocyst
C) Perforated stomach
D) Ruptured bladder
E) Ruptured spleen

, ~ ~ p , ,.,,.
Previous Next Lab Values Calculator Review Help Pause
hi Comprehensive Basic Science Self-Assessment - Google Chrome x
i starttest.com/ITDVersions/19.3.0.1/ITDStart.aspx?SVC=54cd3e5f-e4bf-49d0-8371-ba261251f2a1

Exam Section 2: Item 33 of 50 National Board of Medical Examiners Time Remaining:


• Mark Comprehensive Basic Science Self-Assessment 4 hr 55 min 58 sec

33. A 26-year-old man and his 25-year-old wife come to the physician for genetic counseling prior to conception. Both of the patients were previously found to have microcytic anemia
and an increased fraction of hemoglobin A 2. Genetic analysis shows that the man has a heterozygous null mutation of the B-globin gene, and the woman has a heterozygous
mutation known to cause a 50% decrease in B-globin gene function of one allele. Which of the following best represents the B-globin function possibilities for this couple's offspring?

A) All will have decreased B-globin function and transfusion-dependent anemia


B) One in four will have less than 10% B-globin function and transfusion-dependent anemia
C) One in four will have 25% B-globin function and may require occasional transfusions
D) One in two will have 50% B-globin function without need for transfusions
E) One in two will have normal B-globin function without need for transfusions

, ~ ~ p , ,.,,.
Previous Next Lab Values Calculator Review Help Pause
hi Comprehensive Basic Science Self-Assessment - Google Chrome x
i starttest.com/ITDVersions/19.3.0.1/ITDStart.aspx?SVC=54cd3e5f-e4bf-49d0-8371-ba261251f2a1

Exam Section 2: Item 34 of 50 National Board of Medical Examiners Time Remaining:


• Mark Comprehensive Basic Science Self-Assessment 4 hr 55 min 49 sec

34. A 6-year-old girl is brought to the office by her mother because of a 2-day history of a severely itchy scalp. The mother reports that 10 children in her daughter's class also have the
same symptoms. The patient has no history of major medical illness and receives no medications. Vital signs are within normal limits. Examination of multiple hair shafts shows 1- to
2-mm, white, globular protuberances. Which of the following is the most likely cause of this patient's condition?

A) Cladosporium cladosporioides
B) Epidermophyton floccosum
C) Pediculus humanus capitis
D) Sarcoptes scabiei
E) Trichophyton rubrum

, ~ ~ p , ,.,,.
Previous Next Lab Values Calculator Review Help Pause
hi Comprehensive Basic Science Self-Assessment - Google Chrome x
i starttest.com/ITDVersions/19.3.0.1/ITDStart.aspx?SVC=54cd3e5f-e4bf-49d0-8371-ba261251f2a1

Exam Section 2: Item 35 of 50 National Board of Medical Examiners Time Remaining:


• Mark Comprehensive Basic Science Self-Assessment 4 hr 55 min 43 sec

35. A 50-year-old man undergoes surgical transection of the obturator nerve for unbalanced muscle spasm affecting the hip. Which of the following muscles is most likely to be
paralyzed by this procedure?

CJ A) Adductor brevis
B) External oblique
C) lliopsoas
U D) Obturator internus
E) Piriformis

, ~ ~ p , ,.,,.
Previous Next Lab Values Calculator Review Help Pause
hi Comprehensive Basic Science Self-Assessment - Google Chrome x
i starttest.com/ITDVersions/19.3.0.1/ITDStart.aspx?SVC=54cd3e5f-e4bf-49d0-8371-ba261251f2a1

Exam Section 2: Item 36 of 50 National Board of Medical Examiners Time Remaining:


• Mark Comprehensive Basic Science Self-Assessment 4 hr 55 min 38 sec

36. A 24-year-old man is brought to the office because of fever and chills for 1 day. He is a paraplegic and uses a wheelchair. He has a history of recurrent urinary tract infections treated
with different antibiotics over the past year. His temperature is 38.5°C (101.3°F). Urine cultures are ordered, and treatment with ciprofloxacin is begun . Two days later, he is still
febrile, and the initial urine cultures grow Escherichia coli resistant to ciprofloxacin. Which of the following is the most likely cause of this organism's resistance?

A) Acquisition of a plasmid encoding ciprofloxacin acetylase


B) Acquisition of a plasmid encoding 238 ribosomal RNA methylase
C) Alteration in DNA gyrase
D) Alteration in penicillin-binding proteins
E) Alteration in 308 ribosomal RNA
F) Alteration in RNA polymerase

, ~ ~ p , ,.,,.
Previous Next Lab Values Calculator Review Help Pause
hi Com prehensive Basic Science Self-Assessment - Goog le Chrome x
i starttest.com/ITDVersions/19.3.0.1/ITDStart.aspx?SVC=3f50c3c0-1874-400e-af3a-5036e79dec9f

Exam Section 1: Item 37 of 50 National Board of Medical Examiners Time Remaining:


• Mark Comprehensive Basic Science Self-Assessment 4 hr 52 min 45 sec

37. A 30-year-old man who is a migrant farm worker comes to the emergency department because of a 1-week history of fever, chills, and severe headache. He immigrated to the
midwestern USA from Honduras 1 year ago. He is alert. His temperature is 40°C (104 °F), pulse is 101 /min, respirations are 20/min, and blood pressure is 105/62 mm Hg. The
lungs are clear. Cardiac examination shows no abnormalities. The abdomen is soft with active bowel sounds. Neurologic examination shows a supple neck; there is no nuchal
rigidity. He is oriented to person , place, and time. Laboratory studies show:
Hemoglobin 14 g/dL
Hematocrit 42%
Leukocyte count 4600/mm 3
Segmented neutrophils 57%
Lymphocytes 33%
Monocytes 10%
Urine
Color clear yellow
Blood 1+
Urobilinogen trace

, ~ ~ p, ,.,,.
Previous Next Lab Values Calculator Review Help Pause
hi Com prehensive Basic Science Self-Assessment - Goog le Chrome x
i starttest.com/ITDVersions/19.3.0.1/ITDStart.aspx?SVC=3f50c3c0-1874-400e-af3a-5036e79dec9f

37. A 30-year-old man who is a migrant farm worker comes to the emergency department because of a 1-week history of fever, chills, and severe headache. He immigrated to the
midwestern USA from Honduras 1 year ago. He is alert. His temperature is 40°C (104 °F), pulse is 101 /min, respirations are 20/min, and blood pressure is 105/62 mm Hg. The
lungs are clear. Cardiac examination shows no abnormalities. The abdomen is soft with active bowel sounds. Neurologic examination shows a supple neck; there is no nuchal
rigidity. He is oriented to person , place, and time. Laboratory studies show:
Hemoglobin 14 g/dL
Hematocrit 42%
Leukocyte count 4600/mm 3
Segmented neutrophils 57%
Lymphocytes 33%
Monocytes 10%
Urine
Color clear yellow
Blood 1+
Urobilinogen trace

A photomicrograph of a peripheral blood smear is shown. This patient's overall condition is most likely caused by an infectious agent with which of the following features?

A) Anaerobic metabolism
U B) Chloroquine resistance
C) Exotoxin production
D) Formation of hypnozoites
E) Induction of adhesion molecules
hi Comprehensive Basic Science Self-Assessment - Google Chrome x
i starttest.com/ITDVersions/19.3.0.1/ITDStart.aspx?SVC=54cd3e5f-e4bf-49d0-8371-ba261251f2a1

Exam Section 2: Item 38 of 50 National Board of Medical Examiners Time Remaining:


• Mark Comprehensive Basic Science Self-Assessment 4 hr 55 min 28 sec

38. A 37-year-old woman comes to the physician because of nausea and vomiting. She drank half a bottle of whiskey the previous evening because "I was in great despair because my
boyfriend broke up with me. I normally don't drink alcohol , but I had to calm my nerves." She describes her mood as depressed, angry, and unstable. She says, "There is no one I
can trust except you. I recently started a job and , from what I can tell , everyone I work with is a jerk." This patient most likely has which of the following personality traits?

A) Antisocial
B) Borderline
C) Dependent
D) Narcissistic
E) Paranoid

, ~ ~ p , ,.,,.
Previous Next Lab Values Calculator Review Help Pause
hi Comprehensive Basic Science Self-Assessment - Google Chrome x
i starttest.com/ITDVersions/19.3.0.1/ITDStart.aspx?SVC=54cd3e5f-e4bf-49d0-8371-ba261251f2a1

Exam Section 2: Item 39 of 50 National Board of Medical Examiners Time Remaining:


• Mark Comprehensive Basic Science Self-Assessment 4 hr 55 min 23 sec

39. A 55-year-old man with type 1 diabetes mellitus comes to the physician because of intermittent burning pain of his feet during the past 4 months. Examination of the feet shows
allodynia bilaterally. Sensation to pinprick is decreased. Motor strength, deep tendon reflexes, joint position, and vibration sense are normal. Which of the following is the most likely
cause of the pain in this patient?

A) Increased activation of glutamate receptors in the dorsal root ganglia


B) Increased activity of presynaptic y-aminobutyric acid receptors in the dorsal horn
C) Increased activity of voltage-gated K+ channels in the thalamus
D) Inhibition of vanilloid receptors in the dorsal root afferents
E) Persistent activation of voltage-gated Na+ channels in the nociceptor

, ~ ~ p , ,.,,.
Previous Next Lab Values Calculator Review Help Pause
hi Comprehensive Basic Science Self-Assessment - Google Chrome x
i starttest.com/ITDVersions/19.3.0.1/ITDStart.aspx?SVC=54cd3e5f-e4bf-49d0-8371-ba261251f2a1

Exam Section 2: Item 40 of 50 National Board of Medical Examiners Time Remaining:


• Mark Comprehensive Basic Science Self-Assessment 4 hr 55 min 18 sec

40. A 75-year-old man is brought unconscious to the emergency department by his wife after he collapsed 30 minutes
ago. The wife says he had abdominal pain that morning, but he did not intend to visit the physician. He has not had a
physical examination since childhood. His wife says she has been begging him to go to the physician ever since he
noticed an abdominal mass 2 years ago. He has smoked 2 packs of cigarettes daily for 60 years and drinks three
12-ounce cans of beer daily. His temperature is 36.8°C (98.2°F), pulse is 80/min and irregular with premature
ventricular contraction , respirations are 35/min , and blood pressure is 110/60 mm Hg. Physical examination shows a
pulsatile abdominal mass and minimal bowel sounds. A CT scan of the abdomen is shown. Which of the following is
the most likely underlying cause of this patient's condition?

A) Atherosclerosis
u B) Diverticulosis with abscess
C) Marfan syndrome
D) Mycotic inflammation
E) Syphilis
() F) Takayasu arteritis

, ~ ~ p , ,.,,.
Previous Next Lab Values Calculator Review Help Pause
hi Comprehensive Basic Science Self-Assessment - Google Chrome x
i starttest.com/ITDVersions/19.3.0.1/ITDStart.aspx?SVC=54cd3e5f-e4bf-49d0-8371-ba261251f2a1

Exam Section 2: Item 41 of 50 National Board of Medical Examiners Time Remaining:


• Mark Comprehensive Basic Science Self-Assessment 4 hr 55 min 10 sec

41. A 32-year-old man is brought to the emergency department after sustaining a deep laceration that may have injured the nerve supply to the flexor digitorum profundus muscle of the
index finger. Which of the following movements can be used to specifically test for the function of this muscle?

CJ A) Abduction of a finger
B) Adduction of a finger
C) Flexion at the distal interphalangeal joint
U D) Flexion at the metacarpophalangeal joint
E) Flexion at the proximal interphalangeal joint

, ~ ~ p , ,.,,.
Previous Next Lab Values Calculator Review Help Pause
hi Comprehensive Basic Science Self-Assessment - Google Chrome x
i starttest.com/ITDVersions/19.3.0.1/ITDStart.aspx?SVC=54cd3e5f-e4bf-49d0-8371-ba261251f2a1

Exam Section 2: Item 42 of 50 National Board of Medical Examiners Time Remaining:


• Mark Comprehensive Basic Science Self-Assessment 4 hr 55 min 5 sec

42. A 32-year-old woman who is 20 weeks' pregnant has had increasingly severe episodes of heartburn for the past 3 weeks. She has mild epigastric tenderness on palpation of the
abdomen. Which of the following drugs is contraindicated because of an increased risk for spontaneous abortion?

CJ A) Bismuth subsalicylate
B) Famotidine
C) Misoprostol
U D) Omeprazole
E) Sucralfate

, ~ ~ p , ,.,,.
Previous Next Lab Values Calculator Review Help Pause
hi Comprehensive Basic Science Self-Assessment - Google Chrome x
i starttest.com/ITDVersions/19.3.0.1/ITDStart.aspx?SVC=54cd3e5f-e4bf-49d0-8371-ba261251f2a1

Exam Section 2: Item 43 of 50 National Board of Medical Examiners Time Remaining:


• Mark Comprehensive Basic Science Self-Assessment 4 hr 54 min 58 sec

43. A 55-year-old man is brought to the emergency department because of a 1-hour history of severe chest pain, nausea, and vomiting. He is agitated, clammy, and sweating profusely.
His temperature is 36.?°C (98°F), pulse is 130/min, respirations are 36/min, and blood pressure is 85/45 mm Hg. Jugular venous pressure is 12 cm H 20 and crackles are heard in
the lung bases bilaterally. Which of the following hemodynamic changes is most likely in this patient?

Pulmonary Capillary Wedge Cardiac Output Systemic Vascular


Pressure Resistance
U A) i i i
U B) i l i
u C) l l l
D) l l i
E) l i l

, ~ ~ p , ,.,,.
Previous Next Lab Values Calculator Review Help Pause
hi Comprehensive Basic Science Self-Assessment - Google Chrome x
i starttest.com/ITDVersions/19.3.0.1/ITDStart.aspx?SVC=54cd3e5f-e4bf-49d0-8371-ba261251f2a1

Exam Section 2: Item 44 of 50 National Board of Medical Examiners Time Remaining:


• Mark Comprehensive Basic Science Self-Assessment 4 hr 54 min 53 sec

44. A 62-year-old man comes to the physician for a follow-up examination. One month ago, he was diagnosed with atrial and ventricular arrhythmia, and appropriate pharmacotherapy
was initiated at that time. Today, his pulse is 64/min, and blood pressure is 136/88 mm Hg; 1 month ago, his pulse was 78/min, and blood pressure was 152/95 mm Hg. Physical
examination shows no other abnormalities. An ECG shows normal sinus rhythm with a prolonged QT interval. Which of the following drugs is the most likely cause of these findings?

A) Adenosine
B) Flecainide
C) Metoprolol
D) Phenytoin
E) Sotalol

, ~ ~ p , ,.,,.
Previous Next Lab Values Calculator Review Help Pause
hi Com prehensive Basic Science Self-Assessment - Goog le Chrome x
i starttest.com/ITDVersions/19.3.0.1/ITDStart.aspx?SVC=54cd3e5f-e4bf-49d0-8371-ba261251f2a1

Exam Section 2: Item 45 of 50 National Board of Medical Examiners Time Remaining:


• Mark Comprehensive Basic Science Self-Assessment 4 hr 54 min 46 sec

45. A 34-year-old woman with a long history of rheumatoid arthritis refractory to treatment with corticosteroids and methotrexate comes to the physician because of a 2-week history of
increasingly severe low back pain, decreased appetite, and malaise. She has never had low back pain in the past. She has not had fever, lower extremity weakness, constipation,
bladder or bowel incontinence, weight loss, or sensory changes. Six weeks ago, she began treatment with infliximab because of bilateral wrist, hand, and knee pain. Her
temperature is 37°C (98.6°F). Physical examination shows mild tenderness over the L 1-2 vertebrae. Laboratory studies show:
Hemoglobin 11.3 g/dL
Hematocrit 32%
Leukocyte count 7800/mm 3
Platelet count 290,000/mm3

An MRI of the spine shows osteomyelitis of L1-2 with destruction of the intervertebral disc space and a 1.2-cm adjacent abscess. The abscess is debrided, and samples of the
abscess and bone tissue are sent for analysis. A Gram stain is negative. Pathologic examination of the bone specimen shows acute and chronic inflammation with granuloma
formation. Which of the following is the most likely cause of these findings?

A) Ankylosing spondylitis
U B) Multiple myeloma
C) Progression of rheumatoid arthritis
() D) Sarcoidosis
E) Tuberculous osteomyelitis

, ~ ~ p , ,.,,.
Previous Next Lab Values Calculator Review Help Pause
hi Comprehensive Basic Science Self-Assessment - Google Chrome x
i starttest.com/ITDVersions/19.3.0.1/ITDStart.aspx?SVC=54cd3e5f-e4bf-49d0-8371-ba261251f2a1

Exam Section 2: Item 46 of 50 National Board of Medical Examiners Time Remaining:


• Mark Comprehensive Basic Science Self-Assessment 4 hr 54 min 41 sec

46. A 32-year-old woman comes to the physician because of a 7-day history of sneezing, nasal stuffiness, and watery eyes. She has a history of similar symptoms each spring while
gardening. Her temperature is 37°C (98.6°F). Which of the follow ing types of cells are most likely to be increased in her nasal secretions as a result of this reaction?

CJ A) Basophils
B) Eosinophils
C) Lymphocytes
U D) Mast cells
E) Monocytes

, ~ ~ p , ,.,,.
Previous Next Lab Values Calculator Review Help Pause
hi Comprehensive Basic Science Self-Assessment - Google Chrome x
i starttest.com/ITDVersions/19.3.0.1/ITDStart.aspx?SVC=54cd3e5f-e4bf-49d0-8371-ba261251f2a1

Exam Section 2: Item 47 of 50 National Board of Medical Examiners Time Remaining:


• Mark Comprehensive Basic Science Self-Assessment 4 hr 54 min 36 sec

47. A 35-year-old man comes to the physician 1 week after he had a high blood pressure reading taken while he was donating blood. His pulse is 76/min, and blood pressure is
180/100 mm Hg. Physical examination shows no other abnormalities. Treatment with losartan is started. Which of the following is most likely to occur in this patient?

CJ A) Decreased plasma renin activity


B) Decreased serum bradykinin concentration
C) Increased serum aldosterone concentration
U D) Increased serum angiotensin II concentration
E) Increased serum norepinephrine concentration

, ~ ~ p , ,.,,.
Previous Next Lab Values Calculator Review Help Pause
hi Comprehensive Basic Science Self-Assessment - Google Chrome x
i starttest.com/ITDVersions/19.3.0.1/ITDStart.aspx?SVC=54cd3e5f-e4bf-49d0-8371-ba261251f2a1

Exam Section 2: Item 48 of 50 National Board of Medical Examiners Time Remaining:


• Mark Comprehensive Basic Science Self-Assessment 4 hr 54 min 30 sec

48. Immediately after a spontaneous vaginal delivery, a full-term male newborn is found to have a tuft of hair on the midline in the lumbar region. Palpation of the area shows an
absence of spinous processes in that region. Imaging studies show no additional abnormalities. The most likely cause of this defect was failure of which of the following events
during gestation?

A) Development of the notochord


B) Formation of the neural tube
C) Formation of the yolk sac
D) Fusion of the sclerotomes
E) Migration of the neural crests

, ~ ~ p , ,.,,.
Previous Next Lab Values Calculator Review Help Pause
hi Comprehensive Basic Science Self-Assessment - Google Chrome x
i starttest.com/ITDVersions/19.3.0.1/ITDStart.aspx?SVC=54cd3e5f-e4bf-49d0-8371-ba261251f2a1

Exam Section 2: Item 49 of 50 National Board of Medical Examiners Time Remaining:


• Mark Comprehensive Basic Science Self-Assessment 4 hr 54 min 24 sec

49. A 35-year-old woman comes to the physician because of a 1-year history of fatigue and lethargy; she also has had a 9-kg (20-lb) weight gain during this period. She says that she
has to wear a coat in her office because she is always cold. Physical examination shows coarse skin. Examination of the neck shows no palpable thyroid gland tissue. Neurologic
examination shows a delayed relaxation phase of the calcaneal (Achilles) reflexes. Serum concentrations of thyroxine (T 4) and triiodothyronine (T 3) are decreased. A thyroid scan
shows localization of the radioiodine in the midline of the inferior aspect of the oral cavity; no radioactivity is detected in the neck. The radioiodine is most likely localized inferior to
which of the following structures?

A) Foramen cecum
B) Frenulum of the tongue
C) Median glossoepiglottic fold
lJ D) Sublingual caruncle
U E) Tracheobronchial bifurcation

, ~ ~ p , ,.,,.
Previous Next Lab Values Calculator Review Help Pause
hi Comprehensive Basic Science Self-Assessment - Google Chrome x
i starttest.com/ITDVersions/19.3.0.1/ITDStart.aspx?SVC=54cd3e5f-e4bf-49d0-8371-ba261251f2a1

Exam Section 2: Item 50 of 50 National Board of Medical Examiners Time Remaining:


• Mark Comprehensive Basic Science Self-Assessment 4 hr 54 min 17 sec

50. Results of a 5-year screening program for HIV infection in a population of 10,000 commercial sex workers are shown:
Time Number Remaining in Population Number of New Patients With Positive Serology
Intake 10,000 4000
1 year 6000 400
2 years 5600 250
3 years 5350 300
4 years 5050 300
5 years 4800 250

Which of the following percentages is closest to the average annual incidence of infection in this population?

A) 1%
U B) 5%
C) 15%
U D) 20%
lJ E) 35%
U F) 40%

, ~ ~ p , ,.,,.
Previous Next Lab Values Calculator Review Help Pause
hi Comprehensive Basic Science Self-Assessment - Google Chrome x
i starttest.com/ITDVersions/19.3.0.1/ITDStart.aspx?SVC=54cd3e5f-e4bf-49d0-8371-ba261251f2a1

Exam Section 3: Item 1 of 50 National Board of Medical Examiners Time Remaining:


• Mark Comprehensive Basic Science Self-Assessment 4 hr 59 min 41 sec

1. An investigator is studying a new drug for the treatment of patients undergoing adjuvant radiation therapy. The drug is designed to be administered prior to irradiation to minimize
localized tissue damage at the irradiated site. This drug most likely inhibits which of the following effects of external beam radiation?

CJ A) Depurination
B) DNA ligase inactivation
C) DNA polymerase activation
U D) Formation of pyrimidine dimers
E) Free radical formation
F) Tautomerization

~ ~ p , ,.,,.
Next Lab Values Calculator Review Help Pause
hi Comprehensive Basic Science Self-Assessment - Google Chrome x
i starttest.com/ITDVersions/19.3.0.1/ITDStart.aspx?SVC=54cd3e5f-e4bf-49d0-8371-ba261251f2a1

Exam Section 3: Item 2 of 50 National Board of Medical Examiners Time Remaining:


• Mark Comprehensive Basic Science Self-Assessment 4 hr 59 min 31 sec

2. A 70-year-old man comes to the physician because of a 1-year history of progressive shortness of breath and nonproductive cough. He is now unable to tolerate even mild activity.
Physical examination shows clubbing of the fingers. lnspiratory crackles are heard at both lung bases. A CT scan of the chest shows patchy subpleural reticular opacities.
Examination of a lung biopsy specimen shows a heterogeneous pattern with alternating areas of normal lung and interstitial inflammation and fibrosis. On questioning, he has not
taken any medications or had environmental exposures associated with pulmonary fibrosis. Which of the following pulmonary function tests in this patient will most likely show a result
greater than the predicted range?

A) Alveolar-arterial Po 2 difference
B) Diffusing capacity measured with carbon monoxide
C) Functional residual capacity
lJ D) FVC
U E) Residual volume
U F) Tidal volume

, ~ ~ p , ,.,,.
Previous Next Lab Values Calculator Review Help Pause
hi Comprehensive Basic Science Self-Assessment - Google Chrome x
i starttest.com/ITDVersions/19.3.0.1/ITDStart.aspx?SVC=54cd3e5f-e4bf-49d0-8371-ba261251f2a1

Exam Section 3: Item 3 of 50 National Board of Medical Examiners Time Remaining:


• Mark Comprehensive Basic Science Self-Assessment 4 hr 59 min 1 sec

3. A 4-year-old boy has a 1-cm round mid line mass just inferior to the hyoid bone. The mass was present at birth and remains unchanged. The mass is most likely derived from tissue
that originated from which of the following structures?

CJ A) Pharyngeal arch
B) Submandibular gland
C) Thymus
U D) Tongue
E) Trachea

, ~ ~ p , ,.,,.
Previous Next Lab Values Calculator Review Help Pause
hi Comprehensive Basic Science Self-Assessment - Google Chrome x
i starttest.com/ITDVersions/19.3.0.1/ITDStart.aspx?SVC=54cd3e5f-e4bf-49d0-8371-ba261251f2a1

Exam Section 3: Item 4 of 50 National Board of Medical Examiners Time Remaining:


• Mark Comprehensive Basic Science Self-Assessment 4 hr 58 min 30 sec

4. A 35-year-old man with quadriplegia develops a urinary tract infection. He was admitted to the neurology unit of the hospital 7 months ago. He has had an intermittent indwelling
urinary catheter during this entire period. A culture of urine grows an organism that is susceptible only to aminoglycosides. Gentamicin is administered. If nephrotoxicity occurs in this
patient, it is most likely to originate in which of the following labeled sites depicted in the drawing of the nephron shown?

U A) lJ B) C) D) E)

, ~ ~ p , ,.,,.
Previous Next Lab Values Calculator Review Help Pause
hi Comprehensive Basic Science Self-Assessment - Google Chrome x
i starttest.com/ITDVersions/19.3.0.1/ITDStart.aspx?SVC=54cd3e5f-e4bf-49d0-8371-ba261251f2a1

Exam Section 3: Item 5 of 50 National Board of Medical Examiners Time Remaining:


• Mark Comprehensive Basic Science Self-Assessment 4 hr 58 min 20 sec

5. A 52-year-old man is admitted to the hospital for treatment of a massive acute myocardial infarction. Treatment with oxygen, (3-adrenergic blockers, aspirin, and reperfusion therapy is
initiated. Eighteen days later, he develops ventricular fibrillation , from which he cannot be resuscitated. A photomicrograph of cardiac tissue from the site of infarct obtained at autopsy
is shown. Which of the following best describes the appearance of this patient's heart?

U A) Abscess
0 B) Coagulation necrosis
C) Dense fibrous scar
D) Granulation tissue
u E) Granuloma

, ~ ~ p , ,.,,.
Previous Next Lab Values Calculator Review Help Pause
hi Comprehensive Basic Science Self-Assessment - Google Chrome x
i starttest.com/ITDVersions/19.3.0.1/ITDStart.aspx?SVC=54cd3e5f-e4bf-49d0-8371-ba261251f2a1

Exam Section 3: Item 6 of 50 National Board of Medical Examiners Time Remaining:


• Mark Comprehensive Basic Science Self-Assessment 4 hr 58 min 13 sec

6. An 80-year-old man is admitted to the hospital because of a 2-week history of a severe, persistent urinary tract infection. An 80-mg dose of Drug X is administered intravenously.
Thirty minutes after the infusion is complete, his serum concentration of Drug Xis 4 µg/ml. Assuming a distribution half-life of 3 minutes and an elimination half-life of 24 hours, the
volume of distribution of this drug (in L) in the peripheral compartment is closest to which of the following?

A) 2
B) 5
C) 10
D) 15
E) 20
F) 50

, ~ ~ p , ,.,,.
Previous Next Lab Values Calculator Review Help Pause
hi Comprehensive Basic Science Self-Assessment - Google Chrome x
i starttest.com/ITDVersions/19.3.0.1/ITDStart.aspx?SVC=54cd3e5f-e4bf-49d0-8371-ba261251f2a1

Exam Section 3: Item 7 of 50 National Board of Medical Examiners Time Remaining:


• Mark Comprehensive Basic Science Self-Assessment 4 hr 58 min 7 sec

7. A 68-year-old man has loss of pain and temperature sensations on the right side of the face and left side of the body, paralysis of the vocal cord on the right, and absence of the gag
reflex on the right. Which of the following regions of the brain stem is most likely to be involved?

CJ A) Left ventromedial medulla


B) Right ventromedial medulla
C) Left dorsolateral medulla
U D) Right dorsolateral medulla
E) Left ventrolateral pons
F) Right ventrolateral pons
G) Left ventromedial midbrain
H) Right ventromedial midbrain

, ~ ~ p , ,.,,.
Previous Next Lab Values Calculator Review Help Pause
hi Comprehensive Basic Science Self-Assessment - Google Chrome x
i starttest.com/ITDVersions/19.3.0.1/ITDStart.aspx?SVC=54cd3e5f-e4bf-49d0-8371-ba261251f2a1

Exam Section 3: Item 8 of 50 National Board of Medical Examiners Time Remaining:


• Mark Comprehensive Basic Science Self-Assessment 4 hr 58 min 3 sec

8. A 56-year-old man who is admitted to the hospital for treatment of pneumonia is found to have advanced lung cancer. His respiratory status improves with intravenous antibiotic
therapy. Before therapy is completed , he refuses palliative radiation therapy and wants to be discharged. He states that he is all alone, that no one cares about him, and that he does
not want to waste someone else's money for treatment. He promises that he will return in 6 months after he makes money from selling his invention that will cure arthritis. Which of
the following is the most appropriate next step?

A) Arrange for ambulatory administration of antibiotics


B) Determine whether the patient has decision-making capacity
u C) Institute home-visit care
U D) Seek a court order to force the patient to stay in the hospital
E) Transfer the patient to a psychiatric outpatient facility

, ~ ~ p, ,.,,.
Previous Next Lab Values Calculator Review Help Pause
hi Comprehensive Basic Science Self-Assessment - Google Chrome x
i starttest.com/ITDVersions/19.3.0.1/ITDStart.aspx?SVC=54cd3e5f-e4bf-49d0-8371-ba261251f2a1

Exam Section 3: Item 9 of 50 National Board of Medical Examiners Time Remaining:


• Mark Comprehensive Basic Science Self-Assessment 4 hr 57 min 58 sec

9. A 68-year-old man comes to the physician because he is concerned about changes in his sexual performance during the past year. He noticed the changes when he started dating a
40-year-old woman after 25 years of living alone. "My orgasms are shorter and less intense than when I was married. I have to wait 2 or 3 hours before having sex again." His pulse is
72/min and regular, and blood pressure is 138/78 mm Hg. Physical examination shows hair growth on the tops of the feet and toes. Dorsalis pedis and posterior tibial pulses are +2
bilaterally, and his capillary refill time is 1 to 2 seconds. There is symmetric enlargement of the prostate without nodules. Laboratory studies show a hemoglobin A 1c of 5.5%, a serum
glucose concentration of 121 mg/dl, and a serum prostate-specific antigen concentration of 4.5 ng/ml (N<4 ). Which of the following is the most likely cause of this patient's changes
in sexual function?

A) Atherosclerosis
B) Benign prostatic hyperplasia
C) Diabetic neuropathy
() D) Prostate cancer
u E) Psychogenic erectile dysfunction
U F) Normal aging

, ~ ~ p , ,.,,.
Previous Next Lab Values Calculator Review Help Pause
hi Comprehensive Basic Science Self-Assessment - Google Chrome x
i starttest.com/ITDVersions/19.3.0.1/ITDStart.aspx?SVC=54cd3e5f-e4bf-49d0-8371-ba261251f2a1

Exam Section 3: Item 10 of 50 National Board of Medical Examiners Time Remaining:


• Mark Comprehensive Basic Science Self-Assessment 4 hr 57 min 51 sec

10. A public health consultant is contacted by a health maintenance organization for recommendations about primary health prevention techniques for a population of women ages 30 to
40 years. Which of the following is the most appropriate primary preventative recommendation for this group?

CJ A) Abstinence from alcohol


B) Low-carbohydrate diet
C) Regular exercise
U D) HIV testing every 6 months
E) Annual mammography

, ~ ~ p , ,.,,.
Previous Next Lab Values Calculator Review Help Pause
hi Comprehensive Basic Science Self-Assessment - Google Chrome x
i starttest.com/ITDVersions/19.3.0.1/ITDStart.aspx?SVC=54cd3e5f-e4bf-49d0-8371-ba261251f2a1

Exam Section 3: Item 11 of 50 National Board of Medical Examiners Time Remaining:


• Mark Comprehensive Basic Science Self-Assessment 4 hr 57 min 46 sec

11. A 17-year-old girl comes to the physician because she has never had a menstrual period. She is not sexually active. She is 180 cm(5 ft 11 in) tall and weighs 50 kg (110 lb); BMI is
15 kg/m2. Breast development is Tanner stage 5, and pubic and axillary hair development is Tanner stage 1. Pelvic examination shows a blind vaginal pouch. Which of the following
is most likely to be found on further testing?

A) Decreased serum estrogen concentration


B) Decreased serum luteinizing hormone concentration
C) Hyperplastic adrenal cortices
D) Increased serum testosterone concentration
E) Streak ovaries
F) Uterus

, ~ ~ p , ,.,,.
Previous Next Lab Values Calculator Review Help Pause
hi Comprehensive Basic Science Self-Assessment - Google Chrome x
i starttest.com/ITDVersions/19.3.0.1/ITDStart.aspx?SVC=54cd3e5f-e4bf-49d0-8371-ba261251f2a1

Exam Section 3: Item 12 of 50 National Board of Medical Examiners Time Remaining:


• Mark Comprehensive Basic Science Self-Assessment 4 hr 57 min 38 sec

12. A 50-year-old woman is admitted to the hospital for management of an acute exacerbation of heart failure. She has been admitted to the hospital more than eight times during the
past year for similar episodes. On admission, intravenous administration of a diuretic is begun, and her fluid intake is restricted to 2 L daily. Her temperature is 36.3°C (97.3°F), pulse
is 88/min, respirations are 20/min, and blood pressure is 140/72 mm Hg. Bilateral basilar crackles are heard on pulmonary examination. There is pitting edema of the lower
extremities. Her fluid balance and weight are closely monitored. Two days later, the patient's weight and clinical status are unchanged. The nurse finds the patient in the bathroom
drinking directly from the faucet and holding a full pitcher of water, despite having already reached her fluid limit for the day. Which of the following is the most likely explanation for
this patient's lack of clinical improvement?

A) Antisocial personality disorder


B) Conversion disorder
C) Diabetes insipidus
() D) Factitious disorder
u E) Inappropriate secretion of ADH (vasopressin)

, ~ ~ p , ,.,,.
Previous Next Lab Values Calculator Review Help Pause
hi Comprehensive Basic Science Self-Assessment - Google Chrome x
i starttest.com/ITDVersions/19.3.0.1/ITDStart.aspx?SVC=54cd3e5f-e4bf-49d0-8371-ba261251f2a1

Exam Section 3: Item 13 of 50 National Board of Medical Examiners Time Remaining:


• Mark Comprehensive Basic Science Self-Assessment 4 hr 57 min 35 sec

13. A 65-year-old woman with well-controlled type 2 diabetes mellitus comes to the physician for a follow-up examination. At her last office visit 1 year ago, physical examination and
laboratory studies showed no abnormalities. Her temperature is 37.2°C (99°F), pulse is 82/min, respirations are 18/min, and blood pressure is 135/80 mm Hg. Physical examination
shows no other abnormalities. Laboratory studies show:
Hemoglobin 11.2 g/dl
Hematocrit 33%
Serum
Urea nitrogen 30 mg/dl
Creatinine 2.1 mg/dl

Abdominal ultrasonography shows decreased size of both kidneys. MR aortography shows bilateral proximal renal artery stenoses. Which of the following is the most likely
diagnosis?

A) Atherosclerosis
B) Congenital renal artery hypoplasia
U C) Fibromuscular dysplasia
D) Takayasu arteritis
U E) Temporal arteritis

, ~ ~ p , ,.,,.
Previous Next Lab Values Calculator Review Help Pause
hi Comprehensive Basic Science Self-Assessment - Google Chrome x
i starttest.com/ITDVersions/19.3.0.1/ITDStart.aspx?SVC=54cd3e5f-e4bf-49d0-8371-ba261251f2a1

Exam Section 3: Item 14 of 50 National Board of Medical Examiners Time Remaining:


• Mark Comprehensive Basic Science Self-Assessment 4 hr 57 min 29 sec

14. A 59-year-old man has repeated episodes of gouty arthritis unresponsive to preventive therapy. Which of the following drugs is most likely to increase uric acid excretion in this man?

A) Acetylsalicylic acid
U B) Colchicine
U C) Ketorolac
D) Penicillin
E) Probenecid

, ~ ~ p , ,.,,.
Previous Next Lab Values Calculator Review Help Pause
hi Comprehensive Basic Science Self-Assessment - Google Chrome x
i starttest.com/ITDVersions/19.3.0.1/ITDStart.aspx?SVC=54cd3e5f-e4bf-49d0-8371-ba261251f2a1

Exam Section 3: Item 15 of 50 National Board of Medical Examiners Time Remaining:


• Mark Comprehensive Basic Science Self-Assessment

15. A 35-year-old African American man comes to the physician for a routine examination. He recently read a pamphlet at a health fair on the importance of screening for skin cancer.
He enjoys sailing and usually goes out on his boat every weekend when the weather is nice. He does not use sunscreen , but he states that he does not "burn." Physical examination
shows no abnormalities. This patient is at increased risk for melanoma at which of the following locations?

A) Back
B) Chest
C) Forehead
D) Palms
E) Scalp

, ~ ~ p , ,.,,.
Previous Next Lab Values Calculator Review Help Pause
hi Comprehensive Basic Science Self-Assessment - Google Chrome x
i starttest.com/ITDVersions/19.3.0.1/ITDStart.aspx?SVC=54cd3e5f-e4bf-49d0-8371-ba261251f2a1

Exam Section 3: Item 16 of 50 National Board of Medical Examiners Time Remaining:


• Mark Comprehensive Basic Science Self-Assessment 4 hr 57 min 18 sec

16. A patient with cancer who is being treated with high-dose chemotherapy has severe bone marrow suppression. Which of the following cytokines is most likely to be beneficial for the
bone marrow suppression?

CJ A) Granulocyte colony-stimulating factor


B) Interferon-a
C) lnterferon-y
U D) lnterleukin-2 (IL-2)
E) lnterleukin-4 (IL-4)
F) lnterleukin-6 (IL-6)
G) Transforming growth factor-B
H) Tumor necrosis factor

, ~ ~ p , ,.,,.
Previous Next Lab Values Calculator Review Help Pause
hi Comprehensive Basic Science Self-Assessment - Google Chrome x
i starttest.com/ITDVersions/19.3.0.1/ITDStart.aspx?SVC=54cd3e5f-e4bf-49d0-8371-ba261251f2a1

Exam Section 3: Item 17 of 50 National Board of Medical Examiners Time Remaining:


• Mark Comprehensive Basic Science Self-Assessment 4 hr 57 min 12 sec

17. The breakdown of dipeptides and tripeptides to free amino acids takes place primarily in which of the following areas in the gastrointestinal tract?

A) Intestinal mucosa
U B) Lumen of the duodenum
U C) Lumen of the large intestine
D) Lumen of the stomach
E) Mouth

, ~ ~ p , ,.,,.
Previous Next Lab Values Calculator Review Help Pause
hi Comprehensive Basic Science Self-Assessment - Google Chrome x
i starttest.com/ITDVersions/19.3.0.1/ITDStart.aspx?SVC=54cd3e5f-e4bf-49d0-8371-ba261251f2a1

Exam Section 3: Item 18 of 50 National Board of Medical Examiners Time Remaining:


• Mark Comprehensive Basic Science Self-Assessment 4 hr 57 min 6 sec

18. A 55-year-old woman comes to the physician because of a 3-month history of difficulty using her hands and a 3-week history of muscle cramps. She has had a 9-kg (20-lb) weight
loss during the past 5 weeks. Neurologic examination shows tongue fasciculations, and lower extremity weakness and atrophy. Sensory examination is normal. A lesion at which of
the following sites is the most likely cause of these findings?

A) Cerebral white matter


B) Corticospinal tract in the spinal cord
C) Internal capsule
D) Lower motoneurons
E) Peripheral nerves

, ~ ~ p , ,.,,.
Previous Next Lab Values Calculator Review Help Pause
hi Comprehensive Basic Science Self-Assessment - Google Chrome x
i starttest.com/ITDVersions/19.3.0.1/ITDStart.aspx?SVC=54cd3e5f-e4bf-49d0-8371-ba261251f2a1

Exam Section 3: Item 19 of 50 National Board of Medical Examiners Time Remaining:


• Mark Comprehensive Basic Science Self-Assessment 4 hr 57 min 1 sec

19. A 4 7-year-old man comes to the physician because of recurrent episodes of joint pain during the past 3 years. He
describes these as abrupt in onset and involving principally his ankles or knees, usually only one joint at a time. He does
not take any medications. Physical examination shows yellow-white nodules at the tips of several fingers , as well as over
the soles of the feet. His left knee is warm, tender, and swollen with dusky, erythematous overlying skin. A photomicrograph
of joint fluid aspirate is shown. The crystals shown are most likely composed of which of the following?

U A) Basic calcium phosphate (hydroxyapatite)


B) Calcium pyrophosphate dihydrate
u C) Cholesterol
D) Corticosteroid ester
E) Methylmethacrylate
F) Monosodium urate

, ~ ~ p , ,.,,.
Previous Next Lab Values Calculator Review Help Pause
hi Comprehensive Basic Science Self-Assessment - Google Chrome x
i starttest.com/ITDVersions/19.3.0.1/ITDStart.aspx?SVC=54cd3e5f-e4bf-49d0-8371-ba261251f2a1

Exam Section 3: Item 20 of 50 National Board of Medical Examiners Time Remaining:


• Mark Comprehensive Basic Science Self-Assessment 4 hr 56 min 55 sec

20. A 28-year-old man with chronic hepatitis C comes to the physician because of fever and progressive shortness of breath during the past 2 days. His temperature is 39.3°C (102.?°F),
pulse is 110/min, respirations are 28/min, and blood pressure is 120/70 mm Hg. Physical examination shows wasting and intercostal retractions. Crackles are heard over both lung
fields. A chest x-ray shows diffuse hazy infiltrates. His leukocyte count is 2000/mm 3 (90% segmented neutrophils, 5% lymphocytes, and 5% monocytes). To help explain the cause of
his illness, this patient should be tested for which of the following other chronic viral infections?

A) Cytomegalovirus
B) Epstein-Barr virus
u C) Hepatitis D
U D) HIV
E) Human herpes virus-6

, ~ ~ p , ,.,,.
Previous Next Lab Values Calculator Review Help Pause
hi Comprehensive Basic Science Self-Assessment - Google Chrome x
i starttest.com/ITDVersions/19.3.0.1/ITDStart.aspx?SVC=54cd3e5f-e4bf-49d0-8371-ba261251f2a1

Exam Section 3: Item 21 of 50 National Board of Medical Examiners Time Remaining:


• Mark Comprehensive Basic Science Self-Assessment 4 hr 56 min 43 sec

21 . A 73-year-old woman has had easy fatigability for 2 years. She had an ileal resection for Crohn disease 10 years ago. She has
extremely pale oral mucosa. A peripheral blood smear is shown . Which of the following is the most likely mechanism of this
disorder?

A) Decreased serum transferrin concentration


B) Deficiency of glucose 6-phosphate dehydrogenase
U C) Deficiency of protoporphyrinogen oxidase
0 D) Failure of conversion of N 5-methyltetrahydrofolate to tetrahydrofolate

, ~ ~ p , ,.,,.
Previous Next Lab Values Calculator Review Help Pause
hi Comprehensive Basic Science Self-Assessment - Google Chrome x
i starttest.com/ITDVersions/19.3.0.1/ITDStart.aspx?SVC=54cd3e5f-e4bf-49d0-8371-ba261251f2a1

Exam Section 3: Item 22 of 50 National Board of Medical Examiners Time Remaining:


• Mark Comprehensive Basic Science Self-Assessment 4 hr 56 min 9 sec

22. A 35-year-old woman undergoes a left oophorectomy because of a 5 x 5-cm ovarian mass. During this procedure, which of the following structures is at greatest risk for injury when
dividing the suspensory ligament?

CJ A) Left internal iliac artery


B) Left renal vein
C) Umbilical artery
U D) Ureter
E) Uterine artery

, ~ ~ p , ,.,,.
Previous Next Lab Values Calculator Review Help Pause
hi Comprehensive Basic Science Self-Assessment - Google Chrome x
i starttest.com/ITDVersions/19.3.0.1/ITDStart.aspx?SVC=54cd3e5f-e4bf-49d0-8371-ba261251f2a1

Exam Section 3: Item 23 of 50 National Board of Medical Examiners Time Remaining:


• Mark Comprehensive Basic Science Self-Assessment 4 hr 55 min 53 sec

23. A 45-year-old man comes to the physician because of progressive daytime sleepiness and fatigue during the past 2 years. He says that he frequently wakes up in the night and has
been told he snores loudly. He is 180 cm (5 ft 11 in) tall and weighs 159 kg (350 lb); BMI is 49 kg/m 2. Physical examination shows peripheral edema below the knees. A loud S 2 is
heard on auscultation of the chest. This patient's symptoms are most likely caused by an episodic decrease in which of the following?

A) Lung volume
B) Oxygen saturation
C) Pulmonary vascular resistance
D) Systemic vascular resistance
E) Venous return

, ~ ~ p , ,.,,.
Previous Next Lab Values Calculator Review Help Pause
hi Comprehensive Basic Science Self-Assessment - Google Chrome x
i starttest.com/ITDVersions/19.3.0.1/ITDStart.aspx?SVC=54cd3e5f-e4bf-49d0-8371-ba261251f2a1

Exam Section 3: Item 24 of 50 National Board of Medical Examiners Time Remaining:


• Mark Comprehensive Basic Science Self-Assessment 4 hr 55 min 44 sec

24. A 9-year-old boy who was adopted from an African country 2 weeks ago is brought to the physician because of difficulty walking and fatigue. Femoral, popliteal, posterior tibial , and
dorsalis pedis pulses are equal and full. Neurologic examination shows weakness of plantar dorsiflexion and foot intrinsics. He has a broad-based, ataxic gait. Sensation to light
touch is decreased over the lower extremities; vibration sense is absent. Laboratory studies show:
Hemoglobin decreased
Hematocrit decreased
Mean corpuscular hemoglobin normal
Mean corpuscular hemoglobin concentration normal
Mean corpuscular volume normal
Platelet count normal
Serum lactate dehydrogenase increased

This patient most likely has a deficiency of which of the following vitamins?

A) 8 2 (riboflavin)
U 8) 8 12 (cobalamin)
C) C
() D) D
E) E

, ~ ~ p , ,.,,.
Previous Next Lab Values Calculator Review Help Pause
hi Comprehensive Basic Science Self-Assessment - Google Chrome x
i starttest.com/ITDVersions/19.3.0.1/ITDStart.aspx?SVC=54cd3e5f-e4bf-49d0-8371-ba261251f2a1

Exam Section 3: Item 25 of 50 National Board of Medical Examiners Time Remaining:


• Mark Comprehensive Basic Science Self-Assessment 4 hr 55 min 38 sec

25. A 13-year-old boy is brought to the physician by his mother for an examination prior to participating on the school bowling team. His mother says that he has asked to be excused
from gym class this year because of chest swelling that began 6 months ago. He is reluctant to take off his shirt. Physical examination shows bilateral , 1-cm, mildly tender, rubbery
nodules palpable under the areolar region. Sexual development is Tanner stage 3. After empathizing with the patient about how he may be embarrassed by his condition , it is most
appropriate for the physician to state which of the following?

A) "Most boys experience this between the ages of 9 and 10 years."


B) "This will typically resolve within the next 12 to 18 months."
u C) "We need to do a few blood tests to rule out hormonal disorders."
U D) "You'll gradually get more comfortable with your own body."
E) "You're the only one who really notices it."

, ~ ~ p , ,.,,.
Previous Next Lab Values Calculator Review Help Pause
hi Comprehensive Basic Science Self-Assessment - Google Chrome x
i starttest.com/ITDVersions/19.3.0.1/ITDStart.aspx?SVC=54cd3e5f-e4bf-49d0-8371-ba261251f2a1

Exam Section 3: Item 26 of 50 National Board of Medical Examiners Time Remaining:


• Mark Comprehensive Basic Science Self-Assessment 4 hr 55 min 32 sec

26. A 3-year-old boy with AIDS develops giant cell pneumonia 3 months after exposure to an unimmunized cousin who had a morbilliform rash , conjunctivitis, rhinitis, and Koplik spots.
The virus responsible for the pneumonia has which of the following types of genomes?

CJ A) Double-stranded DNA
B) Negative-stranded RNA
C) Positive-stranded RNA
U D) Single-stranded DNA

, ~ ~ p , ,.,,.
Previous Next Lab Values Calculator Review Help Pause
hi Comprehensive Basic Science Self-Assessment - Google Chrome x
i starttest.com/ITDVersions/19.3.0.1/ITDStart.aspx?SVC=54cd3e5f-e4bf-49d0-8371-ba261251f2a1

Exam Section 3: Item 27 of 50 National Board of Medical Examiners Time Remaining:


• Mark Comprehensive Basic Science Self-Assessment 4 hr 55 min 25 sec

27. A 17-year-old girl is brought to the hospital by ambulance 45 minutes after sustaining a closed-head injury during a motor vehicle collision. She is in a coma. Physical examination
shows multiple bleeding lacerations over the head. An MRI of the brain shows a 4-cm, right-sided intracranial hematoma with possible brain-stem herniation. Emergency craniotomy
is indicated, but several attempts to contact the patient's parents to obtain permission for the procedure are unsuccessful. The physician decides to proceed with the operation
without permission. This decision by the physician is most consistent with which of the following ethical principles?

A) Beneficence
B) Common good
u C) Integrity
U D) Nonmaleficence
E) Respect for autonomy

, ~ ~ p , ,.,,.
Previous Next Lab Values Calculator Review Help Pause
hi Comprehensive Basic Science Self-Assessment - Google Chrome x
i starttest.com/ITDVersions/19.3.0.1/ITDStart.aspx?SVC=54cd3e5f-e4bf-49d0-8371-ba261251f2a1

Exam Section 3: Item 28 of 50 National Board of Medical Examiners Time Remaining:


• Mark Comprehensive Basic Science Self-Assessment 4 hr 55 min 20 sec

28. An obese 57-year-old man comes to the physician for a routine examination. He has smoked 2Y2 packs of cigarettes daily for 40 years. Physical examination shows mild dyspnea. A
chest x-ray shows a mass in one lobe of the right lung. Examination of a biopsy specimen obtained on bronchoscopy shows squamous metaplasia of the bronchial mucosa. Which
of the following best describes the changes in this patient's bronchial mucosa?

A) Activation of c-Fos resulting in increased proliferation of atypical squamous epithelial cells


B) Chronic irritation leading to decreased blood flow and atrophy of the mucosal lining
C) Irreversible change in the basal cell layer leading to development of abnormal squamous epithelial cells
D) Respiratory epithelial hyperplasia with compression into a flattened squamous pattern
E) Normal ciliated columnar epithelium replaced by normal squamous epithelium

, ~ ~ p , ,.,,.
Previous Next Lab Values Calculator Review Help Pause
hi Comprehensive Basic Science Self-Assessment - Google Chrome x
i starttest.com/ITDVersions/19.3.0.1/ITDStart.aspx?SVC=54cd3e5f-e4bf-49d0-8371-ba261251f2a1

Exam Section 3: Item 29 of 50 National Board of Medical Examiners Time Remaining:


• Mark Comprehensive Basic Science Self-Assessment 4 hr 55 min 14 sec

29. A 58-year-old man comes to the physician because of a 3-day history of progressive malaise, increased urinary frequency, dribbling of urine, a feeling of incomplete bladder
emptying, and deep, dull pelvic pain. He has not had scrotal or testicular pain or blood in his urine, and there has been no trauma. He now urinates once hourly. He has a history of
mild asthma well controlled with inhaled albuterol as needed. He takes no other medications. He does not smoke, drink alcohol, or use illicit drugs. He is sexually active with one
female partner. The penis and scrotum appear normal. Digital rectal examination shows an enlarged, exquisitely tender prostate. Which of the following infectious agents is the most
likely cause of these findings?

A) Chlamydia trachomatis
B) Escherichia coli
C) Mumps virus
lJ D) Neisseria gonorrhoeae
U E) Ureaplasma urealyticum

, ~ ~ p , ,.,,.
Previous Next Lab Values Calculator Review Help Pause
hi Comprehensive Basic Science Self-Assessment - Google Chrome x
i starttest.com/ITDVersions/19.3.0.1/ITDStart.aspx?SVC=54cd3e5f-e4bf-49d0-8371-ba261251f2a1

Exam Section 3: Item 30 of 50 National Board of Medical Examiners Time Remaining:


• Mark Comprehensive Basic Science Self-Assessment 4 hr 55 min 9 sec

30. An 80-year-old woman , gravida 4, para 4 , comes to the office because of a 6-month history of decreased frequency of bowel movements. She now has one or two bowel
movements weekly. Increased fluid and dietary fiber intake has not resulted in relief. She also reports a sensation of incomplete evacuation of her bowels and having to manually
support the posterior vaginal wall in order to defecate. Colonoscopy at the age of 75 years showed extensive diverticula. Digital rectal examination shows no evidence of
gastrointestinal bleeding, fissure, mass, or hemorrhoids. Rectal tone and reflexes are normal. Which of the following is the most likely cause of this patient's symptoms?

A) Absence of ganglionic cells in the colon


B) Damage to the rectovaginal septum
u C) External anal sphincter spasm
U D) Inhibition of the gastrocolic reflex
E) Transmural mucosa! inflammation

, ~ ~ p , ,.,,.
Previous Next Lab Values Calculator Review Help Pause
hi Comprehensive Basic Science Self-Assessment - Google Chrome x
i starttest.com/ITDVersions/19.3.0.1/ITDStart.aspx?SVC=54cd3e5f-e4bf-49d0-8371-ba261251f2a1

Exam Section 3: Item 31 of 50 National Board of Medical Examiners Time Remaining:


• Mark Comprehensive Basic Science Self-Assessment 4 hr 55 min 4 sec

31. A 29-year-old man is brought to the emergency department because of a 1-hour history of bilateral jaw pain that began immediately after he tried to bite into a th ick double
cheeseburger. Physical examination shows excessive drooling and an inability to elevate the mandible. An x-ray of the skull shows bilateral anterior dislocation of the
temporomandibular joints. Reduction of the temporomandibular joints is recommended. Relaxation of which of the following muscles is most likely to facilitate this procedure?

A) Anterior belly of digastric


B) Buccinator
C) Lateral pterygoid
D) Levator veli palatini
E) Mylohyoid

, ~ ~ p , ,.,,.
Previous Next Lab Values Calculator Review Help Pause
hi Comprehensive Basic Science Self-Assessment - Google Chrome x
i starttest.com/ITDVersions/19.3.0.1/ITDStart.aspx?SVC=54cd3e5f-e4bf-49d0-8371-ba261251f2a1

Exam Section 3: Item 32 of 50 National Board of Medical Examiners Time Remaining:


• Mark Comprehensive Basic Science Self-Assessment 4 hr 54 min 57 sec

32. A 3-month-old boy is brought to the physician because of yellow eyes and skin and weakness since birth. Physical examination shows jaundice, large fontanels, a flat midfacial area,
hypotonia, and hepatomegaly. Serum studies show:
Total bilirubin (mainly direct) increased
AST increased
ALT increased
Very-long-chain fatty acids increased

A liver biopsy specimen shows foamy, lipid-filled hepatocytes, necrosis, and absence of a specific organelle. This organelle is most likely which of the following?

u A) Golgi complex
B) Lysosomes
C) Mitochondria
u D) Peroxisomes
E) Smooth endoplasmic reticulum

, ~ ~ p , ,.,,.
Previous Next Lab Values Calculator Review Help Pause
hi Comprehensive Basic Science Self-Assessment - Google Chrome x
i starttest.com/ITDVersions/19.3.0.1/ITDStart.aspx?SVC=54cd3e5f-e4bf-49d0-8371-ba261251f2a1

Exam Section 3: Item 33 of 50 National Board of Medical Examiners Time Remaining:


• Mark Comprehensive Basic Science Self-Assessment 4 hr 54 min 52 sec

33. A 25-year-old man comes to the emergency department because of a 3-day history of abdominal cramps and diarrhea. He appears anxious. Physical examination shows
piloerection and diffuse abdominal tenderness. Neurologic examination shows no abnormalities. He is oriented to person, place, and time. The most likely cause of this patient's
condition is withdrawal from which of the following substances?

A) Alcohol
B) Benzodiazepine
C) Cocaine
D) Heroin
E) Marijuana

, ~ ~ p , ,.,,.
Previous Next Lab Values Calculator Review Help Pause
hi Comprehensive Basic Science Self-Assessment - Google Chrome x
i starttest.com/ITDVersions/19.3.0.1/ITDStart.aspx?SVC=54cd3e5f-e4bf-49d0-8371-ba261251f2a1

Exam Section 3: Item 33 of 50 National Board of Medical Examiners


• Mark Comprehensive Basic Science Self-Assessment Please Walt

34. A 35-year-old woman has difficulty urinating 2 days postpartum. She has a long history of asthma. Which of the following mechanisms is both an indication and a contraindication
for bethanechol in this patient?

A) r3-Adrenergic antagonism
B) Anticholinesterase action
C) Histaminergic stimulation
D) Nicotinic antagonism
E) Parasympathomimetic stimulation

, ~ ~ p , ,.,,.
Previous Next Lab Values Calculator Review Help Pause
hi Comprehensive Basic Science Self-Assessment - Google Chrome x
i starttest.com/ITDVersions/19.3.0.1/ITDStart.aspx?SVC=54cd3e5f-e4bf-49d0-8371-ba261251f2a1

Exam Section 3: Item 35 of 50 National Board of Medical Examiners Time Remaining:


• Mark Comprehensive Basic Science Self-Assessment 4 hr 54 min 36 sec

35. A 75-year-old man with a 10-year history of progressive renal failure comes to the physician for a follow-up examination. Laboratory studies show a serum urea nitrogen
concentration of 40 mg/dl, and a serum creatinine concentration of 3 mg/dl. Ultrasonography of the urinary tract shows a solitary hydronephrotic kidney and a dilated ureter. The
most likely cause of this patient's renal failure is an increase in which of the following?

A) Hydrostatic pressure in Bowman space


B) Hydrostatic pressure in glomerular capillaries
C) Hydrostatic pressure in peritubular capillaries
D) Oncotic pressure in peritubular capillaries
E) Oncotic pressure in renal tubules

, ~ ~ p , ,.,,.
Previous Next Lab Values Calculator Review Help Pause
hi Comprehensive Basic Science Self-Assessment - Google Chrome x
i starttest.com/ITDVersions/19.3.0.1/ITDStart.aspx?SVC=54cd3e5f-e4bf-49d0-8371-ba261251f2a1

Exam Section 3: Item 36 of 50 National Board of Medical Examiners Time Remaining:


• Mark Comprehensive Basic Science Self-Assessment 4 hr 54 min 29 sec

36. A 55-year-old man is diagnosed with coronary artery disease. He begins treatment with 81-mg aspirin to prevent thrombus formation and vessel occlusion in areas of injured
vascular endothelium caused by atherosclerosis. Which of the following effects of aspirin on platelet function is most likely to decrease this patient's risk for thrombosis?

CJ A) Accelerated breakdown of vascular endothelial growth factors


B) Decreased adherence
C) Inhibition of glycoprotein Ilb/Illa receptors
U D) Potentiation of contraction by thrombosthenin
E) Release of calcium ions from the endoplasmic reticulum

, ~ ~ p , ,.,,.
Previous Next Lab Values Calculator Review Help Pause
hi Comprehensive Basic Science Self-Assessment - Google Chrome x
i starttest.com/ITDVersions/19.3.0.1/ITDStart.aspx?SVC=54cd3e5f-e4bf-49d0-8371-ba261251f2a1

Exam Section 3: Item 37 of 50 National Board of Medical Examiners Time Remaining:


• Mark Comprehensive Basic Science Self-Assessment 4 hr 54 min 23 sec

37. A 35-year-old man is brought to the emergency department because he is disoriented and hallucinating. He has a 20-year history of alcoholism. On admission to the hospital, he has
a seizure. His blood pressure is 180/ 100 mm Hg. Serum potassium concentration is 2.5 mEq/L, and urine potassium concentration is 40 mEq/L. Alcohol withdrawal is suspected.
Which of the following is the most likely cause of the hypokalemia?

A) Catecholamine-mediated intracellular shifts of K +


B) Decreased dietary intake of K+
C) Decreased release of renin
D) Decreased urine flow rate
E) Increased muscle breakdown

, ~ ~ p , ,.,,.
Previous Next Lab Values Calculator Review Help Pause
hi Comprehensive Basic Science Self-Assessment - Google Chrome x
i starttest.com/ITDVersions/19.3.0.1/ITDStart.aspx?SVC=54cd3e5f-e4bf-49d0-8371-ba261251f2a1

Exam Section 3: Item 38 of 50 National Board of Medical Examiners Time Remaining:


• Mark Comprehensive Basic Science Self-Assessment 4 hr 54 min 18 sec

38. A 35-year-old man with small cell carcinoma of the lung has systemic hypertension and hypokalemia. The most likely cause of these findings is ectopic secretion of which of the
following hormones?

CJ A) ACTH
B) ADH (vasopressin)
C) Epinephrine
U D) Parathyroid hormone-related peptide
E) Vasoactive intestinal polypeptide

, ~ ~ p , ,.,,.
Previous Next Lab Values Calculator Review Help Pause
hi Comprehensive Basic Science Self-Assessment - Google Chrome x
i starttest.com/ITDVersions/19.3.0.1/ITDStart.aspx?SVC=54cd3e5f-e4bf-49d0-8371-ba261251f2a1

Exam Section 3: Item 39 of 50 National Board of Medical Examiners Time Remaining:


• Mark Comprehensive Basic Science Self-Assessment 4 hr 54 min 11 sec

39. A 57-year-old man has had hoarseness and difficulty swallowing for 2 days. He is unable to elevate the right side of the palate. Which of the following labeled cranial nerves in the
photograph of the brain stem is most likely damaged?

U A) B) C) D) E) F)

, ~ ~ p , ,.,,.
Previous Next Lab Values Calculator Review Help Pause
hi Comprehensive Basic Science Self-Assessment - Google Chrome x
i starttest.com/ITDVersions/19.3.0.1/ITDStart.aspx?SVC=54cd3e5f-e4bf-49d0-8371-ba261251f2a1

Exam Section 3: Item 40 of 50 National Board of Medical Examiners Time Remaining:


• Mark Comprehensive Basic Science Self-Assessment 4 hr 54 min 6 sec

40. A 2-year-old boy is brought to the physician because of chronic bacterial respiratory infections since birth. He is currently asymptomatic. Physical examination shows no
abnormalities. T- and B-lymphocyte counts and serum antibody concentrations are within the reference ranges. Natural killer cell count and function are normal. Analysis of cellular
expression of human leukocyte antigen by flow cytometry shows absence of class I MHC-expressing cells. A diagnosis of bare lymphocyte syndrome, type I, is made. This patient
most likely has mutations in the genes encoding which of the following?

A) Adenosine deaminase
B) Fas ligand (CD178)
u C) lnterleukin-2 (IL-2) receptor a chain (CD25)
U D) Peptide transporter (TAP)

, ~ ~ p , ,.,,.
Previous Next Lab Values Calculator Review Help Pause
hi Comprehensive Basic Science Self-Assessment - Google Chrome x
i starttest.com/ITDVersions/19.3.0.1/ITDStart.aspx?SVC=54cd3e5f-e4bf-49d0-8371-ba261251f2a1

Exam Section 3: Item 41 of 50 National Board of Medical Examiners Time Remaining:


• Mark Comprehensive Basic Science Self-Assessment 4 hr 54 min 1 sec

41. A 50-year-old woman with HIV infection comes to the physician for a follow-up examination. For the past 6 months, she has been receiving antiretroviral therapy that includes the
nucleoside analogues zidovudine (AZT) and lamivudine (3TC), as well as the protease inhibitor nelfinavir. She is also receiving prophylaxis with pentamidine for pneumocystosis and
azithromycin for Mycobacterium avium complex. Her leukocyte count 2 weeks ago was 1200/mm 3 (50% segmented neutrophils). Her plasma HIV viral load remains undetectable.
Drug-induced bone marrow suppression is suspected. Which of the following drugs is the most likely cause?

A) Azithromycin
B) Lamivudine
U C) Nelfinavir
U D) Pentamidine
E) Zidovudine

, ~ ~ p , ,.,,.
Previous Next Lab Values Calculator Review Help Pause
hi Comprehensive Basic Science Self-Assessment - Google Chrome x
i starttest.com/ITDVersions/19.3.0.1/ITDStart.aspx?SVC=54cd3e5f-e4bf-49d0-8371-ba261251f2a1

Exam Section 3: Item 42 of 50 National Board of Medical Examiners Time Remaining:


• Mark Comprehensive Basic Science Self-Assessment 4 hr 53 min 56 sec

42. A 65-year-old man comes to the physician because of difficulty sleeping for the past month. He says, "I have been awakening between 3 and 4 AM every morning and I can't get
back to sleep." He has impaired concentration and a decreased energy level. He used to read a lot and play cards with friends, but he has lost interest in these activities. He has lost
10 kg (22 lb) over the past 3 months, and he has had recurrent thoughts of death since having a myocardial infarction and undergoing a five-vessel coronary bypass 5 months ago.
His current medications include aspirin, metoprolol, and lisinopril. An ECG shows a heart rate of 72/min with a PR interval of 0.26 sec (N =0.12-0.20 sec), a ORS interval of 0.08 sec
(N <0.12 sec), and Q waves in leads II, Ill , and AVF. Which of the following drugs is most appropriate to add to this patient's regimen?

A) Alprazolam
B) Amitriptyline
C) Buspirone
lJ D) Carbamazepine
U E) Haloperidol
U F) Methylphenidate
G) Paroxetine

, ~ ~ p , ,.,,.
Previous Next Lab Values Calculator Review Help Pause
hi Comprehensive Basic Science Self-Assessment - Google Chrome x
i starttest.com/ITDVersions/19.3.0.1/ITDStart.aspx?SVC=54cd3e5f-e4bf-49d0-8371-ba261251f2a1

Exam Section 3: Item 42 of 50 National Board of Medical Examiners


• Mark Comprehensive Basic Science Self-Assessment Please Walt

43. In a culture of motile spore-forming bacteria, which of the following inhibits bacterial growth by causing double-stranded breaks in DNA?

A) Addition of phenol
B) Addition of streptomycin
C) Irradiation with ultraviolet light
D) Irradiation with x-rays
E) Starvation for a carbon source

, ~ ~ p , ,.,,.
Previous Next Lab Values Calculator Review Help Pause
hi Comprehensive Basic Science Self-Assessment - Google Chrome x
i starttest.com/ITDVersions/19.3.0.1/ITDStart.aspx?SVC=54cd3e5f-e4bf-49d0-8371-ba261251f2a1

Exam Section 3: Item 44 of 50 National Board of Medical Examiners Time Remaining:


• Mark Comprehensive Basic Science Self-Assessment 4 hr 53 min 46 sec

44. A 3-year-old boy has short stature and extremities and a relatively normal-sized trunk, a large head with a prominent forehead and low nasal bridge, and exaggerated lordosis. The
most likely cause of these findings is a genetic abnormality in which of the following?

CJ A) Calcium uptake
B) Endochondral ossification
C) Growth hormone synthesis
U D) Osteoclast activity
E) Tendon formation

, ~ ~ p , ,.,,.
Previous Next Lab Values Calculator Review Help Pause
hi Comprehensive Basic Science Self-Assessment - Google Chrome x
i starttest.com/ITDVersions/19.3.0.1/ITDStart.aspx?SVC=54cd3e5f-e4bf-49d0-8371-ba261251f2a1

Exam Section 3: Item 45 of 50 National Board of Medical Examiners Time Remaining:


• Mark Comprehensive Basic Science Self-Assessment 4 hr 53 min 40 sec

45. A 73-year-old man has an incurable malignant neoplasm of the lung, and his condition is slowly deteriorating. He is virtually incapable of movement and unable to breathe without
mechanical respiration ; however, he remains mentally competent. He requests that the respirator be removed, indicating that he no longer wants to endure the constant suffering.
His children want the respirator to be continued. After discussing the matter with the children, the physician chooses to order removal of the respirator. Which of the following best
describes the physician's action?

A) Legal but not ethical


B) Ethical but not legal
u C) Both legal and ethical
U D) Neither legal nor ethical

, ~ ~ p , ,.,,.
Previous Next Lab Values Calculator Review Help Pause
hi Comprehensive Basic Science Self-Assessment - Google Chrome x
i starttest.com/ITDVersions/19.3.0.1/ITDStart.aspx?SVC=54cd3e5f-e4bf-49d0-8371-ba261251f2a1

Exam Section 3: Item 46 of 50 National Board of Medical Examiners Time Remaining:


• Mark Comprehensive Basic Science Self-Assessment 4 hr 53 min 30 sec

46. In which of the following stages of the cell cycle are mitotic cyclins synthesized?

A) G0
U B) G1
U C) S
D) G2
E) M

, ~ ~ p , ,.,,.
Previous Next Lab Values Calculator Review Help Pause
hi Comprehensive Basic Science Self-Assessment - Google Chrome x
i starttest.com/ITDVersions/19.3.0.1/ITDStart.aspx?SVC=54cd3e5f-e4bf-49d0-8371-ba261251f2a1

Exam Section 3: Item 47 of 50 National Board of Medical Examiners Time Remaining:


• Mark Comprehensive Basic Science Self-Assessment 4 hr 53 min 25 sec

47. A 40-year-old man who goes horseback riding 3 to 4 times weekly develops a painful swollen mass in his left inner thigh. Over the next 2 weeks, the mass becomes circumscribed
and very firm. Which of the following is the most likely diagnosis?

CJ A) Arteriovenous fistula
B) Dermatomyositis
C) Fasciitis
U D) Ganglion
E) Hemangioma
F) Myositis ossificans
G) Sarcoma
H) Synovial cyst

, ~ ~ p , ,.,,.
Previous Next Lab Values Calculator Review Help Pause
hi Comprehensive Basic Science Self-Assessment - Google Chrome x
i starttest.com/ITDVersions/19.3.0.1/ITDStart.aspx?SVC=54cd3e5f-e4bf-49d0-8371-ba261251f2a1

Exam Section 3: Item 48 of 50 National Board of Medical Examiners Time Remaining:


• Mark Comprehensive Basic Science Self-Assessment 4 hr 53 min 19 sec

48. A 3-week-old female newborn is brought to the physician for a follow-up examination after the results of newborn screening showed an increased serum concentration of
immunoreactive trypsin. Cystic fibrosis is suspected. At 4 months of age, her sweat chloride concentration is greater than 60 mmol/L (N<40). Molecular analysis that includes a panel
of the 70 most common cystic fibrosis transmembrane gene mutations is done. Results show a mutation in one allele. Which of the following best explains the findings in this
patient?

A) The immunoreactive trypsin test result is a false positive


B) The patient has another mutation that was not included in the previous analysis
u C) The patient has a different disorder that is a phenocopy of cystic fibrosis
U D) The patient has a mutation in another gene for a protein that interacts with CFTR
E) The sweat chloride test result is a false positive

, ~ ~ p , ,.,,.
Previous Next Lab Values Calculator Review Help Pause
hi Comprehensive Basic Science Self-Assessment - Google Chrome x
i starttest.com/ITDVersions/19.3.0.1/ITDStart.aspx?SVC=54cd3e5f-e4bf-49d0-8371-ba261251f2a1

Exam Section 3: Item 49 of 50 National Board of Medical Examiners Time Remaining:


• Mark Comprehensive Basic Science Self-Assessment 4 hr 53 min 14 sec

49. In a study of drug action on neoplastic cells in culture , drug X markedly inhibits cell replication. A microscopic view of a typical cell
incubated with drug Xis shown. Drug Xis most likely to be which of the following?

A) Cyclophosphamide
B) Cyclosporine
C) Doxorubicin
CJ D) 5-Fluorouracil
E) Vincristine

, ~ ~ p , ,.,,.
Previous Next Lab Values Calculator Review Help Pause
hi Comprehensive Basic Science Self-Assessment - Google Chrome x
i starttest.com/ITDVersions/19.3.0.1/ITDStart.aspx?SVC=54cd3e5f-e4bf-49d0-8371-ba261251f2a1

Exam Section 3: Item 50 of 50 National Board of Medical Examiners Time Remaining:


• Mark Comprehensive Basic Science Self-Assessment 4 hr 53 min 7 sec

50. A 27-year-old woman comes to the physician for a follow-up examination. She underwent a cesarean delivery 2 months ago. Physical examination shows a firm, subcutaneous
nodule adjacent to the surgical incision. Microscopic examination of the resected nodule shows fibrous connective tissue, macrophages, multinucleated giant cells, fibroblasts, a few
lymphocytes, and scattered fragments of polarizable foreign material. Which of the following substances that promotes fibroblast migration and proliferation most likely led to the
development of this lesion?

A) C3a
B) lnterleukin-1 (IL-1)
U C) IL-4
U D) Transforming growth factor-13
E) Tumor necrosis factor

, ~ ~ p , ,.,,.
Previous Next Lab Values Calculator Review Help Pause
hi Comprehensive Basic Science Self-Assessment - Google Chrome x
i starttest.com/ITDVersions/19.3.0.1/ITDStart.aspx?SVC=54cd3e5f-e4bf-49d0-8371-ba261251f2a1

Exam Section 4: Item 1 of 50 National Board of Medical Examiners Time Remaining:


• Mark Comprehensive Basic Science Self-Assessment 4 hr 59 min 29 sec

1. A 15-year-old girl with cystic fibrosis has a mutation in the cystic fibrosis transmembrane regulator (CFTR) gene, which results in deletion of phenylalanine 508. This mutation does
not prevent synthesis of the CFTR protein but does prevent it from folding properly. The improperly folded CFTR protein will accumulate in which of the following cellular
compartments?

A) Cytosol
B) Endoplasmic reticulum
C) Nucleus
D) Peroxisome
E) Secretory granules

~ ~ p , ,.,,.
Next Lab Values Calculator Review Help Pause
hi Comprehensive Basic Science Self-Assessment - Google Chrome x
i starttest.com/ITDVersions/19.3.0.1/ITDStart.aspx?SVC=54cd3e5f-e4bf-49d0-8371-ba261251f2a1

Exam Section 4: Item 2 of 50 National Board of Medical Examiners Time Remaining:


• Mark Comprehensive Basic Science Self-Assessment 4 hr 59 min 20 sec

2. A 22-year-old woman comes to the physician for a follow-up examination. She has a 5-year history of migraines. She began taking an oral contraceptive 6 months ago, but she
stopped the contraceptive 1 month ago because it resulted in increased frequency and severity of her migraines. The headaches have since improved. She does not want to try other
forms of hormonal birth control, and she asks about the most effective alternative. It is most appropriate for the physician to recommend which of the following contraceptive methods
for this patient?

A) Contraceptive sponge
B) Diaphragm
u C) Intrauterine device
U D) Rhythm method
E) Spermicidal foam

, ~ ~ p , ,.,,.
Previous Next Lab Values Calculator Review Help Pause
hi Comprehensive Basic Science Self-Assessment - Google Chrome x
i starttest.com/ITDVersions/19.3.0.1/ITDStart.aspx?SVC=54cd3e5f-e4bf-49d0-8371-ba261251f2a1

Exam Section 4: Item 3 of 50 National Board of Medical Examiners Time Remaining:


• Mark Comprehensive Basic Science Self-Assessment 4 hr 59 min 15 sec

3. An investigator is studying a new virus isolated from a 4-year-old girl with fever and cough. Initial experiments show that the virus forms plaques on cultured human laryngeal cells but
rapidly loses its ability to form plaques when dried or exposed to a pH of 5.0. This infectious agent is most likely similar to which of the following viruses?

CJ A) Coronavirus
B) Coxsackievirus
C) Epstein-Barr virus
U D) Norovirus
E) Rotavirus

, ~ ~ p , ,.,,.
Previous Next Lab Values Calculator Review Help Pause
hi Comprehensive Basic Science Self-Assessment - Google Chrome x
i starttest.com/ITDVersions/19.3.0.1/ITDStart.aspx?SVC=54cd3e5f-e4bf-49d0-8371-ba261251f2a1

Exam Section 4: Item 4 of 50 National Board of Medical Examiners Time Remaining:


• Mark Comprehensive Basic Science Self-Assessment 4 hr 59 min 9 sec

4. An otherwise healthy 35-year-old man sustains a fracture of the left pelvis in a motor vehicle collision. A 3-week period of bed rest for this patient is most likely to cause which of the
following physiologic changes?

CJ A) Decreased blood volume


B) Decreased plasma sodium concentration
C) Increased plasma aldosterone concentration
U D) Increased plasma volume
E) Increased sympathetic nerve activity

, ~ ~ p , ,.,,.
Previous Next Lab Values Calculator Review Help Pause
hi Comprehensive Basic Science Self-Assessment - Google Chrome x
i starttest.com/ITDVersions/19.3.0.1/ITDStart.aspx?SVC=54cd3e5f-e4bf-49d0-8371-ba261251f2a1

Exam Section 4: Item 5 of 50 National Board of Medical Examiners Time Remaining:


• Mark Comprehensive Basic Science Self-Assessment 4 hr 59 min 3 sec

5. A 23-year-old primigravid woman at 22 weeks' gestation is brought to the emergency department at a small rural hospital by her husband because of intermittent bleeding during the
past week. The patient woke up this morning with a copious amount of vaginal bleeding and severe pelvic pain. She has noticed no fetal movements for the past 24 hours. Her pulse
is 120/min, and blood pressure is 90/60 mm Hg. Her obstetrician is a woman whose practice is 2 hours away. The patient is dressed in a burka and explains that she is a conservative
Muslim. She does not want to be examined by a male physician. However, the only physician available in the emergency department is a man. Which of the following is the most
appropriate action by the physician?

A) Ask the patient if she would allow the examination if her husband is present at all times
B) Have a female nurse examine the patient and report her findings to the physician
C) Obtain an emergency court order to examine the patient
lJ D) Request that the hospital chaplain speak with the patient
U E) Transport the patient to a major medical facility 2 hours away where she can be examined by her female obstetrician

, ~ ~ p , ,.,,.
Previous Next Lab Values Calculator Review Help Pause
hi Comprehensive Basic Science Self-Assessment - Google Chrome x
i starttest.com/ITDVersions/19.3.0.1/ITDStart.aspx?SVC=54cd3e5f-e4bf-49d0-8371-ba261251f2a1

Exam Section 4: Item 6 of 50 National Board of Medical Examiners Time Remaining:


• Mark Comprehensive Basic Science Self-Assessment 4 hr 58 min 58 sec

6. A 37-year-old man who is a farmworker comes to the emergency department because of a 12-hour history of severe pain in his abdomen and legs and painful spasms of his jaw.
Physical examination shows marked spasms of the masseter and abdominal musculature. During the examination, a loud noise in the examination room triggers painful spasms and
respiratory compromise requiring intubation. Administration of antitoxin prevents further symptoms, but the patient continues to require sedation and venti latory support for the next
3 weeks. This patient most likely has a syndrome that involves binding of a toxin to which of the following?

A) Acetylcholinesterase
B) N-Acetylneuraminic acid
u C) Calmodulin
U D) Monamine oxidase
E) Synaptobrevin
0 F) Ubiquitin

, ~ ~ p , ,.,,.
Previous Next Lab Values Calculator Review Help Pause
Comprehensive Basic Science Self-Assessment - Google Chrome x
i starttest.com/ITDVersions/19.3.0.1/ITDStart.aspx?SVC=54cd3e5f-e4bf-49d0-8371-ba261251f2a1

Exam Section 4: Item 6 of 50 National Board of Medical Examiners


• Mark Comprehensive Basic Science Self-Assessment Please Walt

7. A 43-year-old woman comes to the physician because of a 1-week history of abdominal pain, nausea, vomiting, itching, and fatigue. Physical examination shows scleral icterus and
right upper quadrant abdominal tenderness. There is no rash. Abdominal ultrasonography shows a large stone in the common bile duct. Complete blood count with differential shows
no abnormalities. Serum total bilirubin concentration is markedly increased. The serum concentration of which of the following is most likely to be markedly increased?

A) Acid phosphatase
B) ALT
C) Alkaline phosphatase
D) AST
E) Lactate dehydrogenase
F) Unconjugated bilirubin
hi Comprehensive Basic Science Self-Assessment - Google Chrome x
i starttest.com/ITDVersions/19.3.0.1/ITDStart.aspx?SVC=54cd3e5f-e4bf-49d0-8371-ba261251f2a1

Exam Section 4: Item 8 of 50 National Board of Medical Examiners Time Remaining:


• Mark Comprehensive Basic Science Self-Assessment 4 hr 58 min 51 sec

8. Serum cholesterol concentrations are measured as part of a community study. The means and standard deviations are given for women by age group.
Age (Years) Cholesterol (mg/dl)
45-49 229 ± 47
50-54 246 ± 50
55-59 255 ± 48
60-62 244 ± 36

Assuming serum cholesterol concentrations follow a normal (gaussian) distribution, which of the following is the probability that a woman between the ages of 50 and 54 years has a
serum cholesterol concentration greater than 296 mg/dl?

A) 1%
B) 2.5%
C) 5%
D) 16%
E) 95%

, ~ ~ p , ,.,,.
Previous Next Lab Values Calculator Review Help Pause
hi Comprehensive Basic Science Self-Assessment - Google Chrome x
i starttest.com/ITDVersions/19.3.0.1/ITDStart.aspx?SVC=54cd3e5f-e4bf-49d0-8371-ba261251f2a1

Exam Section 4: Item 9 of 50 National Board of Medical Examiners Time Remaining:


• Mark Comprehensive Basic Science Self-Assessment 4 hr 58 min 46 sec

9. A 20-year-old woman with asthma comes to the physician because of exacerbation of her symptoms during the past 3 months. She says, "I'm able to control my asthma attacks using
my inhaler, but it concerns me that these attacks are happening so often lately." She is a college student who has always carried a heavy course load; she says that it is sometimes
stressful taking so many classes at one time, but she enjoys the challenge. Three months ago, she moved from the campus dormitory into an off-campus apartment with some
friends. She tells the physician, "I'm really happy that I was able to bring my pet poodle to live in the apartment. Sometimes I have disagreements with my roommates because they
never seem to want to clean up after themselves, and one of them kind of annoys me by smoking in the apartment. Also, I think he keeps the apartment too warm. But all in all, I'm
really happy to be there." Physical examination shows no abnormalities. It is most appropriate for the physician to advise that the patient do which of the following?

A) Ask the roommate not to smoke in the apartment


B) Begin treatment with daily prednisone
C) Decrease her course load until the exacerbations decline in frequency
() D) Get an air cleaner for the apartment
u E) Move back to the dormitory
U F) Try to find a new home for the dog
G) Turn down the thermostat to keep the ambient temperature at 65°F

, ~ ~ p , ,.,,.
Previous Next Lab Values Calculator Review Help Pause
hi Comprehensive Basic Science Self-Assessment - Google Chrome x
i starttest.com/ITDVersions/19.3.0.1/ITDStart.aspx?SVC=54cd3e5f-e4bf-49d0-8371-ba261251f2a1

Exam Section 4: Item 10 of 50 National Board of Medical Examiners Time Remaining:


• Mark Comprehensive Basic Science Self-Assessment 4 hr 58 min 42 sec

10. A 22-year-old man is brought to the emergency department 30 minutes after he was involved in a motorcycle coll ision. Physical examination shows dysmetria on the right. Muscle
strength is normal. Which of the following labeled structures in the photograph of the brain is the most likely site of injury in this patient?

v A) B) C) U D) E)

, ~ ~ p , ,.,,.
Previous Next Lab Values Calculator Review Help Pause
hi Comprehensive Basic Science Self-Assessment - Google Chrome x
i starttest.com/ITDVersions/19.3.0.1/ITDStart.aspx?SVC=54cd3e5f-e4bf-49d0-8371-ba261251f2a1

Exam Section 4: Item 11 of 50 National Board of Medical Examiners Time Remaining:


• Mark Comprehensive Basic Science Self-Assessment 4 hr 58 min 36 sec

11. A 54-year-old man with a myocardial infarction becomes progressively hypotensive. Which of the following physiologic events is most likely to cause swelling of parenchymal cells in
his vital organs within minutes?

CJ A) Decrease in intracellular ATP concentrations


B) Decrease in intracellular pH
C) Increase in ribosomal protein synthesis
U D) Large influx of extracellular Ca 2 +
E) Release of osmotically active glucose from glycogen granules

, ~ ~ p , ,.,,.
Previous Next Lab Values Calculator Review Help Pause
hi Comprehensive Basic Science Self-Assessment - Google Chrome x
i starttest.com/ITDVersions/19.3.0.1/ITDStart.aspx?SVC=54cd3e5f-e4bf-49d0-8371-ba261251f2a1

Exam Section 4: Item 12 of 50 National Board of Medical Examiners Time Remaining:


• Mark Comprehensive Basic Science Self-Assessment 4 hr 58 min 33 sec

12. The pedigrees of patients with schizophrenia most closely resemble those of patients with which of the following?

A) Cystic fibrosis
U B) Diabetes mellitus, type 1
U C) Fragile X syndrome
D) Polycystic kidney disease
E) Tay-Sachs disease

, ~ ~ p , ,.,,.
Previous Next Lab Values Calculator Review Help Pause
hi Comprehensive Basic Science Self-Assessment - Google Chrome x
i starttest.com/ITDVersions/19.3.0.1/ITDStart.aspx?SVC=54cd3e5f-e4bf-49d0-8371-ba261251f2a1

Exam Section 4: Item 13 of 50 National Board of Medical Examiners Time Remaining:


• Mark Comprehensive Basic Science Self-Assessment 4 hr 58 min 29 sec

13. A 62-year-old man comes to the physician for a follow-up examination. One month ago, he was prescribed a proton pump inhibitor (PPI) for epigastric pain that is relieved when he
eats. Physical examination shows no abnormalities. The patient says that the medication has only partially relieved his symptoms, and he still has discomfort. Serum studies show a
gastrin concentration four times the reference range. The physician discontinues the PPI therapy in the patient. Three weeks later, additional laboratory studies are ordered. It is
most appropriate to measure the concentration of which of the following at this time?

A) Plasma vasoactive intestinal polypeptide


B) Serum cholecystokinin
u C) Serum gastrin
U D) Serum histamine
E) Serum somatostatin

, ~ ~ p , ,.,,.
Previous Next Lab Values Calculator Review Help Pause
hi Comprehensive Basic Science Self-Assessment - Google Chrome x
i starttest.com/ITDVersions/19.3.0.1/ITDStart.aspx?SVC=54cd3e5f-e4bf-49d0-8371-ba261251f2a1

Exam Section 4: Item 14 of 50 National Board of Medical Examiners Time Remaining:


• Mark Comprehensive Basic Science Self-Assessment 4 hr 58 min 24 sec

14. A 59-year-old woman has a 10-year history of progressive right-sided hearing loss. An MRI of the head shows a large cerebellopontine angle mass that has compressed the
vestibulocochlear nerve on the right. This mass most likely arose from which of the following cell types?

CJ A) Astrocytes
B) Glioblasts
C) Neural crest cells
U D) Neuroblasts
E) Oligodendrocytes

, ~ ~ p , ,.,,.
Previous Next Lab Values Calculator Review Help Pause
hi Comprehensive Basic Science Self-Assessment - Google Chrome x
i starttest.com/ITDVersions/19.3.0.1/ITDStart.aspx?SVC=54cd3e5f-e4bf-49d0-8371-ba261251f2a1

Exam Section 4: Item 15 of 50 National Board of Medical Examiners Time Remaining:


• Mark Comprehensive Basic Science Self-Assessment 4 hr 58 min 19 sec

15. A 32-year-old woman recently diagnosed with AIDS is at greatest risk for developing which of the following neoplasms?

A) Epstein-Barr virus-induced brain lymphoma


U B) Helicobacter pylori-associated gastric lymphoma
U C) Hepatitis B virus-induced hepatocellular carcinoma
D) Hormone-induced endometrial carcinoma
E) Ultraviolet light-induced skin melanoma

, ~ ~ p , ,.,,.
Previous Next Lab Values Calculator Review Help Pause
hi Comprehensive Basic Science Self-Assessment - Google Chrome x
i starttest.com/ITDVersions/19.3.0.1/ITDStart.aspx?SVC=54cd3e5f-e4bf-49d0-8371-ba261251f2a1

Exam Section 4: Item 16 of 50 National Board of Medical Examiners Time Remaining:


• Mark Comprehensive Basic Science Self-Assessment 4 hr 58 min 15 sec

16. A 28-year-old man has a blood pressure cuff placed around his left arm; the cuff is inflated to totally occlude arterial blood flow for 2 minutes and is then removed. Blood flow in the
left arm increases by 50% during the next 3 minutes and then decreases to control values. Which of the following humeral substances is most likely involved?

CJ A) Acetylcholine
B) Adenosine
C) Aldosterone
U D) Epinephrine
E) Norepinephrine
F) Prostacyclin (PGI 2)
G) Prostaglandin F 2 a
H) Serotonin
u I) Thromboxane A 2

, ~ ~ p , ,.,,.
Previous Next Lab Values Calculator Review Help Pause
hi Comprehensive Basic Science Self-Assessment - Google Chrome x
i starttest.com/ITDVersions/19.3.0.1/ITDStart.aspx?SVC=54cd3e5f-e4bf-49d0-8371-ba261251f2a1

Exam Section 4: Item 17 of 50 National Board of Medical Examiners Time Remaining:


• Mark Comprehensive Basic Science Self-Assessment 4 hr 58 min 11 sec

17. A 37-year-old woman with HIV infection is brought to the emergency department because of a 6-hour history of chest and abdominal pain and shortness of breath with exertion. She
is currently receiving antiretroviral therapy. Her pulse is 100/min, respirations are 20/min, and blood pressure is 104/62 mm Hg. Physical examination shows no abnormalities.
Laboratory studies show:
Serum
HC0 3- 16 mEq/L
AST 255 U/L
ALT 198 U/L
Lactate 90 mg/dL (N=9-18)
Arterial blood gas on room air
pH 7.25
Po 2 98 mm Hg

A drug from which of the following classes is the most likely cause of the findings in this patient?

U A) CCR5 receptor antagonist


B) HIV fusion inhibitor
0 C) HIV protease inhibitor
D) lntegrase inhibitor
E) Nucleoside reverse transcriptase inhibitor

, ~ ~ p , ,.,,.
Previous Next Lab Values Calculator Review Help Pause
hi Comprehensive Basic Science Self-Assessment - Google Chrome x
i starttest.com/ITDVersions/19.3.0.1/ITDStart.aspx?SVC=54cd3e5f-e4bf-49d0-8371-ba261251f2a1

Exam Section 4: Item 18 of 50 National Board of Medical Examiners Time Remaining:


• Mark Comprehensive Basic Science Self-Assessment 4 hr 58 min 7 sec

18. A 32-year-old woman comes to the physician because of pain and swelling of her right cheek for 24 hours. Physical examination shows a mildly swollen and tender area overlying
the parotid gland. Sialolithiasis is suspected. The calculus is most likely present in a duct that passes through which of the following muscles to enter the oral cavity?

CJ A) Buccinator
B) Masseter
C) Orbicularis oris
U D) Temporalis
E) Zygomaticus major

, ~ ~ p , ,.,,.
Previous Next Lab Values Calculator Review Help Pause
hi Comprehensive Basic Science Self-Assessment - Google Chrome x
i starttest.com/ITDVersions/19.3.0.1/ITDStart.aspx?SVC=54cd3e5f-e4bf-49d0-8371-ba261251f2a1

Exam Section 4: Item 19 of 50 National Board of Medical Examiners Time Remaining:


• Mark Comprehensive Basic Science Self-Assessment 4 hr 58 min 3 sec

19. A 35-year-old woman with asthma comes to the emergency department because of a 1-week history of abdominal pain and
diarrhea. She also has had progressive cough and wheezing that paradoxically worsened shortly after she began treatment with
oral prednisone for an asthma exacerbation. She recently returned from a trip to Papua New Guinea. Her temperature is 37.8°C
(100°F), pulse is 96/min, respirations are 24/min, and blood pressure is 124/84 mm Hg. Physical examination shows cutaneous
larva currens over the abdomen. A chest x-ray shows bilateral central alveolar infiltrates. Her leukocyte count is 18,000/mm 3 (with
23% eosinophils). Stool examination and analysis of fluid obtained on bronchoalveolar lavage show the presence of the organism
shown in the photomicrograph. Prednisone is discontinued. Which of the following is the most appropriate pharmacotherapy for
this patient?

A) Dexamethasone
u B) Hydroxychloroquine
C) Mefloquine
D) Praziquantel
E) Thiabendazole

, ~ ~ p , ,.,,.
Previous Next Lab Values Calculator Review Help Pause
hi Comprehensive Basic Science Self-Assessment - Google Chrome x
i starttest.com/ITDVersions/19.3.0.1/ITDStart.aspx?SVC=54cd3e5f-e4bf-49d0-8371-ba261251f2a1

Exam Section 4: Item 20 of 50 National Board of Medical Examiners Time Remaining:


• Mark Comprehensive Basic Science Self-Assessment 4 hr 57 min 57 sec

20. A 65-year-old man is brought to the emergency department 30 minutes after the sudden onset of shortness of breath and discomfort in his chest. He says he feels weak and
apprehensive. His pulse is 110/min, respirations are 22/min , and blood pressure is 100/80 mm Hg. Physical examination shows diaphoresis. An ECG shows ST-segment elevation in
the anterior leads. Compared with a healthy man of the same age, which of the following sets of cardiopulmonary changes is most likely in this patient?

Systemic Vascular Pulmonary Vascular Pulmonary Capillary


Resistance Resistance Wedge Pressure
U A) i i !
U B) i ! i
u C) i ! !
D) ! i i
E) ! i !
F) ! ! i

, ~ ~ p , ,.,,.
Previous Next Lab Values Calculator Review Help Pause
hi Comprehensive Basic Science Self-Assessment - Google Chrome x
i starttest.com/ITDVersions/19.3.0.1/ITDStart.aspx?SVC= 701083bd-6fcd-4283-9abd-818a8e3d62c7

Exam Section 4: Item 21 of 50 National Board of Medical Examiners Time Remaining:


• Mark Comprehensive Basic Science Self-Assessment 4 hr 56 min 41 sec

21. A 3-year-old girl has a history of recurrent infections. In vitro, neutrophils isolated from this patient are capable of phagocytosis and can kill Lactobacillus species but not
Staphylococcus aureus. This patient most likely has a defect involving which of the following enzymes?

CJ A) Catalase
B) Elastase
C) Myeloperoxidase
U D) NADPH oxidase
E) Superoxide dismutase

, ~ ~ p , ,.,,.
Previous Next Lab Values Calculator Review Help Pause
hi Comprehensive Basic Science Self-Assessment - Google Chrome x
i starttest.com/ITDVersions/19.3.0.1/ITDStart.aspx?SVC= 701083bd-6fcd-4283-9abd-818a8e3d62c7

Exam Section 4: Item 22 of 50 National Board of Medical Examiners Time Remaining:


• Mark Comprehensive Basic Science Self-Assessment 4 hr 56 min 35 sec

22. A 12-year-old boy is brought to the physician by his mother for a well-child examination. His mother says that he has outgrown the clothes that she bought him 6 months ago. He is
at the 50th percentile for height and weight. Based on his medical record , the physician estimates the patient's growth velocity to be 6 cm (2.3 in)/year. Physical examination shows
mild acne over the face. Pubic hair and testes development are Tanner stage 2. Which of the following is the most likely cause of the onset of the physical changes in this patient?

A) Constant secretion of gonadotropin-releasing hormone (GnRH)


B) Decreased GnRH receptors
C) Decreased secretion of inhibin
D) Increased autonomous testosterone secretion
E) Nocturnal luteinizing hormone pulses

, ~ ~ p , ,.,,.
Previous Next Lab Values Calculator Review Help Pause
hi Comprehensive Basic Science Self-Assessment - Google Chrome x
i starttest.com/ITDVersions/19.3.0.1/ITDStart.aspx?SVC= 701083bd-6fcd-4283-9abd-818a8e3d62c7

Exam Section 4: Item 23 of 50 National Board of Medical Examiners Time Remaining:


• Mark Comprehensive Basic Science Self-Assessment 4 hr 56 min 29 sec

23. A 4-year-old girl has a history of multiple bone fractures and poor wound healing. A photograph of the face is shown. Which of the following components of wound healing is most
likely to be affected as a direct result of her underlying disease?

A) Cell migration
B) Clot formation
C) Granulation tissue deposition
D) Inflammation
E) Scar formation

, ~ ~ p , ,.,,.
Previous Next Lab Values Calculator Review Help Pause
hi Comprehensive Basic Science Self-Assessment - Google Chrome x
i starttest.com/ITDVersions/19.3.0.1/ITDStart.aspx?SVC= 701083bd-6fcd-4283-9abd-818a8e3d62c7

Exam Section 4: Item 24 of 50 National Board of Medical Examiners Time Remaining:


• Mark Comprehensive Basic Science Self-Assessment 4 hr 56 min 22 sec

24. A 17-year-old girl has significant blood loss after being injured in an automobile collision. Blood loss stimulates bone marrow to synthesize which of the following?

A) o-Aminolevulinate
U B) Bilirubin
U C) Erythropoietin
D) Ferritin
E) Uric acid

, ~ ~ p , ,.,,.
Previous Next Lab Values Calculator Review Help Pause
hi Comprehensive Basic Science Self-Assessment - Google Chrome x
i starttest.com/ITDVersions/19.3.0.1/ITDStart.aspx?SVC= 701083bd-6fcd-4283-9abd-818a8e3d62c7

Exam Section 4: Item 25 of 50 National Board of Medical Examiners Time Remaining:


• Mark Comprehensive Basic Science Self-Assessment 4 hr 56 min 17 sec

25. A 1-month-old male newborn is brought to the physician for a routine examination. His parents both have olive-colored skin, dark hair, and dark eyes. Physical examination shows
hypopigmentation of the skin, light blonde hair, and translucent irises. The inherited disorder that causes this phenotypic expression is most likely due to a defect in the metabolism
of which of the following?

A) Epinephrine
B) Phenylalanine
C) Serotonin
D) Tryptophan
E) Tyrosine

, ~ ~ p , ,.,,.
Previous Next Lab Values Calculator Review Help Pause
hi Comprehensive Basic Science Self-Assessment - Google Chrome x
i starttest.com/ITDVersions/19.3.0.1/ITDStart.aspx?SVC= 701083bd-6fcd-4283-9abd-818a8e3d62c7

Exam Section 4: Item 26 of 50 National Board of Medical Examiners Time Remaining:


• Mark Comprehensive Basic Science Self-Assessment 4 hr 56 min 13 sec

26. A 37-year-old man is admitted to the hospital because of gradually progressive weakness, anorexia, and weight loss over the past 6 months. His blood pressure is 74/40 mm Hg and
his skin is hyperpigmented. Morning serum cortisol concentration is 2 µg/dl. Which of the following is the most likely diagnosis?

CJ A) Adrenocortical carcinoma
B) Amyloidosis
C) Autoimmune adrenalitis
U D) Basophilic pituitary adenoma
E) Metastasis to the adrenal gland
F) Pituitary necrosis (Sheehan syndrome)
G) Sarcoidosis
H) Waterhouse-Friderichsen syndrome

, ~ ~ p , ,.,,.
Previous Next Lab Values Calculator Review Help Pause
hi Comprehensive Basic Science Self-Assessment - Google Chrome x
i starttest.com/ITDVersions/19.3.0.1/ITDStart.aspx?SVC= 701083bd-6fcd-4283-9abd-818a8e3d62c7

Exam Section 4: Item 27 of 50 National Board of Medical Examiners Time Remaining:


• Mark Comprehensive Basic Science Self-Assessment 4 hr 56 min 9 sec

27. A 34-year-old woman is brought to the emergency department 45 minutes after she slipped on an icy sidewalk and landed on her outstretched right hand. Examination of the right
upper extremity shows an exquisitely tender, swollen wrist with a palpable hard mass located immediately proximal to the wrist joint anteriorly. X-rays of the right wrist are shown.
Which of the following bones is most likely dislocated in this patient?

A) Capitate
U B) Hamate
U C) Lunate
D) Pisiform
E) Trapezium
F) Trapezoid
U G) Triquetral

, ~ ~ p , ,.,,.
Previous Next Lab Values Calculator Review Help Pause
hi Comprehensive Basic Science Self-Assessment - Google Chrome x
i starttest.com/ITDVersions/19.3.0.1/ITDStart.aspx?SVC= 701083bd-6fcd-4283-9abd-818a8e3d62c7

Exam Section 4: Item 28 of 50 National Board of Medical Examiners Time Remaining:


• Mark Comprehensive Basic Science Self-Assessment 4 hr 56 min 4 sec

28. HOX genes play a role in embryogenesis through which of the following processes?

A) Allelic exclusion
U B) Alternate mRNA splicing
U C) Regulation of transcription
D) Regulation of translation
E) Signal transduction

, ~ ~ p , ,.,,.
Previous Next Lab Values Calculator Review Help Pause
hi Comprehensive Basic Science Self-Assessment - Google Chrome x
i starttest.com/ITDVersions/19.3.0.1/ITDStart.aspx?SVC= 701083bd-6fcd-4283-9abd-818a8e3d62c7

Exam Section 4: Item 29 of 50 National Board of Medical Examiners Time Remaining:


• Mark Comprehensive Basic Science Self-Assessment 4 hr 55 min 59 sec

29. A 35-year-old woman with a long-standing history of asthma treated with corticosteroids dies of complications of histoplasmosis. Examination at autopsy shows that the lungs are
1.5 times the normal weight. Extensive focal areas of fibrosis and 2- to 5-mm nodules are seen throughout both lungs. Examination of a biopsy specimen of the nodules is most
likely to show which of the following findings?

A) Diffuse lgG deposition


B) Immune complex deposition
C) Increased concentration of eosinophils
D) Infiltration of lymphocytes and monocytes
E) Neutrophilic infiltrates

, ~ ~ p , ,.,,.
Previous Next Lab Values Calculator Review Help Pause
hi Comprehensive Basic Science Self-Assessment - Google Chrome x
i starttest.com/ITDVersions/19.3.0.1/ITDStart.aspx?SVC= 701083bd-6fcd-4283-9abd-818a8e3d62c7

Exam Section 4: Item 30 of 50 National Board of Medical Examiners Time Remaining:


• Mark Comprehensive Basic Science Self-Assessment 4 hr 55 min 55 sec

30. A 2-year-old boy is brought to the emergency department because of a 12-hour history of headache, loss of appetite, and vomiting. His temperature is 39.9°C (103.8°F), pulse is
120/min, respirations are 40/min, and blood pressure is 90/50 mm Hg. Physical examination shows nuchal rigidity. A lumbar puncture is done. Cerebrospinal flu id analysis shows an
increased protein concentration, decreased glucose concentration, abundant neutrophils, and gram-positive diplococci. Immunization with which of the following would most likely
have prevented this patient's infection?

A) Envelope glycoprotein
B) Killed bacterial vaccine
u C) Polysaccharide protein conjugate vaccine
U D) Recombinant vaccine
E) Toxoid

, ~ ~ p , ,.,,.
Previous Next Lab Values Calculator Review Help Pause
hi Comprehensive Basic Science Self-Assessment - Google Chrome x
i starttest.com/ITDVersions/19.3.0.1/ITDStart.aspx?SVC= 701083bd-6fcd-4283-9abd-818a8e3d62c7

Exam Section 4: Item 31 of 50 National Board of Medical Examiners Time Remaining:


• Mark Comprehensive Basic Science Self-Assessment 4 hr 55 min 51 sec

31. A 62-year-old man comes to the physician because of a 2-month history of intermittent, severe pain of his left leg that began shortly after the leg was amputated below the knee. The
pain is described as throbbing, aching, and shooting, and is localized to the distal portion of the absent extremity. Each episode lasts from several seconds to a few minutes. The
pain is exacerbated by feelings of anxiety and changes in temperature and is not relieved by changing position. His vital signs are within normal limits. Examination of the left lower
extremity shows a well-healed surgical scar over the stump. Neurologic examination, including muscle strength testing , range of motion, deep tendon reflexes, and muscle tone, is
within normal limits. Which of the following is the most likely cause of this patient's symptoms?

A) Chronic infection of the residual extremity


B) Fibromyalgia
C) Neuralgia
lJ D) Phantom limb pain
U E) Radiculitis

, ~ ~ p , ,.,,.
Previous Next Lab Values Calculator Review Help Pause
hi Comprehensive Basic Science Self-Assessment - Google Chrome x
i starttest.com/ITDVersions/19.3.0.1/ITDStart.aspx?SVC= 701083bd-6fcd-4283-9abd-818a8e3d62c7

Exam Section 4: Item 32 of 50 National Board of Medical Examiners Time Remaining:


• Mark Comprehensive Basic Science Self-Assessment 4 hr 55 min 44 sec

32. A 33-year-old woman comes to the physician because of a 2-month history of easy fatigability and dark urine in the mornings. She has not had fever or weight loss during this
period. She has not traveled internationally. She takes no medications. Her temperature is 36.9°C (98.4 °F), pulse is 107/min, respirations are 20/min , and blood pressure is
121/73 mm Hg. Physical examination shows scleral icterus. Laboratory studies show:
Hemoglobin 9.2 g/dl
Hematocrit 35%
Serum
Bilirubin, total 3.5 mg/dl
Direct 0.2 mg/dl
Indirect 3.3 mg/dl
Urine
Blood 2+
Protein 1+

The result of a direct antiglobulin (Coombs) test is negative, and the result of an acidified serum test is positive. Which of the following best describes the underlying cellular cause of
this patient's condition?

U A) Abnormal cell morphology


B) Decreased glucose 6-phosphate dehydrogenase activity
U C) Defect in a cell membrane anchor protein
D) Destabilization of the cytoskeleton
E) Underproduction of globin proteins

, ~ ~ p , ,.,,.
Previous Next Lab Values Calculator Review Help Pause
hi Comprehensive Basic Science Self-Assessment - Google Chrome x
i starttest.com/ITDVersions/19.3.0.1/ITDStart.aspx?SVC= 701083bd-6fcd-4283-9abd-818a8e3d62c7

Exam Section 4: Item 33 of 50 National Board of Medical Examiners Time Remaining:


• Mark Comprehensive Basic Science Self-Assessment 4 hr 55 min 38 sec

33. A 3-month-old boy is brought to the emergency department because of shortness of breath and listlessness for 3 hours. He has had an upper respiratory tract infection and has
eaten poorly for the past 3 days. His temperature is 37°C (98.6°F), and respirations are 30/min. Physical examination shows lethargy and mild hepatomegaly. Laboratory studies
show hypoglycemia, lactic acidemia, ketonemia , and metabolic acidosis. Following the intravenous administration of glycerol or fructose on different days, his serum glucose
concentrations do not increase. However, they do increase normally after the intravenous administration of galactose. A defect in which of the following liver metabolic pathways is
the most likely cause of these findings?

A) Fatty acid oxidation


B) Gluconeogenesis
C) Glycogen breakdown
lJ D) Glycogen synthesis
U E) Glycolysis

, ~ ~ p , ,.,,.
Previous Next Lab Values Calculator Review Help Pause
hi Comprehensive Basic Science Self-Assessment - Google Chrome x
i starttest.com/ITDVersions/19.3.0.1/ITDStart.aspx?SVC= 701083bd-6fcd-4283-9abd-818a8e3d62c7

Exam Section 4: Item 34 of 50 National Board of Medical Examiners Time Remaining:


• Mark Comprehensive Basic Science Self-Assessment 4 hr 55 min 32 sec

34. An 83-year-old woman is brought to the physician by her daughter to discuss the results of a complete dementia work-up. The patient has had mild memory impairment for
8 months. She takes no medications. Vital signs are normal. Her Mini-Mental State Examination score is 23/30. A rapid plasma reagin is 1:4, and a microhemagglutination assay for
Treponema pallidum is positive. Which of the following is the best next step for the physician?

A) Discussion of the diagnosis with the daughter privately


B) Discussion of the diagnosis with the patient privately
C) Disregarding the results since the patient is too old for treatment
D) Repeated tests
E) Lumbar puncture

, ~ ~ p , ,.,,.
Previous Next Lab Values Calculator Review Help Pause
hi Comprehensive Basic Science Self-Assessment - Google Chrome x
i starttest.com/ITDVersions/19.3.0.1/ITDStart.aspx?SVC= 701083bd-6fcd-4283-9abd-818a8e3d62c7

Exam Section 4: Item 35 of 50 National Board of Medical Examiners Time Remaining:


• Mark Comprehensive Basic Science Self-Assessment 4 hr 55 min 27 sec

35. A 27-year-old man comes to the physician because he and his wife have not been able to conceive a child. He has poor libido and is unable to maintain an erection. He has been
receiving thyroid hormone and corticosteroid replacement therapy since surgical removal of a pituitary adenoma 2 years ago. His serum testosterone concentration is 0.05 nmol/L
(N=10-35). Semen analysis shows azoospermia. Which of the following is the most appropriate treatment to restore this patient's fertility?

A) Injection of gonadotropin-releasing hormone


B) Injections of gonadotropins
C) Oral clomiphene citrate
D) Oral progesterone
E) Testosterone patches

, ~ ~ p , ,.,,.
Previous Next Lab Values Calculator Review Help Pause
hi Comprehensive Basic Science Self-Assessment - Google Chrome x
i starttest.com/ITDVersions/19.3.0.1/ITDStart.aspx?SVC= 701083bd-6fcd-4283-9abd-818a8e3d62c7

Exam Section 4: Item 36 of 50 National Board of Medical Examiners Time Remaining:


• Mark Comprehensive Basic Science Self-Assessment 4 hr 55 min 18 sec

36. A 30-year-old man develops tingling around the lips and mouth after consuming a small portion of fugu (puffer fish) in a Japanese restaurant. The liver of this fish contains
tetrodotoxin, a substance that causes cardiac arrest when consumed in high doses. This patient's symptoms are most likely due to the blocking action of tetrodotoxin on which of the
following ion channels?

A) Calcium
B) Chloride
C) Potassium
D) Sodium

, ~ ~ p , ,.,,.
Previous Next Lab Values Calculator Review Help Pause
hi Comprehensive Basic Science Self-Assessment - Google Chrome x
i starttest.com/ITDVersions/19.3.0.1/ITDStart.aspx?SVC= 701083bd-6fcd-4283-9abd-818a8e3d62c7

Exam Section 4: Item 37 of 50 National Board of Medical Examiners Time Remaining:


• Mark Comprehensive Basic Science Self-Assessment 4 hr 55 min 13 sec

37. A group of physicians submits a report to a medical journal that describes three patients with idiopathic pulmonary fibrosis who developed hepatotoxicity following treatment with a
recently approved drug. In the report, the physicians state that they are unaware of any previous description of this adverse effect. Which of the following best describes the study
design used by these clinicians?

A) Case series
B) Case-control study
C) Clinical trial
D) Correlational study
E) Prospective cohort study
F) Retrospective cohort study

, ~ ~ p , ,.,,.
Previous Next Lab Values Calculator Review Help Pause
hi Comprehensive Basic Science Self-Assessment - Google Chrome x
i starttest.com/ITDVersions/19.3.0.1/ITDStart.aspx?SVC= 701083bd-6fcd-4283-9abd-818a8e3d62c7

Exam Section 4: Item 38 of 50 National Board of Medical Examiners Time Remaining:


• Mark Comprehensive Basic Science Self-Assessment 4 hr 55 min 7 sec

38. A man accidentally touches the surface of a hot stove; 20 minutes later a blister develops at the site. Light and electron microscopy of the inflamed tissue at this time is most likely to
show which of the following?

CJ A) Accumulation of macrophages
B) Fragmentation and hyalinization of dermal collagen
C) lnterendothelial gaps in venules
U D) Necrosis of arterioles
E) Perivascular collections of eosinophils

, ~ ~ p , ,.,,.
Previous Next Lab Values Calculator Review Help Pause
hi Comprehensive Basic Science Self-Assessment - Google Chrome x
i starttest.com/ITDVersions/19.3.0.1/ITDStart.aspx?SVC= 701083bd-6fcd-4283-9abd-818a8e3d62c7

Exam Section 4: Item 39 of 50 National Board of Medical Examiners Time Remaining:


• Mark Comprehensive Basic Science Self-Assessment 4 hr 54 min 20 sec

39. A 43-year-old woman comes to the physician because of a nonproductive cough for 3 weeks. She has had a 6.8-kg (15-lb) weight loss
during the past 3 months. A chest x-ray shows three 0.3- to 1-cm nodules in the right lung. Cytologic examination of fi ne-needle aspirate
from the largest nodule strongly suggests a malignant neoplasm. A photograph representative of the findings in this patient's lungs is
shown. Which of the following is the most likely diagnosis?

A) Carcinoid tumor
U B) Malignant mesothelioma
C) Metastatic carcinoma
U D) Pulmonary hamartoma
E) Small cell carcinoma

, ~ ~ p , ,.,,.
Previous Next Lab Values Calculator Review Help Pause
hi Comprehensive Basic Science Self-Assessment - Google Chrome x
i starttest.com/ITDVersions/19.3.0.1/ITDStart.aspx?SVC= 701083bd-6fcd-4283-9abd-818a8e3d62c7

Exam Section 4: Item 40 of 50 National Board of Medical Examiners Time Remaining:


• Mark Comprehensive Basic Science Self-Assessment 4 hr 54 min 14 sec

40. A cohort study is done to evaluate the association between use of video display terminals (VDTs) by women and the risk for congenital heart disease in their offspring. The relative
risk (risk ratio) of congenital heart disease in newborns born to women who work for 6 or more hours daily using a VDT is 1.1 (95% confidence interval: 0.8-1.4) compared with
women who are not exposed to VDTs. Which of the following is the p-value calculated from a chi square test?

A) 0
B) 0 < p < 0.01
C) 0.01 < p < 0.05
D) 0.05 < p < 1.0
E) p > 1.0

, ~ ~ p , ,.,,.
Previous Next Lab Values Calculator Review Help Pause
hi Comprehensive Basic Science Self-Assessment - Google Chrome x
i starttest.com/ITDVersions/19.3.0.1/ITDStart.aspx?SVC= 701083bd-6fcd-4283-9abd-818a8e3d62c7

Exam Section 4: Item 41 of 50 National Board of Medical Examiners Time Remaining:


• Mark Comprehensive Basic Science Self-Assessment 4 hr 54 min 9 sec

41. Drug Xis given to a 25-year-old normal subject. This drug causes resting heart rate to increase from 62/min to 74/min. Prior to administration of the drug, the subject's heart rate
increased to 150/min with exercise; after administration of Drug X, his heart rate increased to 98/min at the same level of exercise. The mechanism of action of Drug X most likely
involves which of the following?

A) Antagonist at dopaminergic receptors


B) Full agonist at angiotensin II receptors
C) Full agonist at ADH (vasopressin) receptors
D) Partial agonist at 13-adrenergic receptors
E) Partial agonist at serotoninergic receptors

, ~ ~ p , ,.,,.
Previous Next Lab Values Calculator Review Help Pause
hi Comprehensive Basic Science Self-Assessment - Google Chrome x
i starttest.com/ITDVersions/19.3.0.1/ITDStart.aspx?SVC= 701083bd-6fcd-4283-9abd-818a8e3d62c7

Exam Section 4: Item 42 of 50 National Board of Medical Examiners Time Remaining:


• Mark Comprehensive Basic Science Self-Assessment 4 hr 54 min 5 sec

42. Which of the following terms best describes the fibrous proteins that form the two-dimensional network on the inner surface of the nuclear membrane?

A) Actin filaments
U B) Fibronectin
U C) Granum
D) Lamins
E) Tubulin

, ~ ~ p , ,.,,.
Previous Next Lab Values Calculator Review Help Pause
hi Comprehensive Basic Science Self-Assessment - Google Chrome x
i starttest.com/ITDVersions/19.3.0.1/ITDStart.aspx?SVC= 701083bd-6fcd-4283-9abd-818a8e3d62c7

Exam Section 4: Item 43 of 50 National Board of Medical Examiners Time Remaining:


• Mark Comprehensive Basic Science Self-Assessment 4 hr 54 min O sec

43. A 7-year-old girl is brought to the emergency department (ED) because of a 1-hour history of excessive sleepiness and slurring of speech. The mother reports that the girl seemed to
be feeling well earlier in the day and had eaten lunch 2 hours before this episode. Fingerstick blood glucose concentration ordered on arrival in the ED is 42 mg/dl. Intravenous
dextrose is administered, and 5 minutes later her symptoms resolve. The patient has a history of three previous ED visits for similar symptoms during the past year. Her pulse is
94/min, respirations are 24/min , and blood pressure is 102/64 mm Hg. Physical examination shows no abnormalities. Results of serum studies obtained prior to administration of
dextrose are now available and are shown:
C-peptide 0.5 ng/ml (N=0.8-3.1)
Insulin 32 µU/ml (fasting N=5-20)
Beta-hydroxybutyrate 1.2 mg/dl (N=0.2-3.0)

Which of the following is the most likely diagnosis?

A) Factitious disorder imposed on another


u B) Glucose 6-phosphatase deficiency
C) Nesidioblastosis
U D) Pyruvate carboxylase deficiency
lJ E) Type 1 diabetes mellitus

, ~ ~ p , ,.,,.
Previous Next Lab Values Calculator Review Help Pause
hi Comprehensive Basic Science Self-Assessment - Google Chrome x
i starttest.com/ITDVersions/19.3.0.1/ITDStart.aspx?SVC= 701083bd-6fcd-4283-9abd-818a8e3d62c7

Exam Section 4: Item 44 of 50 National Board of Medical Examiners Time Remaining:


• Mark Comprehensive Basic Science Self-Assessment 4 hr 53 min 56 sec

44. A 55-year-old man comes to the physician because of a 2-week history of palpitations and anxiety. He has primary hypothyroidism treated with levothyroxine. He says that he has
been taking twice the prescribed dosage of his medication for the past 2 months. His pulse is 104/min, and blood pressure is 146/88 mm Hg. Physical examination shows a fine
resting tremor. Which of the following sets of laboratory findings is most likely in this patient?

Serum Serum
Free Thyroxine (FT J Free Triiodothyronine (FT :J Thyroidal Iodine Uptake
U A) Increased increased decreased
U B) Increased decreased increased
u C) Increased decreased decreased
D) Decreased increased increased
E) Decreased increased decreased
F) Decreased decreased increased

, ~ ~ p , ,.,,.
Previous Next Lab Values Calculator Review Help Pause
hi Comprehensive Basic Science Self-Assessment - Google Chrome x
i starttest.com/ITDVersions/19.3.0.1/ITDStart.aspx?SVC= 701083bd-6fcd-4283-9abd-818a8e3d62c7

Exam Section 4: Item 45 of 50 National Board of Medical Examiners Time Remaining:


• Mark Comprehensive Basic Science Self-Assessment 4 hr 53 min 51 sec

45. A 55-year-old man with alcoholism is brought to the emergency department 30 minutes after consuming a bottle of methanol. In order to decrease methanol toxicity in this patient,
the physician recommends that the patient be treated with ethanol. To increase the Kmof hepatic alcohol dehydrogenase for methanol in this patient the ethanol must act as which of
the following?

A) Competitive inhibitor
B) Complete agonist
C) Feedback regulator
D) Noncompetitive antagonist
E) Positive allosteric effector

, ~ ~ p , ,.,,.
Previous Next Lab Values Calculator Review Help Pause
hi Comprehensive Basic Science Self-Assessment - Google Chrome x
i starttest.com/ITDVersions/19.3.0.1/ITDStart.aspx?SVC= 701083bd-6fcd-4283-9abd-818a8e3d62c7

Exam Section 4: Item 46 of 50 National Board of Medical Examiners Time Remaining:


• Mark Comprehensive Basic Science Self-Assessment 4 hr 53 min 44 sec

46. A 21-year-old woman with asthma comes to the physician because her current medication regimen is not relieving her symptoms. The physician tel ls the patient about a clinical trial
of a new drug for asthma. This trial is a large, randomized , prospective, double-blind study on volunteers with asthma. This trial is most likely occurring at which of the following
phases of drug development?

A) Phase 0
B) Phase 1
C) Phase 2
D) Phase 3
E) Phase 4

, ~ ~ p , ,.,,.
Previous Next Lab Values Calculator Review Help Pause
hi Comprehensive Basic Science Self-Assessment - Google Chrome x
i starttest.com/ITDVersions/19.3.0.1/ITDStart.aspx?SVC= 701083bd-6fcd-4283-9abd-818a8e3d62c7

Exam Section 4: Item 47 of 50 National Board of Medical Examiners Time Remaining:


• Mark Comprehensive Basic Science Self-Assessment 4 hr 53 min 32 sec

47. A 27-year-old man who works from home as a software engineer comes to the physician with his girlfriend because she is worried that he may be depressed. His girlfriend tells the
physician, "I practically have to drag him out of the house to see my family or friends, and then when we get there he refuses to talk to them or interact with them. He just stands
around looking uncomfortable, and then he asks to go home half an hour after we arrive." She says that he seems fine when he is alone with her or with his own family and friends.
After she leaves the room , he tells the physician, "I wish I could have relationships with her family and friends, but I can't. I just know they'll hate me once they get to know me." He
has been this way for as long as he can remember. This patient most likely has which of the following types of personality disorders?

A) Antisocial
B) Avoidant
C) Paranoid
lJ D) Schizoid
U E) Schizotypal

, ~ ~ p , ,.,,.
Previous Next Lab Values Calculator Review Help Pause
hi Comprehensive Basic Science Self-Assessment - Google Chrome x
i starttest.com/ITDVersions/19.3.0.1/ITDStart.aspx?SVC= 701083bd-6fcd-4283-9abd-818a8e3d62c7

Exam Section 4: Item 48 of 50 National Board of Medical Examiners Time Remaining:


• Mark Comprehensive Basic Science Self-Assessment 4 hr 53 min 28 sec

48. A 38-year-old woman comes to the physician because of blood-tinged discharge from her right breast for 3 months. Menses have occurred at regular 28-day intervals. She takes no
medications. There is no family history of breast cancer. Physical examination shows no breast masses and no palpable axillary adenopathy. Bloody discharge can be expressed
from the upper outer corner of the right nipple. Mammography shows no abnormalities. Which of the following is the most likely cause of the discharge?

A) Fibrocystic changes of the breast


B) Fibrosarcoma
C) lntraductal papilloma
D) Paget disease of the breast
E) Prolactinoma

, ~ ~ p , ,.,,.
Previous Next Lab Values Calculator Review Help Pause
hi Comprehensive Basic Science Self-Assessment - Google Chrome x
i starttest.com/ITDVersions/19.3.0.1/ITDStart.aspx?SVC= 701083bd-6fcd-4283-9abd-818a8e3d62c7

Exam Section 4: Item 49 of 50 National Board of Medical Examiners Time Remaining:


• Mark Comprehensive Basic Science Self-Assessment 4 hr 53 min 23 sec

49. An 18-year-old woman is brought to the physician because of sharp chest pain and shortness of breath 1 hour after
receiving a nonpenetrating injury during a rugby game. Her respirations are 22/min. Physical examination shows
decreased breath sounds and increased tympany to percussion on the right. A chest x-ray is shown. This patient is at
greatest risk for developing which of the following complications?

A) Amyloidosis
U B) Carcinoid tumor
C) Diffuse alveolar damage
U D) Empyema
E) Pulmonary edema
F) Pulmonary embolism
G) Respiratory acidosis

, ~ ~ p , ,.,,.
Previous Next Lab Values Calculator Review Help Pause
hi Comprehensive Basic Science Self-Assessment - Google Chrome x
i starttest.com/ITDVersions/19.3.0.1/ITDStart.aspx?SVC= 701083bd-6fcd-4283-9abd-818a8e3d62c7

Exam Section 4: Item 50 of 50 National Board of Medical Examiners Time Remaining:


• Mark Comprehensive Basic Science Self-Assessment 4 hr 53 min 18 sec

50. A 15-year-old girl is brought to the physician because of a 3-day history of fever, sore throat, and malaise. Her temperature is 39.2°C (102.6°F). Physical examination shows diffuse
pharyngeal erythema, moderately enlarged tonsils, and tender anterior and posterior cervical lymphadenopathy. A complete blood count shows:
Leukocyte count 19,500/mm 3 (N=3500-10,500)
Segmented neutrophils 30%
Bands 7%
Eosinophils 2%
Lymphocytes 25%
Lymphocytes, atypical 30%
Monocytes 6%

Incubation of this patient's serum with sheep erythrocytes results in agglutination. The atypical lymphocytes in this patient are most likely which of the following cell types?

A) B lymphocytes
B) CD4+ T lymphocytes
U C) CD8+ T lymphocytes
D) FOXP3-expressing regulatory T lymphocytes
U E) Natural killer cells

, ~ ~ p , ,.,,.
Previous Next Lab Values Calculator Review Help Pause

You might also like